SlideShare a Scribd company logo
1 of 50
Download to read offline
INSTITUTO COLOMBIANO PARA EL FOMENTO DE LA 
EDUCACIÓN SUPERIOR 
ICFES 
SUBDIRECCIÓN ACADÉMICA 
GRUPO DE EVALUACIÓN DE 
LA EDUCACIÓN BÁSICA Y MEDIA 
MATEMÁTICA 
ANÁLISIS DE RESULTADOS 2006 
EXAMEN DE ESTADO PARA INGRESO A LA EDUCACIÓN 
SUPERIOR 
Myriam Margarita Acevedo Caicedo 
(Universidad Nacional de Colombia) 
María Cristina Pérez Camacho 
(Profesora pensionada Secretaría de 
Educación Distrital de Bogotá) 
Grace Judith Vesga Bravo 
(ICFES) 
Bogotá, mayo 2007
ANÁLISIS DE RESULTADOS 2006 
Grupo de Evaluación de la Educación Superior - ICFES 
Claudia Lucia Sáenz Blanco 
Grupo de Evaluación de la Educación Básica y Media - ICFES 
Flor Patricia Pedraza Daza 
 ICFES 
ISSN: 1909-3993 
Diseño y diagramación: 
Secretaría General, Grupo de Procesos Editoriales - ICFES
ALVARO URIBE VÉLEZ 
Presidente de la República 
Francisco Santos Calderón 
Vicepresidente de la República 
CECILIA MARÍA VÉLEZ WHITE 
Ministra de Educación Nacional 
INSTITUTO COLOMBIANO PARA EL FOMENTO DE 
LA EDUCACIÓN SUPERIOR 
Directora General 
MARGARITA PEÑA BORRERO 
Secretario General 
GENISBERTO LÓPEZ CONDE 
Subdirector de Logística 
FRANCISCO ERNESTO REYES JIMÉNEZ 
Subdirector Académico 
JULIÁN PATRICIO MARIÑO VON HILDEBRAND 
Oficina Asesora de Planeación 
CLAUDIA NATALIA MUJICA CUELLAR 
Oficina Asesora Jurídica 
MARTHA ISABEL DUARTE DE BUCHHEIM 
Oficina de Control Interno 
LUIS ALBERTO CAMELO CRISTANCHO
ANÁLISIS DE RESULTADOS DE 
LAS PRUEBAS DE ESTADO 
1 Introducción 
Presentamos este segundo libro sobre el análisis de los resultados de las pruebas de matemáticas que hacen parte del 
Examen de Estado para Ingreso a la Educación Superior, los datos se refieren a las aplicaciones realizadas en el 2006. 
Este documento está dirigido especialmente a los maestros y maestras, estudiantes, padres de familia y en general a toda 
la comunidad interesada en conocerlos y estudiarlos. 
En primer lugar se resaltan los cambios de la prueba de núcleo común a partir del 2006, a continuación se presenta la es-tructura 
de la misma y los resultados nacionales, así como los resultados en la prueba de profundización. A continuación se 
incluye el análisis de algunas preguntas tanto de núcleo común como de profundización por cada uno de los componentes 
evaluados. Al final se presentan algunas conclusiones y recomendaciones. 
2 Estructura de prueba y resultados nacionales, contrastes entre 
dos calendarios 
Es importante resaltar los cambios que se hicieron a esta prueba a partir de la primera aplicación del 2006 y que fueron 
anunciados y socializados por medio, de talleres en algunas regiones del país, del análisis de resultados del 2005 y más 
general y ampliamente a través del documento “¿Qué evalúan las pruebas 2006?” publicado y disponible en www.icfes. 
gov.co. Los cambios fueron básicamente: 
• Agrupación y redefinición de los componentes evaluados. Estos pasaron de ser conteo, medición, variación y alea-toriedad 
a conformar tres numérico-variacional, geométrico-métrico y aleatorio. El cambio obedece básicamente 
al interés de presentar una organización de los componentes en torno a los pensamientos propuestos tanto en el 
documento de Lineamientos Curriculares como de Estándares de Calidad del área de matemática. Por otra parte, 
se presentan estas agrupaciones por considerarlas pertinentes para los contextos de evaluación, sin embargo, es 
importante aclarar que, en el trabajo en el aula, un mismo contexto puede tener elementos para indagar aspectos 
relacionados con lo numérico, lo métrico y lo aleatorio, por lo tanto no se pretende que esos componentes sean los 
organizadores curriculares. 
• Redefinición de las competencias a evaluar. Es bien conocido que desde el año 2000 se evalúan en todas las áreas 
las competencias interpretativas, argumentativas y propositivas, competencias generales y transversales; sin em-bargo, 
en el área y partiendo de estas competencias, se definieron competencias específicas relacionadas con los 
procesos que en matemática se realizan y tomando como punto de referencia los propuestos en el documento de 
Lineamientos Curriculares, estas son: Competencia en comunicación y representación, competencia en razonamiento 
y argumentación y competencia en modelación, planteamiento y solución de problemas. 
Estos cambios no implican que la prueba se haya transformado radicalmente, el objeto de evaluación sigue siendo la 
competencia matemática; son cambios con los cuales se espera que los maestros y maestras puedan tener mayor claridad 
sobre lo evaluado y su coherencia con lo propuesto por el Ministerio de Educación Nacional. 
2.1 Objeto de evaluación 
El objeto de evaluación de las pruebas es la competencia matemática relacionada con el uso flexible y comprensivo del 
conocimiento matemático escolar en diversidad de contextos, de la vida diaria, de la matemática misma y de otras ciencias. 
Este uso se evidencia, entre otros, en la capacidad del individuo para analizar, razonar, y comunicar ideas efectivamente 
y para formular, resolver e interpretar problemas. 
En la pruebas un aspecto importante a evaluar es el significado de los conceptos matemáticos y la práctica significativa,
ANÁLISIS DE RESULTADOS DE 
LAS PRUEBAS DE ESTADO 
relacionada esta última con la matematización que exige al estudiante simbolizar, formular, cuantificar, validar, esquematizar, 
representar, generalizar, entre otros. Actividades que le permitirán desarrollar descripciones matemáticas, explicaciones 
o construcciones. 
Desde estas perspectivas, en las pruebas se propusieron problemas que indagaban tanto por el conocimiento matemático 
que ha logrado consolidar el estudiante, como por los procesos de pensamiento. Se exploró el uso de la matemática en 
contextos que permitieran mediante procesos de matematización reconocer los conceptos y estructuras construidos en la 
matemática escolar. Se asumió para la construcción de la prueba la propuesta integradora de los Lineamientos Curriculares 
y los Estándares Básicos de Competencia, respecto a los conocimientos básicos, procesos y contextos. 
En cada uno de los contextos seleccionados se propusieron problemas tanto rutinarios como no rutinarios, con distintos 
niveles de complejidad, con la intención de destacar la importancia de enfrentar al estudiante a situaciones diversas que 
exijan comprensión y uso significativo de los conceptos y procedimientos y que den la posibilidad de seleccionar caminos 
o estrategias diversas para su solución. 
2.2 Componentes evaluados y resultados en el núcleo común 
Para la estructuración de las pruebas se organizaron los pensamientos que se proponen en los Lineamientos Curriculares 
y Estándares Básicos de Competencias en tres componentes: el numérico-variacional, el geométrico-métrico y el aleatorio, 
tomando como referente fundamental los estándares relativos a cada pensamiento en los distintos grupos de grados. 
En lo relativo al componente numérico-variacional, se indagó por la compresión y uso de los números y de la numeración, 
más específicamente por la representaciones decimales de los números reales, el reconocimiento de la densidad de los 
números reales, y la comprensión de las operaciones y sus propiedades y su uso en la resolución de problemas. Se inda-gó 
además, por la identificación de variables, la descripción de fenómenos de cambio y la modelación de situaciones de 
cambio a través de funciones. 
En lo pertinente al eje geométrico-métrico se indagó por el reconocimiento de propiedades y relaciones geométricas, la 
identificación gráfica y algebraica de propiedades de las cónicas (particularmente la parábola), la identificación de ca-racterísticas 
de localización en sistemas de representación cartesiana, la resolución de problemas usando propiedades 
geométricas y la comprensión de conceptos de área y volumen. 
En lo relativo al pensamiento aleatorio se indagó por la representación, lectura e interpretación de datos en contexto; el 
análisis de diversas formas de representación de información numérica, el reconocimiento, descripción y análisis de eventos 
aleatorios y el uso de técnicas de conteo. 
Es importante anotar que cada pensamiento desarrolla habilidades específicas en los estudiantes, relacionadas con sus 
sistemas de representación, con las estructuras conceptuales y con las formas propias de argumentación, por lo tanto 
ninguno de ellos puede ser excluido ni del proceso educativo ni del evaluativo. 
En la gráfica 1 se muestran los promedios de la prueba en las cuatro últimas aplicaciones, se puede observar que en ambos 
calendarios hubo un avance en el promedio entre 2005 y 2006, destacándose especialmente calendario B con aproxima-damente 
5 puntos, aunque también es el calendario en el que más aumenta la dispersión como puede observarse en la 
gráfica 2, esto muestra que el incremento del promedio se debe a que un grupo de personas obtiene mejores resultados 
pero se abre más la brecha entre altos y bajos puntajes.
ANÁLISIS DE RESULTADOS DE 
LAS PRUEBAS DE ESTADO 
Gráfica 1. Promedio 2005 y 2006 
45.00 
44.27 
50.77 
45.42 
55 
50 
45 
40 
2005-1 2005-2 2006-1 2006-2 
Gráfica 2. Desviación 2005 y 2006 
8.37 
8.77 
9.64 
7.92 
10 
9 
8 
7 
2005-1 2005-2 2006-1 2006-2 
En la gráfica 3 se muestra el porcentaje de estudiantes acumulado en cada rango de puntaje al nivel nacional en los dos 
calendarios para el año 2006. Se considera Bajo un puntaje menor o igual a 30 puntos, Medio mayor a 30 y hasta 70 y 
Alto mayor a 70.
ANÁLISIS DE RESULTADOS DE 
LAS PRUEBAS DE ESTADO 
Gráfica 3. Porcentaje acumulado de estudiantes en cada rango de puntaje 
100 
80 
60 
40 
20 
0 
Hasta 
30 
Hasta 
35 
Hasta 
40 
Hasta 
45 
Hasta 
50 
Hasta 
55 
Hasta 
60 
Hasta 
65 
Hasta 
70 
Mayor 
a 70 
Calendario B 2.82 6.06 12.64 23.11 50.06 73.41 86.43 92.88 96.01 100 
Calendario A 3.28 8.05 30.68 46.33 73.8 89.17 95.84 98.48 99.52 100 
Para una mejor interpretación de la información, es importante recordar que un estudiante ubicado en categoría baja logra 
abordar situaciones rutinarias que exigen analizar información puntual y establecer estrategias directas que se caracterizan 
por tener una sola relación, operación o algoritmo para su resolución. Un estudiante ubicado en categoría media utiliza 
aspectos básicos de la matemática escolar, en contextos de no rutina que le exige relacionar y organizar información, 
utilizar diferentes formas de representación y hacer traducciones entre ellas. Y un estudiante ubicado en categoría alta 
muestra capacidad de aplicar los elementos básicos de la matemática escolar en contextos diversos y no rutinarios, rela-cionar 
información, reconocer condiciones y hacer inferencias y generalizaciones, esto es, involucra conceptualizaciones 
más formales. 
Se observa en la gráfica 3, que el porcentaje de estudiantes ubicados en bajo corresponde a 2.82 en calendario B y 3.28 
en calendario A, en el nivel medio al 93.19 y 96.24 respectivamente, por lo tanto al nivel alto sólo llega el 3.49% y el 0.48%. 
Más del 50% de la población sólo alcanza puntajes hasta de 50 puntos y más del 90% llega sólo hasta 65 puntos. Esta 
gráfica evidencia que aunque el promedio nacional se ha incrementado, una gran mayoría de la población se ubica en 
puntajes de la categoría media. 
2.3 Competencias evaluadas y resultados en núcleo común 
A continuación se describe cada una de las competencias evaluadas en el 2006 y los resultados obtenidos. 
En relación con la competencia en comunicación y representación, las pruebas exploraron la capacidad de los estu-diantes 
para establecer relaciones entre materiales físicos e ideas matemáticas, expresar conceptos matemáticos utili-zando 
ilustraciones y para traducir del lenguaje natural al lenguaje simbólico; abordaron, además, aspectos tales como la 
descripción cualitativa y cuantitativa de fenómenos de variación presentados en diferentes contextos mediante diversas 
representaciones (reglas verbales, tablas, gráficas, simbólicas). 
En lo referente a la competencia en razonamiento y argumentación se exploró por la capacidad del estudiante para 
dar cuenta del cómo y del porqué de las estrategias o procedimientos puestos en acción para llegar a conclusiones. Se 
indagó también por aspectos tales como la capacidad para generalizar propiedades y relaciones, reconocer patrones y 
expresarlos matemáticamente. 
En cuanto a la modelación, planteamiento y resolución de problemas, las pruebas exploran el diseño y aplicación de 
estrategias diversas para dar solución a problemas planteados en contextos dentro y fuera de la matemática, la verificación 
e interpretación de resultados de acuerdo con las condiciones iniciales del problema y la generalización de soluciones y 
estrategias.
ANÁLISIS DE RESULTADOS DE 
LAS PRUEBAS DE ESTADO 
En las gráficas 5, 6 y 7 se observa que por lo menos el 90% de los estudiantes se ubica en los dos primeros grados, con 
más del 40% en el grado I, es decir solo logran realizar las tareas propias de cada competencia de manera muy inicial y 
en contextos rutinarios. 
Gráfica 4. Porcentaje de estudiantes ubicados en cada grado en la competencia comunicación y representación 
I II III 
Calendario B 46.62 40.13 11.94 
Calendario A 17.34 79.96 1.65 
Gráfica 5. Porcentaje de estudiantes ubicados en cada grado en la competencia en razonamiento y argumentación 
I II III 
Calendario B 42.06 51.13 5.51 
Calendario A 29.60 68.37 0.91 
Aproximadamente el 12% de los estudiantes de calendario B se ubican en el nivel alto en la competencia en comunicación 
y representación, sin embargo, en calendario A aunque no se llega ni al 2% se destaca que en ésta competencia se ubican 
muchos más estudiantes en nivel medio, el 80%, y menos en nivel bajo el 17%, frente a 40% en medio y 47% en bajo para 
calendario B. Tendencias similares se observan en las otras competencias.
ANÁLISIS DE RESULTADOS DE 
LAS PRUEBAS DE ESTADO 
Gráfica 6. Porcentaje de estudiantes ubicados en cada grado en la competencia de modelación, 
planteamiento y resolución de problemas 
I II III 
Calendario B 55.82 38.33 4.54 
Calendario A 37.10 58.69 3.16 
2.4 Resultados en profundización 
En la profundización se plantean situaciones que exigen al estudiante comprensión de los conceptos y estructuras 
matemáticas básicas. Se indaga con mayor énfasis por la manipulación de proposiciones y expresiones que contienen 
símbolos y fórmulas, la generalización de propiedades y relaciones, la interpretación y uso de definiciones y relaciones y 
por el análisis de cadenas de argumentos. Se enfatiza además en el lenguaje simbólico formal y en las diferentes formas 
de representación. 
En las aplicaciones del 2006 esta prueba se centró en dos componentes el geométrico-métrico y el numérico-variacional. 
Respecto al primero, se indagó por el uso de argumentos geométricos en la solución de problemas, la descripción de 
lugares geométricos, aplicación de criterios de semejanza, el reconocimiento de propiedades de las cónicas, la solución 
de triángulos usando teoremas básicos presentados explícitamente en la prueba. Y con respecto al componente numérico-variacional 
se indagó por el análisis de relaciones entre expresiones algebraicas y gráficas de funciones, y por la modelación 
de situaciones de variación. 
Como se ha comentado en documentos anteriores, no se indaga por conocimientos de un primer semestre universitario 
sino, por los conocimientos y procesos que se proponen en los Lineamientos Curriculares y Estándares Básicos de Com-petencias. 
En esta prueba cada estudiante se ubica de acuerdo a su desempeño en un grado: básico, I, II ó III y recibe además un 
puntaje de 0 a 10. En la gráfica 4, se observa que en ambas aplicaciones más del 60 % de los estudiantes quedan ubicados 
en grado I o grado II y que el porcentaje de estudiante ubicados en nivel básico, es decir aquellos que no alcanzan los 
requisitos mínimos para ubicarse en el grado I, es aún muy alto especialmente en calendario A. Es importante destacar 
que el 15% de los estudiantes de calendario B quedaron ubicados en grado III, esto es un buen muy resultado, teniendo 
en cuenta que en general sólo llega al 2%. 
Los estudiantes que se ubican el grado I son capaces de utilizar de manera directa una fórmula, teorema o expresión o 
La profundización es una prueba electiva que escogen los estudiantes entre lenguaje, biología, ciencias sociales o matemática. En el 2006, tan 
sólo el 17% eligió profundizar en matemática en calendario B y el 15% en calendario A.
ANÁLISIS DE RESULTADOS DE 
LAS PRUEBAS DE ESTADO 
propiedad dada. Hacer traducciones de lenguaje natural a lenguaje simbólico. Reconocer un patrón, argumentaciones no 
formales a partir de gráficas o información numérica, las propiedades de figuras geométricas. Hacer inferencias directas a 
partir de información gráfica o numérica. Identificar la simbología propia de la matemática. 
Los que se ubican en grado II son capaces de combinar expresiones o relaciones, usar propiedades geométricas, aritméticas 
y de variación que requieren transformación de expresiones. Generalizar o identificar una regla, fórmula o modelo que no 
requiere el uso de varios conceptos de manera simultánea. Realizar procesos de reversibilidad en contextos rutinarios. Hacer 
traducciones del lenguaje natural y simbólico al gráfico y viceversa. Hacer inferencias que requieren el establecimiento de 
una o varias relaciones o propiedades. 
Gráfica 7. Porcentaje de estudiantes en cada grado 
Grado Básico I II III 
Calendario B 13.5 32.95 37.43 15.01 
Calendario A 34.06 44.29 18.58 1.71 
Y los estudiantes que se ubican en grado III pueden aplicar teoremas que requieren relaciones con otros conceptos. Realizar 
procesos de reversibilidad en contextos no rutinarios y que requieren establecer varias relaciones. Elaborar argumentos, 
seguir reglas de inferencia. Están en capacidad de utilizar formas de argumentación formales, interpretar condicionales 
y uso de cuantificadores. Modelar situaciones aplicando la definición analítica de las curvas. Solucionar problemas que 
requieran el uso simultáneo de relaciones y conceptos de diferentes pensamientos. 
10
11 
ANÁLISIS DE RESULTADOS DE 
LAS PRUEBAS DE ESTADO 
3 Análisis de preguntas 
A continuación se presenta el análisis de algunas de las preguntas que se propusieron a los estudiantes que presentaron 
la prueba en el 2006, hay ejemplos, tanto de núcleo común como de profundización, de ambas aplicaciones. Se espera 
que este tipo de análisis aporte elementos que le permita especialmente a los maestros y maestras y a los estudiantes, 
estudiar con mayor profundidad la prueba. Por cada una de los ejemplos analizados se describe lo que se evaluó, el nivel de 
exigencia, se presentan posibles estrategias de solución y el porcentaje de estudiantes que eligió cada opción de respuesta, 
no se incluyen los datos correspondientes a omisiones o multimarcas. Estos ejemplos se han agrupado por componente, 
se trata de presentar tanto para núcleo común como para profundización una mirada exhaustiva a los aspectos que se 
indagaron al interior de cada uno. 
3.1 Ejemplos componente Numérico-Variacional 
A continuación se presentan 8 ejemplos entre núcleo común y profundización. 
Contexto para los ejemplos 1 y 2 
Números racionales e irracionales 
Si usted no es matemático y no tiene ninguna relación con la matemática, las 
definiciones de número racional y número irracional no le impresionarán demasiado. 
Número racional es aquél que se puede expresar como cociente de dos números 
enteros, mientras que número irracional es aquel que no admite una expresión de 
este tipo. Los números racionales e irracionales, constituyen lo que se conoce como 
números reales y se pueden expresar en forma decimal y ordenar sobre una línea que 
se denomina la recta real. 
Cuando escribimos 2 
o cualquier otro número irracional en forma 
decimal, encontramos que su desarrollo infinito no consiste en un grupo de 
cifras que se repite periódicamente. Por el contrario, los números racionales 
tienen sucesiones de dígitos que se repiten. Los números 5.3, 0.875, 0.3846 
son 
todos números racionales, sus cifras decimales se repiten. Las expresiones 
decimales de 2,  ,e, 
presentan dicha repetición. En el conjunto de todas 
las expresiones decimales (es decir, en el conjunto de todos los números 
reales) es mucho mas raro que haya una pauta y una repetición que la 
ausencia de las mismas. La armonía es siempre mucho más rara que la 
cacofonía.
12 
ANÁLISIS DE RESULTADOS DE 
LAS PRUEBAS DE ESTADO 
Ejemplo 1, núcleo común 
Componente: Numérico – Variacional Porcentaje por opciones de respuesta 
Competencia: Razonamiento A B C D 
Clave: A 19 6 38 36 
Esta pregunta indaga por el dominio que tienen los estudiantes del conjunto de los números reales, sus propiedades y re-laciones, 
específicamente por la distinción entre números racionales y números irracionales. Exige generalizar propiedades 
y relaciones y explicar usando hechos y propiedades. 
Se esperaría que cualquier estudiante al terminar el grado once, incluso desde grado noveno, pueda responder la pregunta 
por los conocimientos previos que tiene de los números reales, sin embargo la lectura ofrece la posibilidad de hacerlo sin 
recurrir a éstos. En la lectura se explica claramente la diferencia entre un número racional y uno irracional, como justamente 
0.5 es un decimal periódico se puede concluir inmediatamente que representa un número racional, así las opciones B y 
5 = se concluye que la opción correcta es A. 
C no pueden ser correctas. Por otra parte como 0.5 es menor que 0.625 
8 
También puede descartarse la opción D porque la expresión decimal de 0.5 es infinita y periódica. 
Tan sólo el 16% de la población seleccionó la opción correcta y un 74% se distribuyó entre las opciones C y D, posiblemente 
recordaron parte de la lectura o del conocimiento de aula, pero no lograron establecer las diferencias. 
Es necesario hacer un llamado de atención sobre este punto, desde el séptimo grado (muy prematuramente) el estudiante 
memoriza una regla sobre la clasificación de los reales, pero no logra a través de los años darle significado. Se requiere 
un trabajo mas estructurado con este concepto, pues es uno de los pilares del pensamiento variacional en los últimos 
grados.
( 2 1) 2 = 
+ − , que no es un irracional, por lo tanto la opción 
13 
ANÁLISIS DE RESULTADOS DE 
LAS PRUEBAS DE ESTADO 
Ejemplo 2, núcleo común 
En la recta numérica que se muestra se han localizado dos números reales 
y . 
La afirmación “entre los puntos P y Q es posible ubicar otro número irracional” 
es 
A. falsa, porque es el siguiente de 2 . 
B. verdadera, porque un irracional que está entre P y Q es 3 . 
C. falsa, porque solo se pueden ubicar racionales entre P y Q. 
D. verdadera, porque un irracional que está entre P y Q es . 
Componente: Numérico – Variacional Porcentaje por opciones de respuesta 
Competencia: Comunicación A B C D 
Clave: B 19 28 26 28 
Este ítem explora de nuevo el concepto de número real pero ahora específicamente el manejo de las operaciones y sus 
propiedades, y la noción de densidad. Respecto a la competencia indaga por la interpretación y relación de diferentes 
representaciones. 
En primer lugar, si el estudiante tiene clara una noción inicial de densidad y ha interpretado la lectura, debe saber que el 
enunciado propuesto es verdadero, pues entre dos reales cualesquiera hay infinitos reales, por lo tanto es posible ubicar 
por lo menos un irracional. De donde se concluye que las opciones A y C no son correctas. La opción D es falsa porque la 
justificación que se propone para argumentar es falsa: 
1 
2 
2 
correcta es B. Otra forma de validar la respuesta es utilizando una aproximación de los números irracionales así: , y . 
Los porcentajes para cada una de las opciones de respuesta estuvieron, muy próximos –un poco mas bajo para la A, 
19%– , esto podría sugerir más una selección al azar que un análisis de la pregunta. De nuevo los estudiantes recitan una 
frase relativa a la densidad de los reales en la recta, pero posiblemente por la complejidad de la idea o la no ilustración de 
la misma en la práctica escolar, no han logrado interpretarla. Es necesario incluir en el aula no sólo la definición clásica 
de número real, sino el desarrollo de ejercicios variados y de diferentes niveles de dificultad que permitan una verdadera 
aproximación de este concepto por parte de los estudiantes.
14 
ANÁLISIS DE RESULTADOS DE 
LAS PRUEBAS DE ESTADO 
Ejemplo 3, núcleo común 
Deforestación 
En la última década se ha observado que debido a la deforestación, la extensión 
de un bosque se ha venido reduciendo aproximadamente en un 10% anual. Ac-tualmente 
el bosque tiene una extensión de 200 Km2. 
La expresión que representa la extensión E del bosque en función del tiempo t es 
A. E = 200 (0,9)t 
B. E = 200 (0,1)t 
C. E = 200 – 0,2t 
D. E = 200 – 0,8t 
Componente: Numérico – Variacional Porcentaje por opciones de respuesta 
Competencia: Solución de problemas A B C D 
Clave: A 19 31 36 13 
En esta pregunta se indaga por la construcción de una expresión matemática que representa la determinación del modelo 
y la generalización de un proceso de deforestación, dicha expresión permite identificar la relación de variación existente 
entre dos variables. 
Para dar solución al problema el estudiante puede observar la regularidad, aplicando de manera consecutiva las condiciones 
dadas, construyendo así una secuencia relativa a la extensión del bosque a medida que transcurren los años: 
Extensión actual del bosque E = 200 
Extensión del bosque dentro de 1 año E = 200 x 0.9 
Extensión del bosque dentro de 2 años E = 200 x 0.9 x 0.9 
Extensión del bosque dentro de 3 años E = 200 x 0.9 x 0.9 x 0.9 
Extensión del bosque dentro de t años E = 200 x 0.9 x 0.9 x ... x 0.9 
Determinando, de esta manera, que la respuesta correcta es A. Los estudiantes que seleccionan la opción B no entienden 
completamente las condiciones del problema y hacen una traducción literal de éste, sin embargo entiende que el modelo es 
exponencial. Aproximadamente el 50% de los estudiantes escogen las opciones C o D, lo cual muestra que sólo conocen 
el modelo lineal, una forma de obtener la opción C es tomar como valor de decrecimiento el 10% de 200 y para la opción 
D tomar el 10% de 200 y trabajar con el complemento.
15 
ANÁLISIS DE RESULTADOS DE 
LAS PRUEBAS DE ESTADO 
Contexto para los ejemplos 4 y 5 
Ruta Bogotá Cúcuta 
El siguiente gráfico muestra una ruta para ir desde Bogotá a Cúcuta vía terrestre. 
En el gráfico aparece información sobre: distancias, temperaturas y alturas. 
Ejemplo 4, núcleo común 
A partir de la información de la gráfica se puede afirmar que la ciudad que está a una altura 
mayor de 2.000m, tiene una temperatura promedio menor que 17°C y está a más de 500Km 
de Bogotá es 
A. Tunja 
B. Cúcuta. 
C. Pamplona. 
D. Bucaramanga. 
Componente: Numérico – Variacional Porcentaje por opciones de respuesta 
Competencia: Comunicación A B C D 
Clave: C 16 6 73 5 
La pregunta explora por la capacidad del estudiante para interpretar y usar diferentes tipos de representación, esta asociada 
con estándares referidos al planteamiento y solución de situaciones utilizando argumentos que justifiquen relaciones entre 
información numérica. 
Para dar solución a la pregunta, es necesario identificar, en el gráfico presentado, la información referida a diferentes 
ciudades en la ruta Bogotá-Cúcuta respecto a su distancia de Bogotá (en kilómetros), su altura sobre el nivel del mar (en 
metros) y su temperatura (en grados centígrados). Dadas las condiciones del problema, el estudiante debe interpretar 
esta información, y establecer que la altura sobre el nivel del mar y la temperatura se relacionan de manera inversa, para 
después buscar la ciudad de la gráfica (más específicamente de las presentadas en las opciones) la ciudad, que cumple 
simultáneamente las condiciones de estar a una altura mayor de 2.000m tener una temperatura menor de 17°C y estar a 
más de 500Km de Bogotá y concluir que la respuesta correcta es Pamplona.
ANÁLISIS DE RESULTADOS DE 
LAS PRUEBAS DE ESTADO 
El 73% de los estudiantes seleccionaron C, la opción correcta, demostrando capacidad para leer interpretar y relacionar 
información numérica que caracteriza diferentes lugares (ciudades, poblaciones y peajes) en un mapa vial; como puede 
observarse este ítem está asociado con la aplicación de conceptos matemáticos en la vida cotidiana. 
Los estudiantes que seleccionaron la opción A, Tunja, el 16%, equivocaron su lectura respecto a la distancia a Bogotá, las 
otras dos condiciones se cumplen; aquellos que seleccionan la opción B, el 6%, equivocan la interpretación de dos de las 
condiciones pues Cúcuta no está a más de 2.000 metros de altura, ni tiene una temperatura menor de 17°. Finalmente, la 
selección de la opción D, Bucaramanga, realizada por el 5% de los estudiantes, indica que no entienden las condiciones 
del enunciado o no establecen relaciones de orden entre los números que indican altura, distancia y temperatura, pues 
Bucaramanga no cumple ninguna de las condiciones solicitadas. 
Ejemplo 5, núcleo común 
Si un automóvil se desplazara a una velocidad constante durante todo el 
trayecto (Bogotá – Cúcuta), el tramo en el cual la rapidez de variación de la 
altura es mayor es 
A. Tunja – Arcabuco. 
B. San Gil – Aratoca. 
C. Pamplona – El diamante. 
D. Pescadero – Bucaramanga. 
Componente: Numérico – Variacional Porcentaje por opciones de respuesta 
Competencia: Razonamiento A B C D 
Clave: C 14 21 53 12 
La pregunta indaga por aspectos relacionados con el concepto de variación, específicamente por la razón de cambio. Permite 
explorar la capacidad del estudiante para justificar estrategias y dar cuenta del cómo y del porqué de los procedimientos 
que se siguen para llegar a conclusiones. 
El estudiante debe identificar el tramo recorrido por un automóvil en el cual la altura varía con mayor rapidez, suponiendo 
que el automóvil se desplaza con velocidad constante. Para encontrar la respuesta correcta es necesario hallar los cocientes 
entre diferencias de altura y de distancias recorridas (de la ciudad o población donde se inicia cada uno de los tramos y 
la ciudad o población donde terminan), o lo que es lo mismo hallar la pendiente de los segmentos que representan cada 
tramo para determinar cual es la mayor. Una simple observación de los cocientes que representan la rapidez de variación 
de la altura en cada uno de los tramos permite determinar que el trayecto desde Pamplona hasta el Diamante presenta la 
mayor rapidez de variación en la altura: 
16 
Tunja – Arcabuco: 
San Gil – Aratoca: 
Pamplona El Diamante:
17 
ANÁLISIS DE RESULTADOS DE 
LAS PRUEBAS DE ESTADO 
Pescadero – Bucaramanga: 
Es posible llegar a esta misma conclusión observando en el mapa el segmento de recta con mayor inclinación (el más 
pendiente o empinado). 
El 53% de los estudiantes seleccionó C la opción correcta, demostrando capacidad para hacer la lectura de la información 
numérica que aparece en la ilustración y compresión del concepto de razón de cambio en este contexto. El 21% de los 
estudiantes seleccionó la opción B San Gil – Aratoca, que es el tramo que le sigue a Pamplona – El Diamante en cuanto 
a rapidez de variación, es posible que su afirmación esté apoyada en la observación directa del grado de inclinación del 
segmento que representa el recorrido. La selección de las opciones A o D hecha por el 14% y el 12% de los estudiantes 
pudo haber sido hecha al azar o por interpretación incorrecta de las condiciones. 
Contexto para los ejemplos 6 y 7 
Caída de un objeto 
Si un objeto con masa m se deja caer, y se tiene en cuenta la resistencia del aire, 
una función que describe la velocidad v del objeto después de t segundos es 
donde g es la aceleración de la gravedad y c y e son constantes positivas. 
Ejemplo 6, profundización 
A medida que transcurre el tiempo, la velocidad del objeto 
A. permanece constante. 
B. disminuye y se aproxima a cero. 
C. disminuye y se aproxima a 
mg . 
c 
D. aumenta y se aproxima a 
mg . 
c 
Componente: Numérico - Variacional Porcentaje por opciones de respuesta 
Competencia: Comunicación A B C D 
Clave: D 23 15 8 53 
Este ítem explora aspectos referidos a la modelación de situaciones de variación con funciones exponenciales y al uso de 
técnicas de aproximación en procesos infinitos numéricos. Está relacionado además con la capacidad del estudiante para 
interpretar, y usar diferentes tipos de lenguaje, describir relaciones y manipular expresiones simbólicas.
18 
ANÁLISIS DE RESULTADOS DE 
LAS PRUEBAS DE ESTADO 
Para resolver la pregunta el estudiante debía tomar la expresión y analizar tendencias: como , cuando 
t crece (tiende a infinito) tiende a 0 y tiende a 1, de donde crec e y se aproxima 
a 
c 
mg , es decir la clave es D. 
Un porcentaje alto, el 53%, respondió bien la pregunta, pero si la selección partiera de una comprensión de la variación 
esta respuesta debería ser coherente con la selección de la gráfica, pero esto no ocurrió (ver análisis ejemplo 9), puede 
mg 
ser que al observar el tipo de función concluyan que aumenta, pero no reconozcan específicamente la asíntota c 
. Es 
importante sin embargo destacar el análisis logrado para una función poco familiar. El 23% no logra interpretar el modelo y 
asume que v es constante posiblemente porque en el enunciado se hizo alusión a algunas constantes. El 15% que selec-ciona 
B y el 8% que seleccionan C pueden haber realizado un análisis parcial de la tendencia por ejemplo la exponencial 
decrece o se aproxima a cero y seleccionar por ello las opciones mencionadas. 
Ejemplo 7, profundización 
La gráfica que relaciona la velocidad v del objeto con el tiempo t es 
Componente: Numérico - Variacional Porcentaje por opciones de respuesta 
Competencia: Comunicación A B C D 
Clave: A 26 15 31 26 
Esta pregunta está relacionada con análisis en representaciones gráficas cartesianas de los comportamientos de cambio 
de funciones y con la identificación de relaciones entre propiedades de las ecuaciones algebraicas y propiedades de las 
gráficas. Está referida a la capacidad del estudiante para representar y modelar usando lenguaje gráfico y algebraico. 
De nuevo para responder la pregunta el estudiante debía estudiar el modelo, reconocer que las variables v y c no son 
directamente proporcionales y desde luego que v cambia cuando t cambia. Esto permite decidir que las opciones B y C 
son incorrectas. A continuación bastaba analizar si la función crecía o decrecía apoyados en reconocer la imagen de cero. 
Como la imagen de cero es cero y a medida que t crece decrece ( se aproxima a cero), , se aproxima 
a 1, por tanto v tiende a 
mg . La opción correcta es A. 
c 
El porcentaje que seleccionó la respuesta correcta A coincide con los que seleccionaron D, es probable que los últimos 
identifiquen el comportamiento de un modelo exponencial pero no la condición inicial, ni el cambio. La selección de D puede 
provenir, además, de confundir el movimiento de la caída del objeto con la trayectoria. De todas maneras contrasta con 
la pregunta anterior donde el acierto fue mayor y se esperaría que los que respondieron bien a la pregunta ilustrada en el 
ejemplo 8 respecto a la tendencia, estuvieran en capacidad de identificar la correspondiente gráfica, sin embargo las dos 
preguntas son independientes.
19 
ANÁLISIS DE RESULTADOS DE 
LAS PRUEBAS DE ESTADO 
El 31% –un porcentaje mayor que el asociado a la clave– considera que el modelo es lineal, posiblemente es la única gráfica 
con la que está familiarizados y no logran interpretar las condiciones. El porcentaje restante considera que la función es 
constante, como se comentó antes, por la alusión a las constantes en el enunciado o porque no reconocen las variables 
en la expresión. 
Contexto para los ejemplos 8 y 9 
Construir espejos 
Para construir espejos en vidrio, una empresa diseña espejos tipo A de forma de 
hexágono regular, obtenidas del mayor tamaño posible a partir de láminas circula-res 
de vidrio de 1 metro de radio. Cortando por la mitad las piezas tipo A, se obtie-nen 
piezas tipo B. 
Ejemplo 8, núcleo común 
Las piezas tipo A y B se venden a $17.000 y $10.000 respectivamente. La 
empresa vende 5 piezas y recibe un pago por un valor total de $63.900. Si se 
sabe que sobre esta compra se hizo un descuento del 10% sobre el precio total 
de las piezas, ¿cuántas piezas se vendieron de cada tipo? 
A. 2 del tipo A y 3 del tipo B. 
B. 3 del tipo A y 2 del tipo B 
C. 4 del tipo A y 1 del tipo B. 
D. 1 del tipo A y 4 del tipo B. 
Componente: Numérico – Variacional Porcentaje por opciones de respuesta 
Competencia:Solución de problemas. A B C D 
Clave: B 24 43 21 11 
El ítem indaga por el uso de los números reales en sus diferentes representaciones y en diversos contextos, exige dar 
significado a la variable e identificar diferentes métodos para solucionar sistemas de ecuaciones. Además requiere reali-zar 
una traducción del lenguaje natural al lenguaje simbólico formal, y desarrollar y aplicar diferentes estrategias para la 
solución de un problema.
20 
ANÁLISIS DE RESULTADOS DE 
LAS PRUEBAS DE ESTADO 
El estudiante podía resolver la pregunta realizando el siguiente razonamiento, si p es el precio sin el descuento se tiene 
que p  0.1p  63.900, de donde p  71.000 . Si x es el número de piezas tipo A y y es el número de piezas tipo B, 
es posible determinar x e y resolviendo el sistema de ecuaciones lineales 
Sustituyendo y en la primera ecuación y reduciendo se obtiene 7.000x  21.000, de donde x = 3, y = 2 . 
Otra forma de encontrar la solución es ensayar con los valores dados en las opciones. Al ensayar con lo propuesto en la 
clave se tiene que 3 del tipo A tienen un costo de $51.000 y 2 del tipo B de $20.000, así 51.000+20.000=71.000, luego 
se saca el 10% al costo total que es $7.100 y restando este valor a los 71.000 se obtiene 63.900. Estrategia que resulta 
interesante. 
El 43% de los estudiantes respondieron la pregunta correctamente y el 24% posiblemente resolvieron correctamente pero 
se confundieron en la presentación de la respuesta (identificación de las variables), los estudiantes que seleccionaron las 
otras opciones posiblemente se limitan a identificar dos números que sumados den 5 (número de piezas), pero no inter-pretan 
las otras condiciones. 
Ejemplos componente Geométrico–Métrico 
A continuación se presenta el análisis de 5 preguntas de núcleo común y 11 de profundización; en el 2006 el énfasis en la 
profundización se colocó en este componente, sin embargo todas las situaciones propuestas pueden y deben ser abordadas 
por todos los estudiantes. 
Ejemplo 9, núcleo común 
El área que cubren 4 piezas tipo B dispuestas como lo indican la figura, es 
A. 
3 metros cuadrados 
4 
B. 3 3 metros cuadrados 
3 3 metros cuadrados 
C. 2 
D. 6 3 metros cuadrados 
El contexto para este ejemplo se encuentra antes del ejemplo 8, en la sección de ejemplos del componente numérico-variacional.
= , donde P es el perímetro y a el apotema, que corresponde a la altura del triángu-lo 
a = 1− 1 = , de donde . Como la figura 
21 
ANÁLISIS DE RESULTADOS DE 
LAS PRUEBAS DE ESTADO 
Componente: Geométrico – Métrico Porcentaje por opciones de respuesta 
Competencia:Solución de problemas. A B C D 
Clave: B 39 15 29 17 
La pregunta indaga por el cálculo de áreas a través de la composición y descomposición de figuras exige formular un 
problema a partir de una representación gráfica. 
Para resolverlo el estudiante puede determinar el área del hexágono inscrito en un círculo de radio 1m (Pieza tipo A). Una 
forma es utilizar la fórmula 
A P a × 
2 
equilátero de lado uno. Se tiene que P = 6 y 
3 
2 
4 
 
 
2 2 3 3 =   
A = . 
muestra la descomposición de dos hexágonos, entonces la respuesta sería 3 3 
2 
 
  
 
Pero, la pregunta no requería recordar la fórmula para calcular el área de un hexágono, simplemente saber determinar el 
área de un triángulo equilátero de lado 1, bastaría determinar la altura 
3 , de donde el área sería . Con 
2 
12 3  . 
doce triángulos se recubren las cuatro piezas tipo B, por tanto el área es 3 3 
4 
También se puede resolver el problema hallando el área del paralelogramo, para lo cual se debe asociar la apotema 
del hexágono con la altura del paralelogramo, y de esta manera se obtiene que la base es 6 y la altura 
3 , luego 
2 
3 3 
A  b h  6 3  
2 
El porcentaje más alto de estudiantes seleccionó la opción A, es muy posible que no hayan realizado ningún tipo de cálculo 
y simplemente porque observaron un cuatro en el denominador, lo asociaron a 4 piezas, pero desconocen los procedi-mientos 
antes mencionados, no hay siquiera un reconocimiento visual de las relaciones. La selección de C, superior al 
25% pudo originarse en un cálculo incorrecto o en la utilización de una fórmula no pertinente. La opción D es escogida por 
un porcentaje superior al asociado a la clave, aparece allí un seis y éste podría asociar a los seis lados del hexágono. En 
estos casos, las opciones de respuesta provenían de cálculos incorrectos pero posiblemente el estudiante no realiza éstos 
sino que se orienta por alguna información parcial (número de lados por ejemplo). 
Se esperaba que la pregunta resultara de nivel de dificultad medio, sin embargo es posible que por la aparición de números 
irracionales en la operatoria o por el desconocimiento de procedimientos para determinar el área de figuras planas, tan 
sólo la haya contestado el 15%.
22 
ANÁLISIS DE RESULTADOS DE 
LAS PRUEBAS DE ESTADO 
Contexto para los ejemplos 10 y 11 
Diseño de un parque 
En un lote de forma rectangular cuyos lados miden 80 y 60 metros, se va a cons-truir 
un parque. La figura muestra el plano del parque. Los puntos B, D, F y G son 
los puntos medios de los lados del rectángulo ACEH, K es un punto de AE tal que 
CK es perpendicular a AE . 
Ejemplo 10, núcleo común 
La longitud de AE es 
A. 100 metros 
B. 140 metros. 
C. 2 7 metros 
D. 2 35 metros. 
Componente: Geométrico – Métrico Porcentaje por opciones de respuesta 
Competencia: Solución de problemas A B C D 
Clave: A 31 54 8 7 
El ítem indaga por la relación existente entre las medidas de los lados de un triángulo rectángulo, por la aplicación del 
teorema de Pitágoras.
23 
ANÁLISIS DE RESULTADOS DE 
LAS PRUEBAS DE ESTADO 
Para dar solución a este problema se requiere que el estudiante identifique la longitud del camino AE como la hipotenusa 
de un triángulo rectángulo cuyos catetos miden 80 y 60 metros y posteriormente aplique el Teorema de Pitágoras: 
Por lo tanto la opción correcta es A. La selección de la opción B, como respuesta correcta, hecha por el 54% de los estu-diantes 
puede indicar que no identifican el teorema de Pitágoras como la estrategia a utilizar para dar solución al problema 
o que hacen una mala aplicación de este puesto que la opción resulta de sumar las medidas de los catetos. 
La selección de las opciones C, o, D puede indicar una equivocada aplicación del teorema, confundiendo los cuadrados 
de las medidas de los catetos con el doble: en el caso de la C ,o, hallando la raíz de la suma de las 
medidas y no de los cuadrados de estas en el caso de la D. 
Nótese que a pesar ser una aplicación directa del teorema, sin dificultad en la parte operatoria, solamente el 31% de los 
estudiantes llegó la respuesta correcta. Esto puede indicar que en las aulas el trabajo con el teorema de Pitágoras se limita 
a su aplicación mecánica y rutinaria, y que no se está abordando su aplicación en otro tipo de problemas como el que se 
presenta, en los cuales no se induce a la utilización del teorema ya que no se explicita la existencia de un triángulo rectángulo 
y no se utilizan los términos asociados a este (hipotenusa y catetos). Es importante trabajar otro tipo de aplicaciones. 
Ejemplo 11, núcleo común 
El área de la zona cubierta de pasto es 
A. 1.800 metros cuadrados. 
B. 2.400 metros cuadrados. 
C. 3.600 metros cuadrados. 
D. 4.800 metros cuadrados 
. 
Componente: Geométrico – Métrico Porcentaje por opciones de respuesta 
Competencia: Razonamiento A B C D 
Clave: A 43 26 16 15 
El ítem indaga por la comprensión y aplicación de conceptos básicos de proporcionalidad y por el uso de argumentos 
geométricos para formular problemas en contextos matemáticos, se explora además la competencia del estudiante para 
explicar usando hechos y propiedades. 
Para determinar el área de la zona cubierta de pasto, basta hallar la diferencia entre las áreas de los triángulos AEH y 
GFH, esto es 
También es posible determinar la solución, hallando el área de trapecio y usando semejanza. 
El 43% de los estudiantes respondió correctamente la pregunta y un 26% se limitó a determinar el área de uno de los 
triángulos y seleccionó B. Porcentajes próximos seleccionaron C o D, sumando las áreas en lugar de restar o determinando 
el área de un rectángulo.
ANÁLISIS DE RESULTADOS DE 
LAS PRUEBAS DE ESTADO 
La pregunta sería pertinente para los primeros grados de la básica secundaria e incluso para la primaria y sin embargo 
presenta dificultad para más de la mitad de la población que presentó la prueba. 
Ejemplo 12, núcleo común 
Triángulos 
Los polígonos se clasifican de acuerdo a sus propiedades y relaciones: medidas 
de los lados, medidas de los ángulos, relaciones entre sus lados, etc. 
Los triángulos se clasifican de acuerdo a las medidas de sus lados en isósceles, 
equiláteros y escálenos. Un triángulo con dos lados congruentes se llama isósce-les, 
con tres lados congruentes se llama equilátero. Un triángulo escaleno es aquel 
en el cual todos sus lados tienen diferente medida. 
De acuerdo a la clasificación de los triángulos, NO es correcto afirmar que 
A. si un triángulo es equilátero es isósceles. 
B. si un triángulo no es escaleno es equilátero. 
C. existen triángulos rectángulos que son isósceles. 
D. existen triángulos isósceles que no son equiláteros. 
Componente: Geométrico - Métrico Porcentaje por opciones de respuesta 
Competencia: Razonamiento A B C D 
Clave: B 41 21 22 15 
La pregunta indaga por la clasificación de triángulos de acuerdo a sus propiedades, exige interpretar criterios presentados 
explícitamente en el contexto de la prueba, está relacionada con la generalización de propiedades y relaciones e indaga 
por la interpretación de definiciones y el análisis de proposiciones cuantificadas. 
Para contestar correctamente, el estudiante debía retomar las definiciones presentadas inicialmente e interpretar la negación. 
Es correcto afirmar que si es equilátero, (tiene los tres lados congruentes) tiene dos lados congruentes, es decir, es isósceles 
por lo tanto la opción A no es falsa. También es correcto afirmar que existen triángulos rectángulos que son isósceles, por 
ejemplo todo triángulo con ángulos de 45°, 45° y 90° así la opción C también es correcta. Es correcto además que existen 
triángulos isósceles que no son equiláteros, por ejemplo un triángulo rectángulos con lados 1, 1 y 2 , así la opción D 
es correcta. Pero es incorrecto afirmar que si un triángulo no es escaleno es equilátero, puede ser isósceles o equilátero, 
aunque la pregunta es fácil, excepto posiblemente por la negación, tan sólo la respondió correctamente el 21% 
Es posible que un porcentaje alto de los estudiantes haya ignorado la negación y se limite a seleccionar opciones diferentes 
a la clave por considerarlas correctas, lo que atrajo especialmente hacia la A, que muy seguramente es más familiar en el 
aula. Sería pertinente que particularmente en geometría se analicen enunciados diversos donde se expresen condiciones, 
negaciones y se trabaje con proposiciones compuestas; es importante destacar que las herramientas de la lógica formal 
se construyen en cada uno de los pensamientos no esquemáticamente en apartes dispersos del currículo. 
24
25 
ANÁLISIS DE RESULTADOS DE 
LAS PRUEBAS DE ESTADO 
Contexto para ejemplo 13. 
La parábola 
Una parábola es el lugar geométrico de todos los puntos del plano que equidistan 
de un punto fijo llamado foco y una recta fija llamada directriz. En el siguiente 
cuadro se muestran ecuaciones y gráficas que corresponden a parábolas con el 
vértice, el foco y la directriz ubicados en diferentes puntos del plano 
Ejemplo 13, núcleo común 
La gráfica de la parábola con foco en el punto (6,4) y directriz que pasa por el punto 
(0,-2) se presenta en,
26 
ANÁLISIS DE RESULTADOS DE 
LAS PRUEBAS DE ESTADO 
Componente: Geométrico – Métrico Porcentaje por opciones de respuesta 
Competencia: Comunicación A B C D 
Clave: A 36 28 15 21 
Esta pregunta hace referencia a la identificación de las características de localización de objetos geométricos en sistemas 
de representación cartesiana y al uso de las propiedades de las cónicas, particularmente de la parábola. Exige interpretar, 
representar y usar diferentes tipos de lenguaje para describir relaciones. 
Para resolver la pregunta el estudiante debía simplemente interpretar la información que se presenta en los recuadros 
iniciales, pasando por la definición de la parábola y la identificación de sus elementos, si interpreta esta caracterización 
puede descartar fácilmente las opciones diferentes a la clave pues, o bien los puntos de la curva no equidistan del foco 
y la directriz, o la recta señalada no es la directriz cuando la parábola es abierta hacía la izquierda o la derecha, en este 
sentido puede afirmarse que el ítem acude esencialmente a la visualización. 
El 36% de los estudiantes respondieron correctamente la pregunta, se esperaba que este porcentaje fuera más alto pues 
no requiere mayor elaboración. Porcentajes cercanos optaron por B o D posiblemente sin análisis de condiciones, sim-plemente 
por ubicación de la recta supuesta como directriz, si hubieran elegido un punto de estas parábolas las hubieran 
descartado si comprenden la definición. 
Obsérvese que en este contexto se plantean preguntas que indagan por la noción puramente matemática de la parábola. 
Es importante recuperar en el aula de clase las definiciones matemáticas y trabajar estos conceptos no sólo en aplicaciones 
cotidianas o de otras áreas sino también en aplicaciones al interior de la matemática misma. 
Contexto para los ejemplos 14 y 15 
En un triángulo ABC como el que muestra la figura, a, b y c corresponden a las 
longitudes de sus lados. 
Los siguientes teoremas relacionan lados y ángulos de un triángulo ABC cualquiera. 
Teorema del Seno Teorema del coseno 
SenC 
c 
SenA  SenB 
 
b 
a 
2 2 2 
a = b + c − 
2 
bcCosA 
2 2 2 
b = a + c − 
2 
acCosB 
c a b 2 
abCosC 
2 2 2 
= + −
27 
ANÁLISIS DE RESULTADOS DE 
LAS PRUEBAS DE ESTADO 
Ejemplo 14, profundización 
En el triángulo que muestra la figura los valores de b y Sen α son 
A. b = 7 y Sen B. b = 7 y Sen 
C. b = 5 y Sen 
D. b = 5 y Sen 
Componente: Geométrico – Métrico Porcentaje por opciones de respuesta 
Competencia: Razonamiento A B C D 
Clave: A 35 19 34 12 
Se indaga en esta pregunta por la aplicación de los teoremas del seno y del coseno, está relacionado con el uso de argu-mentos 
geométricos, propiedades y relaciones para resolver y formular problemas. Se indaga además por la justificación 
de estrategias y procedimientos en el tratamiento de situaciones problema. 
El estudiante debe identificar la información presentada en la figura y aplicar en primer lugar el teorema del coseno 
Luego debe aplicar el teorema del seno 
de donde .
senA = senB 
= 
senA = senA 
= 2 
28 
ANÁLISIS DE RESULTADOS DE 
LAS PRUEBAS DE ESTADO 
El 35% resolvió correctamente la pregunta pero un porcentaje similar seleccionó la opción C, se limitan a copiar un dato de 
la información inicial o aplican incorrectamente las relaciones al triángulo dado. El 19% selecciona B, es posible que haya 
aplicado correctamente el teorema del coseno, pero intercambia información al aplicar el segundo teorema. La selección de 
D puede provenir de una selección al azar o de composición arbitraria de los números dados sin observar condiciones. 
Es posible que este ítem resulte complejo debido a que el estudiante debe aplicar sucesivamente ambos teoremas, sin 
embargo este tipo de problemas se supone son ampliamente trabajados durante el grado décimo, además el estudiante 
no requiere saber los teoremas de memoria pues son presentados dentro del contexto, por lo cual se esperaría que un 
porcentaje más alto de la población lo resolviera correctamente, más aún al estar en la sesión de profundización y no de 
núcleo común, lo cual resultaría pertinente. Invitamos a los maestros y maestras a resolver este tipo de problemas con 
todos sus estudiantes e identificar las razones por las cuales no logran resolverlo apropiadamente y así poder reforzar 
estos aspectos. 
Ejemplo 15, profundización 
Si en un triángulo ABC se cumple que SenA = SenB = 2SenC, entonces el perí-metro 
del triángulo es 
A. 3b 
B. 5c 
C. 2a + 2c 
D. a + b + 2c 
Componente: Geométrico – Métrico Porcentaje por opciones de respuesta 
Competencia: Razonamiento A B C D 
Clave: B 7 10 14 69 
En esta pregunta se indaga por la aplicación del teorema del seno para el cálculo del perímetro de un triángulo, está rela-cionada 
con el reconocimiento y contraste de propiedades y relaciones geométricas. 
Para resolver el problema se retoman relaciones del teorema del seno, 
senC 
c 
b 
a 
usando las condiciones dadas en la pregunta se tiene que 
senA 
c 
b 
a 
de donde, b = a y 2c = a , y por lo tanto a = b = 2c 
El perímetro del triángulo será entonces P = a + b + c = 2c + 2c + c = 5c 
La mayoría de los estudiantes que presentaron la prueba no lograron interpretar la pregunta y seleccionaron la opción D
29 
ANÁLISIS DE RESULTADOS DE 
LAS PRUEBAS DE ESTADO 
(69%) posiblemente lo asumieron por la razón entre los senos, 1:1:2 se limitaron a copiar los números o es posible que ni 
siquiera recuerden la expresión para el perímetro y opten por una que se parece aunque no tenga sentido. Porcentajes 
bajos seleccionaron las otras opciones, por azar o aplicación de transformaciones incorrectas. 
Nuevamente llama la atención que siendo una pregunta de profundización y que sólo requiere una aplicación del teorema 
del seno para encontrar el perímetro, haya sido contestada por un porcentaje tan bajo de la población, esto posiblemente 
también muestra que cuando se aborda este tipo de aplicación se hace de manera rutinaria, generalmente se pide al 
estudiante determinar todos los valores del triángulo: medida de los lados y de los ángulos, de manera numérica y no a 
través del uso de relaciones entre los mismos. Es necesario proponer más y variados ejercicios de aplicación de este tipo 
de conceptos. 
Contexto para los ejemplos 16 y 17 
Mosaicos 
En la ilustración se observan algunos mosaicos formados por polígonos regulares. 
En cada mosaico los lados de los polígonos que se utilizan deben tener la misma 
medida. 
Ejemplo 16, profundización 
En el mosaico que se muestra, 
la medida del ángulo α es 
A. 60° 
B. 90° 
C. 120° 
D. 150° 
Componente: Geométrico – Métrico Porcentaje por opciones de respuesta 
Competencia: Comunicación A B C D 
Clave: C 24 15 55 6
ANÁLISIS DE RESULTADOS DE 
LAS PRUEBAS DE ESTADO 
El ítem explora por la capacidad de estudiante para interpretar información presentada en diferentes tipos de represen-tación 
y para construir argumentaciones que den solución a situaciones diversas, está relacionado con la aplicación de 
propiedades de figuras planas. 
El estudiante debe averiguar la medida de uno de los ángulos que forman el vértice de un mosaico al cual concurren dos 
triángulos equiláteros y dos hexágonos regulares; para dar solución a la pregunta, debe observar que la suma de los ángulos 
que concurren a un vértice es 360º, y plantear las relaciones que le permitan hallar la medida del ángulo solicitado: 
• los ángulos interiores de un triángulo equilátero son congruentes, como la suma de los ángulos interiores de todo 
30 
triángulo es 180º, entonces cada ángulo mide 60º 
• de donde 
La pregunta también puede resolverse si se averigua directamente la medida del ángulo interior de un hexágono regular, 
dividiéndolo en triángulos: 
Como se forman 4 triángulos entonces la suma de los ángulos interiores de un hexágono debe ser 180°× 4 = 720° , por 
ser hexágono regular los seis ángulos tienen la misma medida de donde y por lo tanto 
El 55% de los estudiantes seleccionó C, la opción correcta, aquellos que seleccionaron A, el 24%, o B, el 15%, no enten-dieron 
las condiciones del problema ni observaron la ilustración, pues evidentemente el ángulo α es obtuso por lo tanto no 
puede medir ni 60º ni 90º. 
Teniendo en cuenta que este no es un contexto común y seguramente resultó novedoso para muchos de los estudiantes, 
el porcentaje que contestó correctamente es bastante alto, lo cual muestra un dominio importante en propiedades de las 
figuras geométricas en el grupo que escogió profundizar en matemática. 
Ejemplo 17, profundización 
NO es posible construir un mosaico si a un mismo vértice concurren 
A. 2 octágonos y 1 cuadrado. 
B. 2 octágonos y 2 cuadrados. 
C. 1 hexágono regular y 4 triángulos equiláteros. 
D. 2 hexágonos regulares y 2 triángulos equiláteros. 
Componente: Geométrico – Métrico Porcentaje por opciones de respuesta 
Competencia: Razonamiento A B C D 
Clave: B 21 34 27 16 
La pregunta explora la capacidad del estudiante para dar cuenta del cómo y del porqué de los caminos que se siguen para
180º (8 − 
2) como son dos se tiene que los ángulos de los 
180º (6 − 
2) más 240° por los cuatro ángulos de los triángulos 
31 
ANÁLISIS DE RESULTADOS DE 
LAS PRUEBAS DE ESTADO 
llegar a conclusiones; está asociada la aplicación de estrategias que involucran aspectos de medición. 
Para dar solución a la pregunta el estudiante puede aplicar la fórmula, que se presenta en el recuadro, en cada una de las 
opciones hasta encontrar aquella en la cual la suma de la medida de los ángulos que concurren al vértice NO sea 360º, 
concluyendo de esta manera que la opción correcta es B pues: 
135 
• Opción A. Cada ángulo del octágono mide = ° 
8 
octágonos suman 270° , más 90° del cuadrado. Por lo tanto si se puede construir. 
• Opción B. Basta con sumar 90° a la cuenta anterior así se deduce inmediatamente que no puede construirse. 
120 
• Opción C. Cada ángulo del hexágono mide = ° 
6 
equiláteros. Por lo tanto si se puede construir. 
• Opción D. Los ángulos de los dos hexágonos mines 240° más 120° de los dos ángulos de los triángulos equiláteros. 
El 34% de los estudiantes seleccionó la opción correcta, la selección de otras opciones indica que los estudiantes no en-tendieron 
las condiciones que deben cumplir los ángulos de polígonos regulares que concurren al vértice de un mosaico, 
o se equivocaron en los reemplazos requeridos para la correcta aplicación de la fórmula. 
Contexto para los ejemplos 18 y 19 
La siguiente figura muestra una maqueta para una construcción. 
La maqueta está formada por un paralelepípedo y una pirámide de base cuadrada 
de 20cm de lado. Las caras laterales de la pirámide son triángulos equiláteros.
32 
ANÁLISIS DE RESULTADOS DE 
LAS PRUEBAS DE ESTADO 
Ejemplo 18, profundización 
La altura total de la maqueta 
A. está entre 10cm y 20cm. 
B. está entre 20cm y 25cm. 
C. está entre 25cm y 35cm. 
D. está entre 35cm y 40 cm. 
Componente: Geométrico – Métrico Porcentaje por opciones de respuesta 
Competencia: Solución de problemas A B C D 
Clave: C 14 19 35 31 
La pregunta indaga por la altura de la pirámide, para lo cual se requiere la aplicación de propiedades de la pirámide, de 
triángulos equiláteros y aplicación del teorema de Pitágoras y pasar de una representación tridimensional a una bidimen-sional; 
aborda aspectos de referidos a la solución de problemas en los cuales se usan propiedades geométricas y explora 
por la capacidad de los estudiantes en el planteo y aplicación de estrategias para dar solución a diferentes situaciones. 
Para resolver la pregunta el estudiante debe averiguar la altura total de la maqueta sumando a la altura del paralelepípedo 
(15cm) la altura de la pirámide cuadrangular que está construida sobre él. 
Para hallar la altura de esta pirámide es necesario hallar, inicialmente, la altura de los triángulos equiláteros que forman 
las caras laterales 
Aplicando el teorema de Pitágoras se tiene que 
Conociendo esta altura, es posible aplicar nuevamente el teorema de Pitágoras para hallar la altura de la pirámide, consi-derando 
que por ser una pirámide regular el centro de la base coincide con el pie de la altura:
33 
ANÁLISIS DE RESULTADOS DE 
LAS PRUEBAS DE ESTADO 
Como el estudiante no dispone de calculadora para realizar esta operación, debe encontrar un rango del valor de esta 
altura considerando que la raíz de 200 es mayor que 14 (142 =196) y menor que 15 (152=225) Por lo tanto la altura total de 
la pirámide está entre 29cm y 30cm. 
El 35% de los estudiantes seleccionó C, la opción correcta, Los estudiantes que seleccionaron la opción D, el 31% , posible-mente 
no entendieron las condiciones del problema o tuvieron problemas operatorios. El 14% y el 19% que seleccionaron las 
opciones A y B respectivamente, posiblemente resolvieron el problema de manera parcial, seleccionando como respuesta 
las alturas del triángulo o de la pirámide sin tener en cuenta la del paralelepípedo. 
Ejemplo 19, profundización 
La base del paralelepípedo se va a recubrir con láminas de forma rectangular 
de lados 4cm y 1 cm. El mínimo número de láminas que se necesitan es 
A. 16 
B. 25 
C. 75 
D. 100 
Componente: Geométrico – Métrico Porcentaje por opciones de respuesta 
Competencia: Solución de problemas A B C D 
Clave: D 24 26 25 24 
Este ítem explora la capacidad del estudiante para diseñar e implementar estrategias que den solución a un problema, está 
relacionado con el cálculo de área por recubrimiento. 
Para averiguar el número mínimo de láminas de forma rectangular de lados 4cm y 1cm que se necesitan para recubrir un 
cuadrado de lado 20, el estudiante, puede determinar que es posible acomodar sobre un lado del cuadrado 5 láminas (por 
el lado de 4cm) y sobre el otro lado 20 láminas (por el lado de 1cm), por lo tanto se necesitan 100 láminas. 
También es posible dar solución a la pregunta considerando que puesto que el número de láminas que se pueden acomodar 
sobre los lados del cuadrado es exacto, ya que 20 es múltiplo de 4 y de 1 puede dividirse el área del cuadrado (400 cm2) 
entre el área de la lámina rectangular (4 cm2) obteniendo de esta manera que son necesarias 100 láminas. 
A pesar de ser una pregunta, sin dificultad específica aparente, solamente el 24% de los estudiantes seleccionó D, los 
estudiantes que seleccionaron A o B, el 24% y 26% consideraron que se podían acomodar, sobre los lados del cuadrado, 
4 y 5 láminas respectivamente.
34 
ANÁLISIS DE RESULTADOS DE 
LAS PRUEBAS DE ESTADO 
Contexto para los ejemplos 20, 21 y 22 
Triángulos semejantes 
Dos triángulos ABC y A’B’C’ son semejantes si se cumple uno cualquiera de 
los siguientes criterios: 
1. Los ángulos correspondientes son congruente, es decir 
. ∠A ≅∠A’, ∠B ≅∠B’, ∠C ≅∠C’ 
2. Dos pares de lados correspondientes son proporcionales y los ángulos comprendidos son 
congruentes, es decir 
y ∠A ≅∠A’ y ∠B ≅B’ y ∠C ≅∠C’ 
3. Lados correspondientes son proporcionales, es decir
35 
ANÁLISIS DE RESULTADOS DE 
LAS PRUEBAS DE ESTADO 
Ejemplo 20, profundización 
En cada figura se muestra un par de triángulos 
De los pares de triángulos, son semejantes, los mostrados en las figuras 
A. 1 y 2 
B. 2 y 4 
C. 1 y 3 
D. 3 y 4 
Componente: Geométrico – Métrico Porcentaje por opciones de respuesta 
Competencia: Comunicación A B C D 
Clave: B 21 28 16 34 
Está relacionado este ítem con la resolución y formulación de problemas que involucran relaciones de semejanza usando 
representaciones visuales. Para resolver el problema el estudiante debía determinar simplemente las razones entre los 
lados correspondientes de los triángulos presentados, si están en la misma razón serán semejantes. 
• En la figura 1, , por tanto los triángulos no son semejantes. 
• En la figura 2, los triángulos tienen dos pares de lados proporcionales (razón 
3 ) y un par de ángulos opuestos por 
5 
el vértice congruentes por tanto los triángulos son semejantes. 
• En la figura 3, la razón entre los lados no es constante ( ) los triángulos no son semejantes. 
• En la figura 4, , pares de lados correspondientes son proporcionales, los triángulos son semejan-tes. 
Así la respuesta correcta es B, son semejantes los pares de triángulos mostrados en las figuras 2 y 4. 
El 28% respondió correctamente la pregunta un porcentaje muy bajo para el nivel de dificultad y teniendo en cuenta que 
es una pregunta de profundización, posiblemente la noción de semejanza está en el nivel de “tener la misma forma” y 
se asumen criterios puramente visuales, desconocen posiblemente los criterios que permiten caracterizar la relación. 
Nótese que no se requería para la solución la memorización de las condiciones pues justamente era lo que presentaba el
ANÁLISIS DE RESULTADOS DE 
LAS PRUEBAS DE ESTADO 
contexto, por eso se insiste en que el porcentaje de estudiantes que contestan este tipo de preguntas debería ser mayor. 
La opción D, atrajo más que la clave, posiblemente por una consideración de razones incompletas de aparentes lados 
correspondientes. ( ). 
Es recomendable que los maestros y maestras retomen este tipo de ejercicios y los realicen con todos los estudiantes, 
la noción de semejanza aunque es un tema que generalmente aparece en el currículo de grado noveno, es ampliamente 
utilizada para la solución de diferentes tipos de problemas en los grados posteriores, es uno de los conceptos fundamentales 
que cualquier estudiantes al terminar sus estudios debe poder utilizar. 
Ejemplo 21, profundización 
Los triángulos ABC y A’B’C’ son semejantes 
Las medidas de los lados x y y son respectivamente 
Componente: Geométrico – Métrico Porcentaje por opciones de respuesta 
Competencia: Razonamiento A B C D 
Clave: A 25 31 30 13 
El ítem está relacionado con la aplicación de los criterios de semejanza, requiere establecer relaciones y utilizar transfor-maciones. 
Como los triángulos ABC y A´B´C´ son semejantes para resolver el problema el estudiante debía plantear la razón entre los 
lados correspondientes es decir , de donde y . Por 
lo tanto la respuesta correcta es A. 
Solamente un 25% de los estudiantes respondió correctamente la pregunta, sin embargo un 31% respondieron B, posible-mente 
confundieron las variables, se podría afirmar en consecuencia que el 56% realizó procedimientos correctos. 
36
37 
ANÁLISIS DE RESULTADOS DE 
LAS PRUEBAS DE ESTADO 
Los estudiantes que seleccionaron C y D muy seguramente no identificaron correctamente los lados proporcionales, sin 
observar nominación de los vértices, plantearon la razón comparando lados y B´C´, por ejemplo, y determinaron el 
valor , seleccionando las opciones que lo contenían. Faltaría en ese caso un análisis cuidadoso de las condiciones 
planteadas inicialmente. 
Ejemplo 22, profundización 
Sea ABC un triángulo, D un punto de y E un punto de , como se 
muestra en la figura 
A. ∠AED ≅ ∠ABC . 
B. AB ≅ BC y AD = DE. 
C. el triángulo ADE es semejante al triángulo ABC. 
D. el ángulo ACB es congruente con el ángulo BAC. 
Componente: Geométrico – Métrico Porcentaje por opciones de respuesta 
Competencia: Razonamiento. A B C D 
Clave: C 15 23 47 14 
Este ítem indaga por el reconocimiento y contrastación de propiedades y relaciones geométricas en la demostración de 
teoremas básicos. Pretende analizar si el estudiante reconoce que es una prueba en matemáticas y como se diferencia de 
otros tipos de razonamiento, distinguiendo y evaluando cadenas de argumentos. 
Los ángulos ∠AED y ∠ACB son congruentes por ser correspondientes entre las paralelas y . El ∠CAB 
es común a los triángulos ADE y ABC , por tanto estos dos triángulos tienen sus tres ángulos congruentes, por criterio 
Angulo-Angulo-Angulo se puede concluir que los dos triángulos son semejantes. Por ser semejantes tienen lados corres-pondientes 
proporcionales de donde . 
La opción correcta es entonces C. Las otras opciones no se pueden deducir con las condiciones dadas en el problema y 
así se dieran, no implican la proporcionalidad de los lados. 
El 47% de los estudiantes seleccionó la opción correcta, es un porcentaje importante, en algunos casos es posible que 
hayan realizado el razonamiento correcto, pero también existe la posibilidad de que por un criterio visual, hayan reconocido
ANÁLISIS DE RESULTADOS DE 
LAS PRUEBAS DE ESTADO 
los dos triángulos como semejantes. Solamente en una prueba abierta que es pertinente para el aula, el profesor podría 
observar si el estudiante realiza un razonamiento adecuado. El resto del grupo se distribuyó con porcentajes relativamente 
similares, más alto para la opción B que, de nuevo, observando la figura podría asumir incorrectamente que los lados 
tienen la misma medida. 
Ejemplo 23, profundización 
Trayectoria de un barco 
La hipérbola es el lugar geométrico de los puntos del plano cuya diferencia de 
distancias a dos puntos fijos llamados focos es constante. 
En el plano cartesiano se muestra la trayectoria hiperbólica que describe un 
barco, dos radares A y B, ubicados en los focos de la trayectoria y un puesto de 
control 0. 
Desde cualquier punto P de la trayectoria el barco envía señales a los radares 
ubicados en los puntos A y B a distancias d1 y d2 respectivamente. Las señales se 
desplazan a una velocidad constante. 
Teniendo en cuenta que la trayectoria que describe el barco es hiperbólica se 
debe cumplir que 
A. d1 + d2 es constante 
B. d1 – d2 es constante 
C. d2 es constante 
D. d1 es constante 
Componente: Geométrico – Métrico Porcentaje por opciones de respuesta 
Competencia: Razonamiento. A B C D 
Clave: B 44 27 15 13 
Está relacionado el ítem con la identificación de características de localización de las cónicas en un sistema de representación 
cartesiana. Indaga por la generalización de propiedades y relaciones y la capacidad de expresarlas matemáticamente. 
Para responder la pregunta el estudiante simplemente debe traducir la definición “La hipérbola es el lugar geométrico de 
los puntos del plano cuya diferencia de distancias a dos puntos fijos llamados focos es constante”. Si la distancia 
38
39 
ANÁLISIS DE RESULTADOS DE 
LAS PRUEBAS DE ESTADO 
del punto P a un foco es d1 y la distancia de P al otro foco es d2 , 1 2 d − d es una constante. Por tanto la clave es B. 
El porcentaje más alto de estudiantes seleccionó la opción A (43%), es posible que recuerden la definición analítica de la 
elipse, confunden las dos curvas y seleccionan por ello esta opción, o también es posible que no interpreten la palabra 
“diferencia” y opten por la expresión más familiar, la suma. El 27% logran realizar una traducción correcta y porcentajes 
similares posiblemente sin interpretar condiciones seleccionan las otras opciones. 
Al igual que el contexto de la parábola este debe resultar poco familiar para los estudiantes, por eso este sólo se incluyó en 
profundización, sin embargo se supone que esta cónica es estudiada junto con las demás en grado décimo y por lo tanto 
un conocimiento inicial como la definición debería resultar familiar y fácil para los estudiantes. Nótese que no se indagaba 
por la memorización de hipérbola, bastaba con leer el contexto y traducir matemáticamente la descripción. 
Contexto para los ejemplos 24 y 25 
Otra mirada al teorema de Pitágoras 
Los triángulos sombreados que aparecen en cada figura son rectángulos. So-bre 
los lados de cada triángulo se han construido figuras planas semejantes.
40 
ANÁLISIS DE RESULTADOS DE 
LAS PRUEBAS DE ESTADO 
Ejemplo 24, profundización 
Si el área del cuadrado 1 es la mitad del área del cuadrado 2, entonces el área 
del cuadrado 3 es 
A. la mitad del área del cuadrado 2. 
B. el doble del área del cuadrado 2. 
C. el triple del área del cuadrado 1. 
D. la tercera parte del área del cuadrado 1 
Componente: Geométrico – Métrico Porcentaje por opciones de respuesta 
Competencia: Razonamiento. A B C D 
Clave: C 9 26 51 12 
Como se indica en el nombre del contexto, en los ejemplos 25 y 26 se indaga por la aplicación del teorema de Pitágoras, 
mediante el establecimiento de relaciones geométricas. 
Para dar solución a la pregunta el estudiante debe, inicialmente, identificar los catetos y la hipotenusa del triángulo rectán-gulo, 
que no está en posición canónica, sobre cuyos lados se han construido cuadrados, para aplicar la relación existente 
entre las áreas de estos cuadrados teniendo en cuenta las condiciones del enunciado. 
Teniendo en cuenta la ilustración y aplicando el teorema de Pitágoras se tiene que: Área del cuadrado construido sobre la 
hipotenusa = suma de las áreas de los cuadrados construidos sobre los catetos, por lo tanto 
Área del cuadrado 3 = Área del cuadrado 1 + Área del cuadrado2 
Como el área del cuadrado 1 es la mitad del área del cuadrado 2, o lo que es lo mismo, el área del cuadrado 2 es el doble 
del área del cuadrado 1, 
entonces 
Área del cuadrado 3 = Área del cuadrado 1 + 2 x Área del cuadrado1 
= 3 x área del cuadrado 1 
Determinando, de esta manera, que la respuesta correcta es C. 
El 51% de los estudiantes seleccionó la opción correcta, lo cual permite afirmar que identifican los catetos y la hipotenusa 
de triángulos rectángulos dibujados en diferentes posiciones y que aplican correctamente el teorema de Pitágoras. Los 
estudiantes que seleccionaron la opción B, el 26%, posiblemente utilizaron, de manera equivocada, la información del 
enunciado acerca de la relación entre las áreas de los cuadrados 1 y 2 y finalmente, los estudiantes que seleccionaron las 
opciones A , el 9%, y D, el 12% no entendieron las condiciones de problema ni utilizaron estrategias que les permitieran 
establecer relaciones entre las áreas de los cuadrados. 
La ilustración de esta pregunta es utilizada para demostrar el Teorema de Pitágoras, es decir, debe ser familiar para los 
estudiantes y aunque el porcentaje que contestó correctamente es alto, nuevamente se llama la atención pues es una 
pregunta que corresponde a la profundización, por lo tanto debería existir una mayor apropiación de las relaciones que 
existen entre las áreas de los cuadrados que se construyen sobre los lados y la hipotenusa de un triángulo rectángulo, es 
decir, está es la aplicación más conocida del Teorema de Pitágoras.
41 
ANÁLISIS DE RESULTADOS DE 
LAS PRUEBAS DE ESTADO 
Ejemplo 25, profundización 
Los radios de las circunferencias en las cuales se pueden inscribir los hexágo-nos 
1 y 2 son 6cm y 8cm respectivamente. El perímetro y el área del triángulo 
rectángulo son 
A. 12cm y 6cm2. 
B. 12cm y 24cm2. 
C. 24cm y 48cm2. 
D. 24cm y 24cm2. 
Componente: Geométrico – Métrico Porcentaje por opciones de respuesta 
Competencia: Razonamiento. A B C D 
Clave: D 19 36 30 13 
La pregunta indaga por el perímetro y el área de un triángulo rectángulo, cuando se conocen las medidas de los radios de 
las circunferencias en las cuales se pueden inscribir los hexágonos regulares que aparecen construidos sobre los lados del 
triángulo rectángulo. Está relacionada con estándares referidos a la interpretación y aplicación de propiedades y relaciones 
geométricas de figuras planas; explora por la capacidad del estudiante para diseñar procedimientos y estrategias aplicando 
propiedades y relaciones geométricas que permitan solucionar situaciones planteadas en diferentes contextos. 
Para dar respuesta al problema, el estudiante debe deducir, inicialmente, que como el radio de una circunferencia en la 
cual se ha inscrito un hexágono es congruente con el lado de éste, los catetos del triángulo rectángulo miden 6cm y 8cm. 
Una vez conocidas las medidas de los catetos puede aplicar el teorema de Pitágoras para encontrar que la medida la 
hipotenusa, es de 10 cm completando de esta manera la información necesaria para hallar el perímetro y el área: 
Perímetro: ; Área: 
El 13% de los estudiantes utilizó correctamente la relación de congruencia entre el radio de la circunferencia y el lado del 
hexágono inscrito, utilizó el teorema de Pitágoras y aplicó el concepto de altura de un triángulo para hallar la opción correcta 
D. Los estudiantes que escogieron C identificaron las condiciones del problema pero no calcularon de manera correcta el 
área del triángulo. Los estudiantes que seleccionaron las otras opciones posiblemente se limitaron a realizar operaciones 
con la información numérica que aparece en el enunciado.
ANÁLISIS DE RESULTADOS DE 
LAS PRUEBAS DE ESTADO 
3.3 Ejemplos componente Aleatorio 
Se presenta a continuación el análisis de 6 ejemplos correspondientes a núcleo común. 
Contexto para los ejemplos 26 y 27 
Salarios 
Los salarios mensuales de los 25 empleados de una empresa están distribuidos 
de la siguiente manera 
• 21 empleados ganan 1 salario mínimo mensual 
• 2 empleados ganan 10 salarios mínimos mensuales 
• 1 empleado gana 14 salarios mínimos mensuales 
• 1 empleado gana 25 salarios mínimos mensuales 
42 
Ejemplo 26, núcleo común 
El valor que mejor representa el conjunto de datos sobre el salario mensual del 
grupo de empleados es 
A. 1 salario mínimo mensual. 
B. 10 salarios mínimos mensuales. 
C. 14 salarios mínimos mensuales. 
D. 25 salarios mínimos mensuales. 
Componente: Aleatorio Porcentaje por opciones de respuesta 
Competencia: Razonamiento A B C D 
Clave: A 50 9 6 34 
La pregunta indaga por la comprensión del significado de las medidas de tendencia central, en este caso específico por la 
interpretación de la moda como el dato más representativo de una información numérica. 
Para dar solución a la pregunta el estudiante debe deducir, de la información que se presenta en el recuadro, que de un 
total de 25 empleados 21 ganan 1 salario mínimo, es decir que 1 salario mínimo mensual es el valor que con más frecuencia 
se presenta en el conjunto de datos. Luego el dato que mejor describe el conjunto es el de 1 salario mínimo mensual, por 
lo tanto la opción correcta es A, que fue seleccionada por la mitad de la población. 
Quienes escogen las otras opciones pueden hacerlo por una selección al azar o por análisis erróneos. Esta pregunta es 
bastante sencilla y aunque la contesto la mitad de la población, llama la atención el porcentaje tan alto que escogió como 
opción correcta la D, es probable que simplemente sea porque la empresa tiene 25 empleados. Es importante que los 
maestras y maestras al enseñar los conceptos de promedio, media, media y moda, además de ejercitar el cálculo de estos, 
se trabaje en su significado y cuando un concepto u otro representa mejor un conjunto de datos, siempre se cree que es 
el promedio, en este caso esa opción no se colocó para no inducir a los estudiantes a ese error.
43 
ANÁLISIS DE RESULTADOS DE 
LAS PRUEBAS DE ESTADO 
Ejemplo 27, núcleo común 
En el departamento de producción de la empresa trabajan 4 mujeres y 6 hombres. 
La edad promedio de las mujeres es 30 años y la de los hombres es 40. La edad 
promedio de los trabajadores del departamento de producción es 
A. 30 años. 
B. 35 años. 
C. 36 años. 
D. 40 años. 
Componente: Aleatorio Porcentaje por opciones de respuesta 
Competencia: Solución de problemas A B C D 
Clave: C 8 43 27 21 
La pregunta está orientada a explorar la capacidad del estudiante para aplicar diferentes estrategias en la solución de 
problemas relacionados con conceptos básicos asociados con medidas de tendencia central, en este caso se indaga por 
la correcta utilización del concepto de promedio ponderado. 
Para dar solución al problema, es necesario hallar la suma de las edades, en años, de todos los empleados teniendo en 
cuenta el número de mujeres y de hombres que trabajan en el departamento de producción: en el caso de las mujeres 
puesto que son 4 y su edad promedio es 30, el número total de años se obtiene efectuando 4 x 30 = 120 años y de manera 
análoga, para los hombres multiplicando 6 x 40 = 240 años. La suma de este número de años es entonces 360; como en 
total son 10 empleados, para encontrar la edad promedio de todos los trabajadores es necesario efectuar 
encontrando así que la opción correcta es C. 
El 27% de los estudiantes resolvió correctamente la pregunta, mientras que un 43% seleccionó la opción B que resulta de 
hallar el promedio de las edades promedio, de mujeres y hombres, sin tener en cuenta el número de trabajadores del depar-tamento. 
Los estudiantes que seleccionan las opciones A, o, D, posiblemente, no entienden las condiciones del problema 
y escogen opciones en las cuales aparecen las edades promedio de hombres o mujeres del enunciado de la pregunta. 
Es necesario que en las aulas se sigan trabajando actividades de profundización que permitan al estudiante diferenciar y 
aplicar correctamente los conceptos de promedio y promedio ponderado.
4 
ANÁLISIS DE RESULTADOS DE 
LAS PRUEBAS DE ESTADO 
Contexto para el ejemplo 28 
Diseño de placas 
El Ministerio de Transporte es la Institución en Colombia encargada de diseñar y 
establecer las características de la placa única nacional para los vehículos auto-motores. 
A partir de 1990 las placas tienen tres letras y tres dígitos, debajo llevan 
el nombre del municipio donde se encuentra matriculado el vehículo. Para la fabri-cación 
de las placas se utilizan 27 letras y 10 dígitos. La empresa que fabrica las 
placas ha comprobado que de una producción de 100 placas fabricadas aproxima-damente 
5 placas tienen algún defecto. 
Ejemplo 28, núcleo común 
La primera letra de la placa de los carros particulares matriculados en Bogotá 
es A o B. El número total de placas que pueden fabricarse para identificar carros 
particulares matriculados en Bogotá es 
A. 272 x 103 
B. 273 x 102 
C. 2 x 272 x 102 
D. 2 x 272 x 103 
Componente: Aleatorio Porcentaje por opciones de respuesta 
Competencia: Solución de problemas A B C D 
Clave: D 25 25 28 22 
El ítem indaga por la formulación y resolución de problemas usando conceptos básicos de conteo (combinaciones, permu-taciones, 
arreglos condicionados) y explora el desarrollo y aplicación de diversas estrategias para resolver problemas. 
Para encontrar la respuesta correcta el estudiante debía realizar un argumento como el siguiente: como la primera letra es A 
o B entonces solo se tienen dos posibilidades de selección, para las otras letras se tienen 272 opciones y para seleccionar 
los números 103 maneras. En total el número de placas posibles en Bogotá es 2 x 272 x 103. 
Los porcentajes de elección de las opciones de respuesta se distribuyeron de manera similar –alrededor del 25%–, con 
una leve preferencia por la opción C. Esto parece indicar que la selección o bien fue realizada al azar o los estudiantes 
intentaron realizar un conteo pero no se precisaron las posibilidades para cada posición o no se entendieron bien. 
Se esperaría que en una pregunta que requiere un conteo sencillo el porcentaje de estudiantes que responda correctamen-te 
fuera mucho mayor, aunque en los últimos años se han ido incorporando estos temas en los currículos, es necesario 
profundizar más en estos y de ser posible incluirlo desde el grado octavo o noveno de manera que se llegue a ejercicios 
que requieran técnicas de conteo más sofisticadas en los cursos superiores. Este tipo de temas, además, tienen muchas 
aplicaciones en situaciones de la vida cotidiana que pueden ser aprovechadas para la correcta apropiación.
45 
ANÁLISIS DE RESULTADOS DE 
LAS PRUEBAS DE ESTADO 
Contexto para los ejemplos 29, 30 y 31 
Mundiales de fútbol 
Cada cuatro años la FIFA (Federation International Football Association) rea-liza 
el Campeonato Mundial de Fútbol en el que participan 32 selecciones. 
Las 32 selecciones se distribuyen mediante un sorteo, en 8 grupos de 4 equi-pos 
cada uno. Para evitar el enfrentamiento entre favoritos, en la primera 
ronda eliminatoria los 8 equipos considerados como los mejores se asignan 
como cabeza de grupo. 
En la primera ronda cada equipo juega una vez contra cada uno de los demás 
equipos de su grupo y se eliminan dos equipos de cada grupo. Entre los 16 
clasificados se eliminan 8 y en la siguiente ronda se eliminan 4. Entre los 4 
que quedan se determina el campeón, subcampeón, tercero y cuarto. 
Ejemplo 29, núcleo común 
Si en la primera ronda de un campeonato, en uno de los grupos el promedio 
de goles anotado por partido fue de 2,5 goles, el total de goles anotados en 
ese grupo fue 
A. 10 
B. 15 
C. 20 
D. 24 
Componente: Aleatorio Porcentaje por opciones de respuesta 
Competencia: Razonamiento A B C D 
Clave: B 42 27 21 10 
La pregunta está relacionada con estándares asociados a la utilización comprensiva de conceptos básicos de conteo y de 
medidas de tendencia central, en este caso con el concepto de promedio. Teniendo como contexto una lectura referente 
a los mundiales de fútbol, el ítem indaga por el número de goles anotados, durante la primera ronda de un campeonato 
mundial en uno de los grupos, cuando se sabe el número de equipos de cada grupo y el promedio de goles por partido. 
Para dar solución al problema, el estudiante debe, inicialmente, determinar que en el grupo se jugaron 6 partidos puesto 
que en cada grupo hay 4 equipos y cada uno de ellos juega 1 vez contra los otros, este conteo puede realizarse mediante 
la construcción del listado de partidos
46 
ANÁLISIS DE RESULTADOS DE 
LAS PRUEBAS DE ESTADO 
Por lo tanto en total se juegan 6 partidos. 
O de manera más formal, encontrando el número de combinaciones sin repetición que se pueden hacer con cuatro equipos 
para formar grupos de 2 
Posteriormente, teniendo en cuenta que el promedio de goles por partido fue de 2,5 y que se jugaron 6 partidos debe 
efectuar 2,5 x 6 para concluir que el número de goles anotados en este grupo fue de 15. 
Tal vez, por ser una pregunta cuya respuesta no es inmediata pues la estrategia de solución exige tanto la aplicación de 
diferentes conceptos asociados con el pensamiento aleatorio, como la aplicación de un proceso de reversibilidad (el rela-cionado 
con el concepto de promedio) solamente un 27% de los estudiantes seleccionó B la opción correcta. 
Los estudiantes que seleccionaron otras opciones, posiblemente no entendieron las condiciones del problema, pues como 
puede observarse éste no presenta dificultad de tipo operatorio. Un alto porcentaje de estudiantes, el 42%, seleccionó la 
opción A, que resulta de multiplicar el promedio de goles por el número de equipos del grupo. El 21% seleccionó la opción 
C que resulta de adicionar algunos números que aparecen en la lectura inicial, Los estudiantes que seleccionaron la opción 
D, el 10%, multiplicaron el número de partidos –6- por el número de equipos de cada grupo –4-.
47 
ANÁLISIS DE RESULTADOS DE 
LAS PRUEBAS DE ESTADO 
Ejemplo 30, núcleo común 
La probabilidad de que en un mundial el equipo campeón, no sea uno de los 
equipos cabeza de grupo es 
A. 
7 
8 
1 
B. 
8 
C. 
3 
4 
D. 
1 
4 
Componente: Aleatorio Porcentaje por opciones de respuesta 
Competencia: Solución de problemas A B C D 
Clave: C 14 32 22 31 
Este ítem está relacionado con el cálculo de la probabilidad de eventos y el uso de conceptos básicos de probabilidad. 
Exige el planteamiento y solución de un problema a partir de situaciones o contextos externos a la matemática. 
Para resolverlo el estudiante debe tomar el total de los equipos 32 y restar los 8 cabeza de grupo, quedan 24 (casos favo-rables 
al evento: no ser cabeza de grupo). La probabilidad es entonces 
por lo tanto la opción correcta es la C que fue elegida por el 22% de los estudiantes. 
El 32% de los estudiantes seleccionó la opción B posiblemente pensando que como hay 8 equipos cabeza de grupo 
cada uno tiene de probabilidad de ser campeón, no interpretan la condición del problema o simplemente los atrae el 
8, algo similar pudo pasar con el 14% que selecciono A. Respecto a la opción D hubo un porcentaje alto que pensó en la 
probabilidad del complemento , pero, en este caso no interpretan la negación, y calculan la probabilidad de que 
un equipo cabeza de grupo sea campeón. 
Nótese que se indaga por una probabilidad muy fácil de calcular, que debería resultar familiar para los estudiantes, sin 
embargo es probable que la dificultad aumente por el tipo de contexto empleado, que si bien es conocido, seguramente no 
se usa para indagar por conceptos de probabilidad. Es importante insistir que la noción de probabilidad debe introducirse 
en los primeros grados, de tal manera que un estudiante al terminar su secundaria pueda además resolver problemas de 
probabilidad condicional y a través del uso de herramientas más sofisticadas.
48 
ANÁLISIS DE RESULTADOS DE 
LAS PRUEBAS DE ESTADO 
Ejemplo 31, núcleo común 
En la siguiente gráfica se muestra el número total de partidos jugados y el 
número total de goles anotados en algunos de los campeonatos mundiales de 
fútbol. 
. 
El promedio de goles por partido fue mayor en el campeonato mundial de 
A. España 82. 
B. México 86. 
C. Italia 90. 
D. Francia 98. 
Componente: Aleatorio Porcentaje por opciones de respuesta 
Competencia: Comunicación A B C D 
Clave: A 19 2 6 72 
Utilizando como contexto información numérica que aparece en un diagrama de barras, referente al número de goles y al 
número de partidos jugados en diferentes campeonatos mundiales de fútbol, la pregunta indaga por el campeonato con 
mayor promedio de goles. Esta pregunta explora la capacidad del estudiante para interpretar diferentes tipos de represen-tación 
de información numérica, específicamente un diagrama de barras; está asociada con estándares relacionados con 
conceptos de medidas de tendencia central. 
Para dar solución a la pregunta, el estudiante debe aplicar el concepto de promedio hallando el mayor de los cocientes 
obtenidos al dividir el número de goles entre el número de partidos, en cada uno de los campeonatos mundiales, aunque 
basta con hacerlo con los datos de los mundiales que aparecen en las opciones de respuesta. Una correcta interpretación 
de la información le permite descartar los campeonatos de Italia 90 y México 86 en los cuales se jugó el mismo número 
de partidos que en España 82 pero con menos goles en total. De manera que para seleccionar la respuesta correcta debe 
determinar que el cociente 146/52 correspondiente al promedio de goles en España 82 es mayor que el cociente 171/64 
correspondiente a Francia 90. 
Únicamente el 19% de los estudiantes interpretó la información y aplicó correctamente el concepto de promedio seleccio-nando 
España 82, la opción correcta. La mayoría de los estudiantes, el 72%, seleccionó la opción D Francia 98, es posible
49 
ANÁLISIS DE RESULTADOS DE 
LAS PRUEBAS DE ESTADO 
que hayan asociado el mayor número de goles al mayor promedio, es decir, consideraron que por ser la barra del número 
de goles la más alta, este campeonato era el de mayor promedio de goles. Esto muestra que si bien se está trabajando 
en las aulas la lectura de gráficas de barras, se hace de manera muy inicial y tradicional, es decir traducir el valor que 
muestra la barra, pero es necesario introducir más problemas que requieran usar la información para llegar a otro tipo de 
conclusiones y que involucren otros conceptos. 
4. Conclusiones y recomendaciones 
 El promedio de la prueba en ambos calendarios ha seguido aumentando en los últimos dos años, destacándose 
especialmente calendario B con aproximadamente 5 puntos, aunque también es el calendario en el que más aumenta 
la dispersión. Esto muestra que el incremento del promedio se debe a que un grupo de personas obtiene mejores 
resultados pero se abre más la brecha entre altos y bajos puntajes. Como se dijo en el documento de análisis de 
resultados del año 2005, las practicas de aula deben encaminarse a promover desarrollos significativos en toda la 
población. 
 Aproximadamente el 95% de los estudiantes siguen ubicándose en el nivel medio y más del 50% de la población 
sólo alcanza puntajes hasta de 50 puntos y más del 90% llega sólo hasta 65 puntos. Esto ratifica que aunque el 
promedio nacional se ha incrementado, una gran mayoría de la población se ubica en puntajes de la categoría 
media. Es necesario seguir trabajando en las aulas en el fortalecimiento de conceptos básicos de la matemática 
escolar, conceptos que deben ser claros para cualquier estudiante que termina su secundaria y que debe darle la 
posibilidad de usarlos de manera significativa en diferentes situaciones, como las propuestas en el examen. Sola-mente 
mediante el fortalecimiento de los aspectos básicos será posible que los estudiantes se enfrenten con éxito 
a situaciones no rutinarias que les exigen poner en práctica diferentes conceptos, relacionarlos y utilizar diversas 
estrategias de solución para llegar a la respuesta correcta. 
 En profundización, por ser una prueba opcional se esperaría que los resultados fueran más altos. Sin embargo, en 
el 2006 el 13.5% de los estudiantes de calendario B y el 34% de los de calendario A quedaron ubicados en grado 
básico, es decir no alcanzaron los requisitos mínimos del primer grado. En contraste tan sólo el 15% de los estu-diantes 
de calendario B y el 2% de los de calendario A se ubicaron en el grado superior, grado III. 
 Respecto al componente numérico-variacional, se aprecia un manejo adecuado de información numérica presen-tada 
en gráficas o tablas, pero un número importante de los estudiantes no dan significado alguno al concepto, ni 
diferencian propiedades de los sistemas numéricos. Es el caso por ejemplo de los números reales, las preguntas 
referidas a su representación y estructura resultaron de especial complejidad. Se requiere un trabajo más cuidadoso 
con este concepto, que va mas allá de la introducción prematura de una árida clasificación. No hay que olvidar que 
este concepto es justamente uno de los pilares para la estructuración del pensamiento variacional al finalizar la 
educación media. 
El conocimiento que se pone de manifiesto respecto a los modelos funcionales se reduce generalmente al modelo 
lineal, los otros modelos generan problemas en el análisis y la representación. Sin embargo, las preguntas que 
indagan sobre variación lineal revisten dificultad para un grupo importante, este es un punto que merece atención, 
desde los grados de la básica, no es pertinente introducir listados de funciones diversas para manipular algebrai-camente 
sin que se haya logrado la apropiación del modelo fundamental. 
 Respecto al componente geométrico-métrico los estudiantes reconocen figuras planas, usan algunas de sus propie-dades 
y determinan áreas de figuras simples. Sin embargo tienen dificultades cuando se trata de aplicar resultados 
básicos como el teorema de Pitágoras en contextos no rutinarios, es necesario proponer más y variados problemas de 
aplicación en diversidad de situaciones, con miras a que el estudiante profundice en los conceptos y relaciones. 
Es de anotar que los teoremas básicos de semejanza y los del seno y el coseno se presentaron explícitamente (en 
profundización) con el objeto de apreciar la interpretación y aplicación, pero este análisis resulto muy complejo. Sería
50 
ANÁLISIS DE RESULTADOS DE 
LAS PRUEBAS DE ESTADO 
pertinente que en las aulas se planteen situaciones en las que se propongan definiciones, se analicen enunciados 
diversos donde se expresen condiciones, negaciones y se trabaje con proposiciones diversas relativas al pensamiento 
espacial. Es importante destacar que las herramientas de la lógica formal se construyen al interior de cada uno de 
los pensamientos no esquemáticamente en apartes dispersos del currículo. 
 En cuanto al componente aleatorio se refiere, se aprecia que en las aulas se está trabajando la lectura de gráficas, el 
uso de información tabular y gráfica y algunos aspectos relativos a las medidas de tendencia central, especialmente 
el promedio. Pero posiblemente falta ir más halla del cálculo, pues los estudiantes evaluados presentan dificultad para 
usar información y llegar a conclusiones, y en las preguntas que lo requerían tienen problemas para dar significado 
a las medidas de tendencia central y discutir su pertinencia. 
Un aspecto que posiblemente no se trabaja en las aulas y es muy importante en el desarrollo del pensamiento aleatorio, 
por la diversidad y riqueza de aplicaciones, es el relacionado con las nociones iniciales de conteo. Los problemas 
planteados sobre este tópico que se consideraban de un primer nivel de dificultad resultaron muy complejas, es posible 
que esto este relacionado con la poca o inexistente experiencia de trabajo con este tipo de tarea. Algo similar sucedió 
con las nociones iniciales de probabilidad, es importante que los docentes revisen los currículos, su coherencia con 
los Estándares Básicos de Competencias y que todos los aspectos mencionados sean ampliamente trabajados en 
los diferentes grados. 
5. Bibliografía 
ICFES, Grupo de Evaluación de la Educación Básica y Media, Serie Examen de Estado para Ingreso a la Educación Superior, 
Análisis de Resultados 2005, Matemáticas por ACEVEDO, Myriam y otros. Bogotá, Octubre de 2006. 
ICFES, Grupo de Evaluación de la Educación Básica y Media. ¿Qué evalúan las pruebas 2006?. Documento interno, página 
www.icfes.gov.co 
MEN, Serie Lineamientos Curriculares, Matemáticas. Bogotá, 1998. 
MEN, Documento N°3, Estándares Básicos de Competencias en Lenguaje, Matemáticas, Ciencias y Ciudadanas. Bogotá, 
2006.

More Related Content

What's hot

Examen de grado de estadistica 2014 Colegio de Bachillerato "DURÀN"
Examen de grado de estadistica 2014 Colegio de Bachillerato "DURÀN"Examen de grado de estadistica 2014 Colegio de Bachillerato "DURÀN"
Examen de grado de estadistica 2014 Colegio de Bachillerato "DURÀN"
Ángel Díaz Cevallos
 
Teoria y problemas de aumentos sucesivos as56 ccesa007
Teoria y problemas de aumentos sucesivos as56 ccesa007Teoria y problemas de aumentos sucesivos as56 ccesa007
Teoria y problemas de aumentos sucesivos as56 ccesa007
Demetrio Ccesa Rayme
 
Porcentajes - Aumentos y Descuentos sucesivos ccesa007
Porcentajes -  Aumentos y Descuentos sucesivos  ccesa007Porcentajes -  Aumentos y Descuentos sucesivos  ccesa007
Porcentajes - Aumentos y Descuentos sucesivos ccesa007
Demetrio Ccesa Rayme
 
Prueba matematicas, pre test 7° fraccionarios
Prueba matematicas, pre test 7° fraccionariosPrueba matematicas, pre test 7° fraccionarios
Prueba matematicas, pre test 7° fraccionarios
Carlos Mario Lopez Ramirez
 
Ejercicios tipo prueba racionales
Ejercicios tipo prueba racionalesEjercicios tipo prueba racionales
Ejercicios tipo prueba racionales
Mayra Alejandra
 
Prueba diagnostica de matematicas grado 11
Prueba diagnostica de matematicas grado 11Prueba diagnostica de matematicas grado 11
Prueba diagnostica de matematicas grado 11
Alvaro Soler
 

What's hot (20)

Ejercicios de-sistemas-de-ecuaciones-lineales (1)
Ejercicios de-sistemas-de-ecuaciones-lineales (1)Ejercicios de-sistemas-de-ecuaciones-lineales (1)
Ejercicios de-sistemas-de-ecuaciones-lineales (1)
 
Examen de grado de estadistica 2014 Colegio de Bachillerato "DURÀN"
Examen de grado de estadistica 2014 Colegio de Bachillerato "DURÀN"Examen de grado de estadistica 2014 Colegio de Bachillerato "DURÀN"
Examen de grado de estadistica 2014 Colegio de Bachillerato "DURÀN"
 
Teoria y problemas de aumentos sucesivos as56 ccesa007
Teoria y problemas de aumentos sucesivos as56 ccesa007Teoria y problemas de aumentos sucesivos as56 ccesa007
Teoria y problemas de aumentos sucesivos as56 ccesa007
 
ECUACION DE SEGUNDO GRADO
ECUACION DE SEGUNDO GRADOECUACION DE SEGUNDO GRADO
ECUACION DE SEGUNDO GRADO
 
ÁLGEBRA Pre San Marcos.pdf
ÁLGEBRA Pre San Marcos.pdfÁLGEBRA Pre San Marcos.pdf
ÁLGEBRA Pre San Marcos.pdf
 
Porcentajes - Aumentos y Descuentos sucesivos ccesa007
Porcentajes -  Aumentos y Descuentos sucesivos  ccesa007Porcentajes -  Aumentos y Descuentos sucesivos  ccesa007
Porcentajes - Aumentos y Descuentos sucesivos ccesa007
 
Taller problemas de aplicación sistemas de ecuaciones lineales 2x2
Taller problemas de aplicación sistemas de ecuaciones lineales 2x2Taller problemas de aplicación sistemas de ecuaciones lineales 2x2
Taller problemas de aplicación sistemas de ecuaciones lineales 2x2
 
Aumentos y descuentos sucesivos
Aumentos y descuentos sucesivosAumentos y descuentos sucesivos
Aumentos y descuentos sucesivos
 
Matemática4° medio - Guía de preparación PSU: área y volumen en cuerpos geomé...
Matemática4° medio - Guía de preparación PSU: área y volumen en cuerpos geomé...Matemática4° medio - Guía de preparación PSU: área y volumen en cuerpos geomé...
Matemática4° medio - Guía de preparación PSU: área y volumen en cuerpos geomé...
 
Prueba matematicas, pre test 7° fraccionarios
Prueba matematicas, pre test 7° fraccionariosPrueba matematicas, pre test 7° fraccionarios
Prueba matematicas, pre test 7° fraccionarios
 
Ejercicios tipo prueba racionales
Ejercicios tipo prueba racionalesEjercicios tipo prueba racionales
Ejercicios tipo prueba racionales
 
Unidad 2 . Seleccion sobre Polinomios
Unidad 2 . Seleccion sobre PolinomiosUnidad 2 . Seleccion sobre Polinomios
Unidad 2 . Seleccion sobre Polinomios
 
Prueba inecuaciones hoja 1 (autoguardado)
Prueba inecuaciones hoja 1 (autoguardado)Prueba inecuaciones hoja 1 (autoguardado)
Prueba inecuaciones hoja 1 (autoguardado)
 
Planteo de Ecuaciones.pdf
Planteo de Ecuaciones.pdfPlanteo de Ecuaciones.pdf
Planteo de Ecuaciones.pdf
 
65 ejercicios estadística y gráficos
65 ejercicios estadística y gráficos65 ejercicios estadística y gráficos
65 ejercicios estadística y gráficos
 
Materia "Datos Y Azar" 3ºC
Materia "Datos Y Azar" 3ºCMateria "Datos Y Azar" 3ºC
Materia "Datos Y Azar" 3ºC
 
Prueba diagnostica de matematicas grado 11
Prueba diagnostica de matematicas grado 11Prueba diagnostica de matematicas grado 11
Prueba diagnostica de matematicas grado 11
 
65 ejercicios estadística y gráficos
65 ejercicios estadística y gráficos65 ejercicios estadística y gráficos
65 ejercicios estadística y gráficos
 
68 probabilidades
68 probabilidades68 probabilidades
68 probabilidades
 
Examen de fundamentos de matemática
Examen de fundamentos de matemáticaExamen de fundamentos de matemática
Examen de fundamentos de matemática
 

Viewers also liked (6)

Prueba de matemáticas 2012
Prueba de matemáticas 2012Prueba de matemáticas 2012
Prueba de matemáticas 2012
 
Evaluación 9°
Evaluación 9°Evaluación 9°
Evaluación 9°
 
Ac ep matematicas_2010
Ac ep matematicas_2010Ac ep matematicas_2010
Ac ep matematicas_2010
 
Pregunta tipo icfes memoria
Pregunta tipo icfes memoriaPregunta tipo icfes memoria
Pregunta tipo icfes memoria
 
Ejemplo de preguntas saber 9 matematicas 2015
Ejemplo de preguntas saber 9 matematicas 2015Ejemplo de preguntas saber 9 matematicas 2015
Ejemplo de preguntas saber 9 matematicas 2015
 
Problemario funciones
Problemario funcionesProblemario funciones
Problemario funciones
 

Similar to Icfes 2006 matematica

Icfes fundamentación conceptual área de matemáticas
Icfes fundamentación conceptual área de matemáticasIcfes fundamentación conceptual área de matemáticas
Icfes fundamentación conceptual área de matemáticas
INDEIPCO LTDA
 
Matemáticas pisa 2012
Matemáticas pisa 2012Matemáticas pisa 2012
Matemáticas pisa 2012
secs técnicas
 
Anexo n° 0 protocolo estandares (1)
Anexo n° 0 protocolo estandares  (1)Anexo n° 0 protocolo estandares  (1)
Anexo n° 0 protocolo estandares (1)
milena1016
 
2065 matemticas2 eso
2065 matemticas2 eso2065 matemticas2 eso
2065 matemticas2 eso
EstudioTic
 

Similar to Icfes 2006 matematica (20)

Ac result 2005_analisis_matematicav2
Ac result 2005_analisis_matematicav2Ac result 2005_analisis_matematicav2
Ac result 2005_analisis_matematicav2
 
Propuestas para mejorar la evaluación de las competencias matemáticas al fina...
Propuestas para mejorar la evaluación de las competencias matemáticas al fina...Propuestas para mejorar la evaluación de las competencias matemáticas al fina...
Propuestas para mejorar la evaluación de las competencias matemáticas al fina...
 
Propuestas para mejorar la evaluación de las competencias matemáticas al fina...
Propuestas para mejorar la evaluación de las competencias matemáticas al fina...Propuestas para mejorar la evaluación de las competencias matemáticas al fina...
Propuestas para mejorar la evaluación de las competencias matemáticas al fina...
 
Presentación del Folleto de 6to. grado 2018.pptx
Presentación del Folleto de 6to. grado 2018.pptxPresentación del Folleto de 6to. grado 2018.pptx
Presentación del Folleto de 6to. grado 2018.pptx
 
Icfes fundamentación conceptual área de matemáticas
Icfes fundamentación conceptual área de matemáticasIcfes fundamentación conceptual área de matemáticas
Icfes fundamentación conceptual área de matemáticas
 
PCA 4 GRADO MATEMATICA 2023 - F.B.pdf
PCA 4 GRADO MATEMATICA 2023 - F.B.pdfPCA 4 GRADO MATEMATICA 2023 - F.B.pdf
PCA 4 GRADO MATEMATICA 2023 - F.B.pdf
 
Estándares en evaluación matemática.
Estándares en evaluación matemática.Estándares en evaluación matemática.
Estándares en evaluación matemática.
 
Matemáticas pisa 2012
Matemáticas pisa 2012Matemáticas pisa 2012
Matemáticas pisa 2012
 
PLAN DE MEJORA-4 grado- Año 2023- actual
PLAN DE MEJORA-4 grado- Año 2023- actualPLAN DE MEJORA-4 grado- Año 2023- actual
PLAN DE MEJORA-4 grado- Año 2023- actual
 
Legislacion educativa.pptx
Legislacion educativa.pptxLegislacion educativa.pptx
Legislacion educativa.pptx
 
Pisa
PisaPisa
Pisa
 
PISA 2015 - MARCO DE MATEMÁTICA
PISA 2015 - MARCO DE MATEMÁTICAPISA 2015 - MARCO DE MATEMÁTICA
PISA 2015 - MARCO DE MATEMÁTICA
 
Anexo n° 0 protocolo estandares (1)
Anexo n° 0 protocolo estandares  (1)Anexo n° 0 protocolo estandares  (1)
Anexo n° 0 protocolo estandares (1)
 
PROYECTO DE GRADO 11
PROYECTO DE GRADO 11PROYECTO DE GRADO 11
PROYECTO DE GRADO 11
 
Capitulo 8 procedimientos de evaluacion
Capitulo 8 procedimientos de evaluacionCapitulo 8 procedimientos de evaluacion
Capitulo 8 procedimientos de evaluacion
 
Dosificacion matematicaS 3er. grado
Dosificacion matematicaS 3er. gradoDosificacion matematicaS 3er. grado
Dosificacion matematicaS 3er. grado
 
PLAN DE REFORZAMIENTO 2023 hasta fin de año 2023 (1).docx
PLAN DE REFORZAMIENTO 2023 hasta fin de año 2023 (1).docxPLAN DE REFORZAMIENTO 2023 hasta fin de año 2023 (1).docx
PLAN DE REFORZAMIENTO 2023 hasta fin de año 2023 (1).docx
 
Trabajo calculo
Trabajo calculoTrabajo calculo
Trabajo calculo
 
Estructura del pete_pec
Estructura del pete_pecEstructura del pete_pec
Estructura del pete_pec
 
2065 matemticas2 eso
2065 matemticas2 eso2065 matemticas2 eso
2065 matemticas2 eso
 

More from juan vega

More from juan vega (20)

Violencia
ViolenciaViolencia
Violencia
 
Tema i 2
Tema i 2Tema i 2
Tema i 2
 
Sociales icfes
Sociales icfesSociales icfes
Sociales icfes
 
pruebas saber matematicas
pruebas saber matematicaspruebas saber matematicas
pruebas saber matematicas
 
pruebas saber matematicas 1
pruebas saber matematicas 1pruebas saber matematicas 1
pruebas saber matematicas 1
 
pruebas saber 2004
pruebas saber 2004pruebas saber 2004
pruebas saber 2004
 
Saber matematicas_escolares
Saber matematicas_escolaresSaber matematicas_escolares
Saber matematicas_escolares
 
pruebas Saber fundamento conceptual
pruebas Saber fundamento conceptualpruebas Saber fundamento conceptual
pruebas Saber fundamento conceptual
 
guia pruebas Saber matematicas
guia pruebas Saber matematicasguia pruebas Saber matematicas
guia pruebas Saber matematicas
 
Saber doc 2002_2003_evaluar_para_transformar_mat3
Saber doc 2002_2003_evaluar_para_transformar_mat3Saber doc 2002_2003_evaluar_para_transformar_mat3
Saber doc 2002_2003_evaluar_para_transformar_mat3
 
Saber doc 2002_2003_evaluar_para_transformar_mat2
Saber doc 2002_2003_evaluar_para_transformar_mat2Saber doc 2002_2003_evaluar_para_transformar_mat2
Saber doc 2002_2003_evaluar_para_transformar_mat2
 
Saber doc 2002_2003_evaluar_para_transformar_mat1
Saber doc 2002_2003_evaluar_para_transformar_mat1Saber doc 2002_2003_evaluar_para_transformar_mat1
Saber doc 2002_2003_evaluar_para_transformar_mat1
 
Saber 2002 matematica 7
Saber 2002 matematica 7Saber 2002 matematica 7
Saber 2002 matematica 7
 
Respuestas saber 2002
Respuestas saber 2002Respuestas saber 2002
Respuestas saber 2002
 
Respuestas pruebas saber año 2003
Respuestas pruebas saber año 2003Respuestas pruebas saber año 2003
Respuestas pruebas saber año 2003
 
Quimica 2005 icfes
Quimica 2005 icfesQuimica 2005 icfes
Quimica 2005 icfes
 
Quimica
Quimica Quimica
Quimica
 
Pruebas saber 2008 prueba piloto
Pruebas saber 2008 prueba pilotoPruebas saber 2008 prueba piloto
Pruebas saber 2008 prueba piloto
 
Pruebas saber 2003
Pruebas saber 2003Pruebas saber 2003
Pruebas saber 2003
 
Pruebas saber 2002
Pruebas saber 2002Pruebas saber 2002
Pruebas saber 2002
 

Recently uploaded

Concepto y definición de tipos de Datos Abstractos en c++.pptx
Concepto y definición de tipos de Datos Abstractos en c++.pptxConcepto y definición de tipos de Datos Abstractos en c++.pptx
Concepto y definición de tipos de Datos Abstractos en c++.pptx
Fernando Solis
 
TEMA 14.DERIVACIONES ECONÓMICAS, SOCIALES Y POLÍTICAS DEL PROCESO DE INTEGRAC...
TEMA 14.DERIVACIONES ECONÓMICAS, SOCIALES Y POLÍTICAS DEL PROCESO DE INTEGRAC...TEMA 14.DERIVACIONES ECONÓMICAS, SOCIALES Y POLÍTICAS DEL PROCESO DE INTEGRAC...
TEMA 14.DERIVACIONES ECONÓMICAS, SOCIALES Y POLÍTICAS DEL PROCESO DE INTEGRAC...
jlorentemartos
 

Recently uploaded (20)

Concepto y definición de tipos de Datos Abstractos en c++.pptx
Concepto y definición de tipos de Datos Abstractos en c++.pptxConcepto y definición de tipos de Datos Abstractos en c++.pptx
Concepto y definición de tipos de Datos Abstractos en c++.pptx
 
prostitución en España: una mirada integral!
prostitución en España: una mirada integral!prostitución en España: una mirada integral!
prostitución en España: una mirada integral!
 
Supuestos_prácticos_funciones.docx
Supuestos_prácticos_funciones.docxSupuestos_prácticos_funciones.docx
Supuestos_prácticos_funciones.docx
 
activ4-bloque4 transversal doctorado.pdf
activ4-bloque4 transversal doctorado.pdfactiv4-bloque4 transversal doctorado.pdf
activ4-bloque4 transversal doctorado.pdf
 
TEMA 14.DERIVACIONES ECONÓMICAS, SOCIALES Y POLÍTICAS DEL PROCESO DE INTEGRAC...
TEMA 14.DERIVACIONES ECONÓMICAS, SOCIALES Y POLÍTICAS DEL PROCESO DE INTEGRAC...TEMA 14.DERIVACIONES ECONÓMICAS, SOCIALES Y POLÍTICAS DEL PROCESO DE INTEGRAC...
TEMA 14.DERIVACIONES ECONÓMICAS, SOCIALES Y POLÍTICAS DEL PROCESO DE INTEGRAC...
 
Interpretación de cortes geológicos 2024
Interpretación de cortes geológicos 2024Interpretación de cortes geológicos 2024
Interpretación de cortes geológicos 2024
 
Biografía de Charles Coulomb física .pdf
Biografía de Charles Coulomb física .pdfBiografía de Charles Coulomb física .pdf
Biografía de Charles Coulomb física .pdf
 
1ro Programación Anual D.P.C.C planificación anual del área para el desarroll...
1ro Programación Anual D.P.C.C planificación anual del área para el desarroll...1ro Programación Anual D.P.C.C planificación anual del área para el desarroll...
1ro Programación Anual D.P.C.C planificación anual del área para el desarroll...
 
Posición astronómica y geográfica de Europa.pptx
Posición astronómica y geográfica de Europa.pptxPosición astronómica y geográfica de Europa.pptx
Posición astronómica y geográfica de Europa.pptx
 
SISTEMA RESPIRATORIO PARA NIÑOS PRIMARIA
SISTEMA RESPIRATORIO PARA NIÑOS PRIMARIASISTEMA RESPIRATORIO PARA NIÑOS PRIMARIA
SISTEMA RESPIRATORIO PARA NIÑOS PRIMARIA
 
Desarrollo y Aplicación de la Administración por Valores
Desarrollo y Aplicación de la Administración por ValoresDesarrollo y Aplicación de la Administración por Valores
Desarrollo y Aplicación de la Administración por Valores
 
Tema 19. Inmunología y el sistema inmunitario 2024
Tema 19. Inmunología y el sistema inmunitario 2024Tema 19. Inmunología y el sistema inmunitario 2024
Tema 19. Inmunología y el sistema inmunitario 2024
 
TRABAJO FINAL TOPOGRAFÍA COMPLETO DE LA UPC
TRABAJO FINAL TOPOGRAFÍA COMPLETO DE LA UPCTRABAJO FINAL TOPOGRAFÍA COMPLETO DE LA UPC
TRABAJO FINAL TOPOGRAFÍA COMPLETO DE LA UPC
 
SESION DE PERSONAL SOCIAL. La convivencia en familia 22-04-24 -.doc
SESION DE PERSONAL SOCIAL.  La convivencia en familia 22-04-24  -.docSESION DE PERSONAL SOCIAL.  La convivencia en familia 22-04-24  -.doc
SESION DE PERSONAL SOCIAL. La convivencia en familia 22-04-24 -.doc
 
CONCURSO NACIONAL JOSE MARIA ARGUEDAS.pptx
CONCURSO NACIONAL JOSE MARIA ARGUEDAS.pptxCONCURSO NACIONAL JOSE MARIA ARGUEDAS.pptx
CONCURSO NACIONAL JOSE MARIA ARGUEDAS.pptx
 
Prueba de evaluación Geografía e Historia Comunidad de Madrid 2º de la ESO
Prueba de evaluación Geografía e Historia Comunidad de Madrid 2º de la ESOPrueba de evaluación Geografía e Historia Comunidad de Madrid 2º de la ESO
Prueba de evaluación Geografía e Historia Comunidad de Madrid 2º de la ESO
 
Plan-de-la-Patria-2019-2025- TERCER PLAN SOCIALISTA DE LA NACIÓN.pdf
Plan-de-la-Patria-2019-2025- TERCER PLAN SOCIALISTA DE LA NACIÓN.pdfPlan-de-la-Patria-2019-2025- TERCER PLAN SOCIALISTA DE LA NACIÓN.pdf
Plan-de-la-Patria-2019-2025- TERCER PLAN SOCIALISTA DE LA NACIÓN.pdf
 
Sesión de clase APC: Los dos testigos.pdf
Sesión de clase APC: Los dos testigos.pdfSesión de clase APC: Los dos testigos.pdf
Sesión de clase APC: Los dos testigos.pdf
 
Linea del tiempo - Filosofos Cristianos.docx
Linea del tiempo - Filosofos Cristianos.docxLinea del tiempo - Filosofos Cristianos.docx
Linea del tiempo - Filosofos Cristianos.docx
 
Tema 11. Dinámica de la hidrosfera 2024
Tema 11.  Dinámica de la hidrosfera 2024Tema 11.  Dinámica de la hidrosfera 2024
Tema 11. Dinámica de la hidrosfera 2024
 

Icfes 2006 matematica

  • 1. INSTITUTO COLOMBIANO PARA EL FOMENTO DE LA EDUCACIÓN SUPERIOR ICFES SUBDIRECCIÓN ACADÉMICA GRUPO DE EVALUACIÓN DE LA EDUCACIÓN BÁSICA Y MEDIA MATEMÁTICA ANÁLISIS DE RESULTADOS 2006 EXAMEN DE ESTADO PARA INGRESO A LA EDUCACIÓN SUPERIOR Myriam Margarita Acevedo Caicedo (Universidad Nacional de Colombia) María Cristina Pérez Camacho (Profesora pensionada Secretaría de Educación Distrital de Bogotá) Grace Judith Vesga Bravo (ICFES) Bogotá, mayo 2007
  • 2. ANÁLISIS DE RESULTADOS 2006 Grupo de Evaluación de la Educación Superior - ICFES Claudia Lucia Sáenz Blanco Grupo de Evaluación de la Educación Básica y Media - ICFES Flor Patricia Pedraza Daza  ICFES ISSN: 1909-3993 Diseño y diagramación: Secretaría General, Grupo de Procesos Editoriales - ICFES
  • 3. ALVARO URIBE VÉLEZ Presidente de la República Francisco Santos Calderón Vicepresidente de la República CECILIA MARÍA VÉLEZ WHITE Ministra de Educación Nacional INSTITUTO COLOMBIANO PARA EL FOMENTO DE LA EDUCACIÓN SUPERIOR Directora General MARGARITA PEÑA BORRERO Secretario General GENISBERTO LÓPEZ CONDE Subdirector de Logística FRANCISCO ERNESTO REYES JIMÉNEZ Subdirector Académico JULIÁN PATRICIO MARIÑO VON HILDEBRAND Oficina Asesora de Planeación CLAUDIA NATALIA MUJICA CUELLAR Oficina Asesora Jurídica MARTHA ISABEL DUARTE DE BUCHHEIM Oficina de Control Interno LUIS ALBERTO CAMELO CRISTANCHO
  • 4. ANÁLISIS DE RESULTADOS DE LAS PRUEBAS DE ESTADO 1 Introducción Presentamos este segundo libro sobre el análisis de los resultados de las pruebas de matemáticas que hacen parte del Examen de Estado para Ingreso a la Educación Superior, los datos se refieren a las aplicaciones realizadas en el 2006. Este documento está dirigido especialmente a los maestros y maestras, estudiantes, padres de familia y en general a toda la comunidad interesada en conocerlos y estudiarlos. En primer lugar se resaltan los cambios de la prueba de núcleo común a partir del 2006, a continuación se presenta la es-tructura de la misma y los resultados nacionales, así como los resultados en la prueba de profundización. A continuación se incluye el análisis de algunas preguntas tanto de núcleo común como de profundización por cada uno de los componentes evaluados. Al final se presentan algunas conclusiones y recomendaciones. 2 Estructura de prueba y resultados nacionales, contrastes entre dos calendarios Es importante resaltar los cambios que se hicieron a esta prueba a partir de la primera aplicación del 2006 y que fueron anunciados y socializados por medio, de talleres en algunas regiones del país, del análisis de resultados del 2005 y más general y ampliamente a través del documento “¿Qué evalúan las pruebas 2006?” publicado y disponible en www.icfes. gov.co. Los cambios fueron básicamente: • Agrupación y redefinición de los componentes evaluados. Estos pasaron de ser conteo, medición, variación y alea-toriedad a conformar tres numérico-variacional, geométrico-métrico y aleatorio. El cambio obedece básicamente al interés de presentar una organización de los componentes en torno a los pensamientos propuestos tanto en el documento de Lineamientos Curriculares como de Estándares de Calidad del área de matemática. Por otra parte, se presentan estas agrupaciones por considerarlas pertinentes para los contextos de evaluación, sin embargo, es importante aclarar que, en el trabajo en el aula, un mismo contexto puede tener elementos para indagar aspectos relacionados con lo numérico, lo métrico y lo aleatorio, por lo tanto no se pretende que esos componentes sean los organizadores curriculares. • Redefinición de las competencias a evaluar. Es bien conocido que desde el año 2000 se evalúan en todas las áreas las competencias interpretativas, argumentativas y propositivas, competencias generales y transversales; sin em-bargo, en el área y partiendo de estas competencias, se definieron competencias específicas relacionadas con los procesos que en matemática se realizan y tomando como punto de referencia los propuestos en el documento de Lineamientos Curriculares, estas son: Competencia en comunicación y representación, competencia en razonamiento y argumentación y competencia en modelación, planteamiento y solución de problemas. Estos cambios no implican que la prueba se haya transformado radicalmente, el objeto de evaluación sigue siendo la competencia matemática; son cambios con los cuales se espera que los maestros y maestras puedan tener mayor claridad sobre lo evaluado y su coherencia con lo propuesto por el Ministerio de Educación Nacional. 2.1 Objeto de evaluación El objeto de evaluación de las pruebas es la competencia matemática relacionada con el uso flexible y comprensivo del conocimiento matemático escolar en diversidad de contextos, de la vida diaria, de la matemática misma y de otras ciencias. Este uso se evidencia, entre otros, en la capacidad del individuo para analizar, razonar, y comunicar ideas efectivamente y para formular, resolver e interpretar problemas. En la pruebas un aspecto importante a evaluar es el significado de los conceptos matemáticos y la práctica significativa,
  • 5. ANÁLISIS DE RESULTADOS DE LAS PRUEBAS DE ESTADO relacionada esta última con la matematización que exige al estudiante simbolizar, formular, cuantificar, validar, esquematizar, representar, generalizar, entre otros. Actividades que le permitirán desarrollar descripciones matemáticas, explicaciones o construcciones. Desde estas perspectivas, en las pruebas se propusieron problemas que indagaban tanto por el conocimiento matemático que ha logrado consolidar el estudiante, como por los procesos de pensamiento. Se exploró el uso de la matemática en contextos que permitieran mediante procesos de matematización reconocer los conceptos y estructuras construidos en la matemática escolar. Se asumió para la construcción de la prueba la propuesta integradora de los Lineamientos Curriculares y los Estándares Básicos de Competencia, respecto a los conocimientos básicos, procesos y contextos. En cada uno de los contextos seleccionados se propusieron problemas tanto rutinarios como no rutinarios, con distintos niveles de complejidad, con la intención de destacar la importancia de enfrentar al estudiante a situaciones diversas que exijan comprensión y uso significativo de los conceptos y procedimientos y que den la posibilidad de seleccionar caminos o estrategias diversas para su solución. 2.2 Componentes evaluados y resultados en el núcleo común Para la estructuración de las pruebas se organizaron los pensamientos que se proponen en los Lineamientos Curriculares y Estándares Básicos de Competencias en tres componentes: el numérico-variacional, el geométrico-métrico y el aleatorio, tomando como referente fundamental los estándares relativos a cada pensamiento en los distintos grupos de grados. En lo relativo al componente numérico-variacional, se indagó por la compresión y uso de los números y de la numeración, más específicamente por la representaciones decimales de los números reales, el reconocimiento de la densidad de los números reales, y la comprensión de las operaciones y sus propiedades y su uso en la resolución de problemas. Se inda-gó además, por la identificación de variables, la descripción de fenómenos de cambio y la modelación de situaciones de cambio a través de funciones. En lo pertinente al eje geométrico-métrico se indagó por el reconocimiento de propiedades y relaciones geométricas, la identificación gráfica y algebraica de propiedades de las cónicas (particularmente la parábola), la identificación de ca-racterísticas de localización en sistemas de representación cartesiana, la resolución de problemas usando propiedades geométricas y la comprensión de conceptos de área y volumen. En lo relativo al pensamiento aleatorio se indagó por la representación, lectura e interpretación de datos en contexto; el análisis de diversas formas de representación de información numérica, el reconocimiento, descripción y análisis de eventos aleatorios y el uso de técnicas de conteo. Es importante anotar que cada pensamiento desarrolla habilidades específicas en los estudiantes, relacionadas con sus sistemas de representación, con las estructuras conceptuales y con las formas propias de argumentación, por lo tanto ninguno de ellos puede ser excluido ni del proceso educativo ni del evaluativo. En la gráfica 1 se muestran los promedios de la prueba en las cuatro últimas aplicaciones, se puede observar que en ambos calendarios hubo un avance en el promedio entre 2005 y 2006, destacándose especialmente calendario B con aproxima-damente 5 puntos, aunque también es el calendario en el que más aumenta la dispersión como puede observarse en la gráfica 2, esto muestra que el incremento del promedio se debe a que un grupo de personas obtiene mejores resultados pero se abre más la brecha entre altos y bajos puntajes.
  • 6. ANÁLISIS DE RESULTADOS DE LAS PRUEBAS DE ESTADO Gráfica 1. Promedio 2005 y 2006 45.00 44.27 50.77 45.42 55 50 45 40 2005-1 2005-2 2006-1 2006-2 Gráfica 2. Desviación 2005 y 2006 8.37 8.77 9.64 7.92 10 9 8 7 2005-1 2005-2 2006-1 2006-2 En la gráfica 3 se muestra el porcentaje de estudiantes acumulado en cada rango de puntaje al nivel nacional en los dos calendarios para el año 2006. Se considera Bajo un puntaje menor o igual a 30 puntos, Medio mayor a 30 y hasta 70 y Alto mayor a 70.
  • 7. ANÁLISIS DE RESULTADOS DE LAS PRUEBAS DE ESTADO Gráfica 3. Porcentaje acumulado de estudiantes en cada rango de puntaje 100 80 60 40 20 0 Hasta 30 Hasta 35 Hasta 40 Hasta 45 Hasta 50 Hasta 55 Hasta 60 Hasta 65 Hasta 70 Mayor a 70 Calendario B 2.82 6.06 12.64 23.11 50.06 73.41 86.43 92.88 96.01 100 Calendario A 3.28 8.05 30.68 46.33 73.8 89.17 95.84 98.48 99.52 100 Para una mejor interpretación de la información, es importante recordar que un estudiante ubicado en categoría baja logra abordar situaciones rutinarias que exigen analizar información puntual y establecer estrategias directas que se caracterizan por tener una sola relación, operación o algoritmo para su resolución. Un estudiante ubicado en categoría media utiliza aspectos básicos de la matemática escolar, en contextos de no rutina que le exige relacionar y organizar información, utilizar diferentes formas de representación y hacer traducciones entre ellas. Y un estudiante ubicado en categoría alta muestra capacidad de aplicar los elementos básicos de la matemática escolar en contextos diversos y no rutinarios, rela-cionar información, reconocer condiciones y hacer inferencias y generalizaciones, esto es, involucra conceptualizaciones más formales. Se observa en la gráfica 3, que el porcentaje de estudiantes ubicados en bajo corresponde a 2.82 en calendario B y 3.28 en calendario A, en el nivel medio al 93.19 y 96.24 respectivamente, por lo tanto al nivel alto sólo llega el 3.49% y el 0.48%. Más del 50% de la población sólo alcanza puntajes hasta de 50 puntos y más del 90% llega sólo hasta 65 puntos. Esta gráfica evidencia que aunque el promedio nacional se ha incrementado, una gran mayoría de la población se ubica en puntajes de la categoría media. 2.3 Competencias evaluadas y resultados en núcleo común A continuación se describe cada una de las competencias evaluadas en el 2006 y los resultados obtenidos. En relación con la competencia en comunicación y representación, las pruebas exploraron la capacidad de los estu-diantes para establecer relaciones entre materiales físicos e ideas matemáticas, expresar conceptos matemáticos utili-zando ilustraciones y para traducir del lenguaje natural al lenguaje simbólico; abordaron, además, aspectos tales como la descripción cualitativa y cuantitativa de fenómenos de variación presentados en diferentes contextos mediante diversas representaciones (reglas verbales, tablas, gráficas, simbólicas). En lo referente a la competencia en razonamiento y argumentación se exploró por la capacidad del estudiante para dar cuenta del cómo y del porqué de las estrategias o procedimientos puestos en acción para llegar a conclusiones. Se indagó también por aspectos tales como la capacidad para generalizar propiedades y relaciones, reconocer patrones y expresarlos matemáticamente. En cuanto a la modelación, planteamiento y resolución de problemas, las pruebas exploran el diseño y aplicación de estrategias diversas para dar solución a problemas planteados en contextos dentro y fuera de la matemática, la verificación e interpretación de resultados de acuerdo con las condiciones iniciales del problema y la generalización de soluciones y estrategias.
  • 8. ANÁLISIS DE RESULTADOS DE LAS PRUEBAS DE ESTADO En las gráficas 5, 6 y 7 se observa que por lo menos el 90% de los estudiantes se ubica en los dos primeros grados, con más del 40% en el grado I, es decir solo logran realizar las tareas propias de cada competencia de manera muy inicial y en contextos rutinarios. Gráfica 4. Porcentaje de estudiantes ubicados en cada grado en la competencia comunicación y representación I II III Calendario B 46.62 40.13 11.94 Calendario A 17.34 79.96 1.65 Gráfica 5. Porcentaje de estudiantes ubicados en cada grado en la competencia en razonamiento y argumentación I II III Calendario B 42.06 51.13 5.51 Calendario A 29.60 68.37 0.91 Aproximadamente el 12% de los estudiantes de calendario B se ubican en el nivel alto en la competencia en comunicación y representación, sin embargo, en calendario A aunque no se llega ni al 2% se destaca que en ésta competencia se ubican muchos más estudiantes en nivel medio, el 80%, y menos en nivel bajo el 17%, frente a 40% en medio y 47% en bajo para calendario B. Tendencias similares se observan en las otras competencias.
  • 9. ANÁLISIS DE RESULTADOS DE LAS PRUEBAS DE ESTADO Gráfica 6. Porcentaje de estudiantes ubicados en cada grado en la competencia de modelación, planteamiento y resolución de problemas I II III Calendario B 55.82 38.33 4.54 Calendario A 37.10 58.69 3.16 2.4 Resultados en profundización En la profundización se plantean situaciones que exigen al estudiante comprensión de los conceptos y estructuras matemáticas básicas. Se indaga con mayor énfasis por la manipulación de proposiciones y expresiones que contienen símbolos y fórmulas, la generalización de propiedades y relaciones, la interpretación y uso de definiciones y relaciones y por el análisis de cadenas de argumentos. Se enfatiza además en el lenguaje simbólico formal y en las diferentes formas de representación. En las aplicaciones del 2006 esta prueba se centró en dos componentes el geométrico-métrico y el numérico-variacional. Respecto al primero, se indagó por el uso de argumentos geométricos en la solución de problemas, la descripción de lugares geométricos, aplicación de criterios de semejanza, el reconocimiento de propiedades de las cónicas, la solución de triángulos usando teoremas básicos presentados explícitamente en la prueba. Y con respecto al componente numérico-variacional se indagó por el análisis de relaciones entre expresiones algebraicas y gráficas de funciones, y por la modelación de situaciones de variación. Como se ha comentado en documentos anteriores, no se indaga por conocimientos de un primer semestre universitario sino, por los conocimientos y procesos que se proponen en los Lineamientos Curriculares y Estándares Básicos de Com-petencias. En esta prueba cada estudiante se ubica de acuerdo a su desempeño en un grado: básico, I, II ó III y recibe además un puntaje de 0 a 10. En la gráfica 4, se observa que en ambas aplicaciones más del 60 % de los estudiantes quedan ubicados en grado I o grado II y que el porcentaje de estudiante ubicados en nivel básico, es decir aquellos que no alcanzan los requisitos mínimos para ubicarse en el grado I, es aún muy alto especialmente en calendario A. Es importante destacar que el 15% de los estudiantes de calendario B quedaron ubicados en grado III, esto es un buen muy resultado, teniendo en cuenta que en general sólo llega al 2%. Los estudiantes que se ubican el grado I son capaces de utilizar de manera directa una fórmula, teorema o expresión o La profundización es una prueba electiva que escogen los estudiantes entre lenguaje, biología, ciencias sociales o matemática. En el 2006, tan sólo el 17% eligió profundizar en matemática en calendario B y el 15% en calendario A.
  • 10. ANÁLISIS DE RESULTADOS DE LAS PRUEBAS DE ESTADO propiedad dada. Hacer traducciones de lenguaje natural a lenguaje simbólico. Reconocer un patrón, argumentaciones no formales a partir de gráficas o información numérica, las propiedades de figuras geométricas. Hacer inferencias directas a partir de información gráfica o numérica. Identificar la simbología propia de la matemática. Los que se ubican en grado II son capaces de combinar expresiones o relaciones, usar propiedades geométricas, aritméticas y de variación que requieren transformación de expresiones. Generalizar o identificar una regla, fórmula o modelo que no requiere el uso de varios conceptos de manera simultánea. Realizar procesos de reversibilidad en contextos rutinarios. Hacer traducciones del lenguaje natural y simbólico al gráfico y viceversa. Hacer inferencias que requieren el establecimiento de una o varias relaciones o propiedades. Gráfica 7. Porcentaje de estudiantes en cada grado Grado Básico I II III Calendario B 13.5 32.95 37.43 15.01 Calendario A 34.06 44.29 18.58 1.71 Y los estudiantes que se ubican en grado III pueden aplicar teoremas que requieren relaciones con otros conceptos. Realizar procesos de reversibilidad en contextos no rutinarios y que requieren establecer varias relaciones. Elaborar argumentos, seguir reglas de inferencia. Están en capacidad de utilizar formas de argumentación formales, interpretar condicionales y uso de cuantificadores. Modelar situaciones aplicando la definición analítica de las curvas. Solucionar problemas que requieran el uso simultáneo de relaciones y conceptos de diferentes pensamientos. 10
  • 11. 11 ANÁLISIS DE RESULTADOS DE LAS PRUEBAS DE ESTADO 3 Análisis de preguntas A continuación se presenta el análisis de algunas de las preguntas que se propusieron a los estudiantes que presentaron la prueba en el 2006, hay ejemplos, tanto de núcleo común como de profundización, de ambas aplicaciones. Se espera que este tipo de análisis aporte elementos que le permita especialmente a los maestros y maestras y a los estudiantes, estudiar con mayor profundidad la prueba. Por cada una de los ejemplos analizados se describe lo que se evaluó, el nivel de exigencia, se presentan posibles estrategias de solución y el porcentaje de estudiantes que eligió cada opción de respuesta, no se incluyen los datos correspondientes a omisiones o multimarcas. Estos ejemplos se han agrupado por componente, se trata de presentar tanto para núcleo común como para profundización una mirada exhaustiva a los aspectos que se indagaron al interior de cada uno. 3.1 Ejemplos componente Numérico-Variacional A continuación se presentan 8 ejemplos entre núcleo común y profundización. Contexto para los ejemplos 1 y 2 Números racionales e irracionales Si usted no es matemático y no tiene ninguna relación con la matemática, las definiciones de número racional y número irracional no le impresionarán demasiado. Número racional es aquél que se puede expresar como cociente de dos números enteros, mientras que número irracional es aquel que no admite una expresión de este tipo. Los números racionales e irracionales, constituyen lo que se conoce como números reales y se pueden expresar en forma decimal y ordenar sobre una línea que se denomina la recta real. Cuando escribimos 2 o cualquier otro número irracional en forma decimal, encontramos que su desarrollo infinito no consiste en un grupo de cifras que se repite periódicamente. Por el contrario, los números racionales tienen sucesiones de dígitos que se repiten. Los números 5.3, 0.875, 0.3846 son todos números racionales, sus cifras decimales se repiten. Las expresiones decimales de 2,  ,e, presentan dicha repetición. En el conjunto de todas las expresiones decimales (es decir, en el conjunto de todos los números reales) es mucho mas raro que haya una pauta y una repetición que la ausencia de las mismas. La armonía es siempre mucho más rara que la cacofonía.
  • 12. 12 ANÁLISIS DE RESULTADOS DE LAS PRUEBAS DE ESTADO Ejemplo 1, núcleo común Componente: Numérico – Variacional Porcentaje por opciones de respuesta Competencia: Razonamiento A B C D Clave: A 19 6 38 36 Esta pregunta indaga por el dominio que tienen los estudiantes del conjunto de los números reales, sus propiedades y re-laciones, específicamente por la distinción entre números racionales y números irracionales. Exige generalizar propiedades y relaciones y explicar usando hechos y propiedades. Se esperaría que cualquier estudiante al terminar el grado once, incluso desde grado noveno, pueda responder la pregunta por los conocimientos previos que tiene de los números reales, sin embargo la lectura ofrece la posibilidad de hacerlo sin recurrir a éstos. En la lectura se explica claramente la diferencia entre un número racional y uno irracional, como justamente 0.5 es un decimal periódico se puede concluir inmediatamente que representa un número racional, así las opciones B y 5 = se concluye que la opción correcta es A. C no pueden ser correctas. Por otra parte como 0.5 es menor que 0.625 8 También puede descartarse la opción D porque la expresión decimal de 0.5 es infinita y periódica. Tan sólo el 16% de la población seleccionó la opción correcta y un 74% se distribuyó entre las opciones C y D, posiblemente recordaron parte de la lectura o del conocimiento de aula, pero no lograron establecer las diferencias. Es necesario hacer un llamado de atención sobre este punto, desde el séptimo grado (muy prematuramente) el estudiante memoriza una regla sobre la clasificación de los reales, pero no logra a través de los años darle significado. Se requiere un trabajo mas estructurado con este concepto, pues es uno de los pilares del pensamiento variacional en los últimos grados.
  • 13. ( 2 1) 2 = + − , que no es un irracional, por lo tanto la opción 13 ANÁLISIS DE RESULTADOS DE LAS PRUEBAS DE ESTADO Ejemplo 2, núcleo común En la recta numérica que se muestra se han localizado dos números reales y . La afirmación “entre los puntos P y Q es posible ubicar otro número irracional” es A. falsa, porque es el siguiente de 2 . B. verdadera, porque un irracional que está entre P y Q es 3 . C. falsa, porque solo se pueden ubicar racionales entre P y Q. D. verdadera, porque un irracional que está entre P y Q es . Componente: Numérico – Variacional Porcentaje por opciones de respuesta Competencia: Comunicación A B C D Clave: B 19 28 26 28 Este ítem explora de nuevo el concepto de número real pero ahora específicamente el manejo de las operaciones y sus propiedades, y la noción de densidad. Respecto a la competencia indaga por la interpretación y relación de diferentes representaciones. En primer lugar, si el estudiante tiene clara una noción inicial de densidad y ha interpretado la lectura, debe saber que el enunciado propuesto es verdadero, pues entre dos reales cualesquiera hay infinitos reales, por lo tanto es posible ubicar por lo menos un irracional. De donde se concluye que las opciones A y C no son correctas. La opción D es falsa porque la justificación que se propone para argumentar es falsa: 1 2 2 correcta es B. Otra forma de validar la respuesta es utilizando una aproximación de los números irracionales así: , y . Los porcentajes para cada una de las opciones de respuesta estuvieron, muy próximos –un poco mas bajo para la A, 19%– , esto podría sugerir más una selección al azar que un análisis de la pregunta. De nuevo los estudiantes recitan una frase relativa a la densidad de los reales en la recta, pero posiblemente por la complejidad de la idea o la no ilustración de la misma en la práctica escolar, no han logrado interpretarla. Es necesario incluir en el aula no sólo la definición clásica de número real, sino el desarrollo de ejercicios variados y de diferentes niveles de dificultad que permitan una verdadera aproximación de este concepto por parte de los estudiantes.
  • 14. 14 ANÁLISIS DE RESULTADOS DE LAS PRUEBAS DE ESTADO Ejemplo 3, núcleo común Deforestación En la última década se ha observado que debido a la deforestación, la extensión de un bosque se ha venido reduciendo aproximadamente en un 10% anual. Ac-tualmente el bosque tiene una extensión de 200 Km2. La expresión que representa la extensión E del bosque en función del tiempo t es A. E = 200 (0,9)t B. E = 200 (0,1)t C. E = 200 – 0,2t D. E = 200 – 0,8t Componente: Numérico – Variacional Porcentaje por opciones de respuesta Competencia: Solución de problemas A B C D Clave: A 19 31 36 13 En esta pregunta se indaga por la construcción de una expresión matemática que representa la determinación del modelo y la generalización de un proceso de deforestación, dicha expresión permite identificar la relación de variación existente entre dos variables. Para dar solución al problema el estudiante puede observar la regularidad, aplicando de manera consecutiva las condiciones dadas, construyendo así una secuencia relativa a la extensión del bosque a medida que transcurren los años: Extensión actual del bosque E = 200 Extensión del bosque dentro de 1 año E = 200 x 0.9 Extensión del bosque dentro de 2 años E = 200 x 0.9 x 0.9 Extensión del bosque dentro de 3 años E = 200 x 0.9 x 0.9 x 0.9 Extensión del bosque dentro de t años E = 200 x 0.9 x 0.9 x ... x 0.9 Determinando, de esta manera, que la respuesta correcta es A. Los estudiantes que seleccionan la opción B no entienden completamente las condiciones del problema y hacen una traducción literal de éste, sin embargo entiende que el modelo es exponencial. Aproximadamente el 50% de los estudiantes escogen las opciones C o D, lo cual muestra que sólo conocen el modelo lineal, una forma de obtener la opción C es tomar como valor de decrecimiento el 10% de 200 y para la opción D tomar el 10% de 200 y trabajar con el complemento.
  • 15. 15 ANÁLISIS DE RESULTADOS DE LAS PRUEBAS DE ESTADO Contexto para los ejemplos 4 y 5 Ruta Bogotá Cúcuta El siguiente gráfico muestra una ruta para ir desde Bogotá a Cúcuta vía terrestre. En el gráfico aparece información sobre: distancias, temperaturas y alturas. Ejemplo 4, núcleo común A partir de la información de la gráfica se puede afirmar que la ciudad que está a una altura mayor de 2.000m, tiene una temperatura promedio menor que 17°C y está a más de 500Km de Bogotá es A. Tunja B. Cúcuta. C. Pamplona. D. Bucaramanga. Componente: Numérico – Variacional Porcentaje por opciones de respuesta Competencia: Comunicación A B C D Clave: C 16 6 73 5 La pregunta explora por la capacidad del estudiante para interpretar y usar diferentes tipos de representación, esta asociada con estándares referidos al planteamiento y solución de situaciones utilizando argumentos que justifiquen relaciones entre información numérica. Para dar solución a la pregunta, es necesario identificar, en el gráfico presentado, la información referida a diferentes ciudades en la ruta Bogotá-Cúcuta respecto a su distancia de Bogotá (en kilómetros), su altura sobre el nivel del mar (en metros) y su temperatura (en grados centígrados). Dadas las condiciones del problema, el estudiante debe interpretar esta información, y establecer que la altura sobre el nivel del mar y la temperatura se relacionan de manera inversa, para después buscar la ciudad de la gráfica (más específicamente de las presentadas en las opciones) la ciudad, que cumple simultáneamente las condiciones de estar a una altura mayor de 2.000m tener una temperatura menor de 17°C y estar a más de 500Km de Bogotá y concluir que la respuesta correcta es Pamplona.
  • 16. ANÁLISIS DE RESULTADOS DE LAS PRUEBAS DE ESTADO El 73% de los estudiantes seleccionaron C, la opción correcta, demostrando capacidad para leer interpretar y relacionar información numérica que caracteriza diferentes lugares (ciudades, poblaciones y peajes) en un mapa vial; como puede observarse este ítem está asociado con la aplicación de conceptos matemáticos en la vida cotidiana. Los estudiantes que seleccionaron la opción A, Tunja, el 16%, equivocaron su lectura respecto a la distancia a Bogotá, las otras dos condiciones se cumplen; aquellos que seleccionan la opción B, el 6%, equivocan la interpretación de dos de las condiciones pues Cúcuta no está a más de 2.000 metros de altura, ni tiene una temperatura menor de 17°. Finalmente, la selección de la opción D, Bucaramanga, realizada por el 5% de los estudiantes, indica que no entienden las condiciones del enunciado o no establecen relaciones de orden entre los números que indican altura, distancia y temperatura, pues Bucaramanga no cumple ninguna de las condiciones solicitadas. Ejemplo 5, núcleo común Si un automóvil se desplazara a una velocidad constante durante todo el trayecto (Bogotá – Cúcuta), el tramo en el cual la rapidez de variación de la altura es mayor es A. Tunja – Arcabuco. B. San Gil – Aratoca. C. Pamplona – El diamante. D. Pescadero – Bucaramanga. Componente: Numérico – Variacional Porcentaje por opciones de respuesta Competencia: Razonamiento A B C D Clave: C 14 21 53 12 La pregunta indaga por aspectos relacionados con el concepto de variación, específicamente por la razón de cambio. Permite explorar la capacidad del estudiante para justificar estrategias y dar cuenta del cómo y del porqué de los procedimientos que se siguen para llegar a conclusiones. El estudiante debe identificar el tramo recorrido por un automóvil en el cual la altura varía con mayor rapidez, suponiendo que el automóvil se desplaza con velocidad constante. Para encontrar la respuesta correcta es necesario hallar los cocientes entre diferencias de altura y de distancias recorridas (de la ciudad o población donde se inicia cada uno de los tramos y la ciudad o población donde terminan), o lo que es lo mismo hallar la pendiente de los segmentos que representan cada tramo para determinar cual es la mayor. Una simple observación de los cocientes que representan la rapidez de variación de la altura en cada uno de los tramos permite determinar que el trayecto desde Pamplona hasta el Diamante presenta la mayor rapidez de variación en la altura: 16 Tunja – Arcabuco: San Gil – Aratoca: Pamplona El Diamante:
  • 17. 17 ANÁLISIS DE RESULTADOS DE LAS PRUEBAS DE ESTADO Pescadero – Bucaramanga: Es posible llegar a esta misma conclusión observando en el mapa el segmento de recta con mayor inclinación (el más pendiente o empinado). El 53% de los estudiantes seleccionó C la opción correcta, demostrando capacidad para hacer la lectura de la información numérica que aparece en la ilustración y compresión del concepto de razón de cambio en este contexto. El 21% de los estudiantes seleccionó la opción B San Gil – Aratoca, que es el tramo que le sigue a Pamplona – El Diamante en cuanto a rapidez de variación, es posible que su afirmación esté apoyada en la observación directa del grado de inclinación del segmento que representa el recorrido. La selección de las opciones A o D hecha por el 14% y el 12% de los estudiantes pudo haber sido hecha al azar o por interpretación incorrecta de las condiciones. Contexto para los ejemplos 6 y 7 Caída de un objeto Si un objeto con masa m se deja caer, y se tiene en cuenta la resistencia del aire, una función que describe la velocidad v del objeto después de t segundos es donde g es la aceleración de la gravedad y c y e son constantes positivas. Ejemplo 6, profundización A medida que transcurre el tiempo, la velocidad del objeto A. permanece constante. B. disminuye y se aproxima a cero. C. disminuye y se aproxima a mg . c D. aumenta y se aproxima a mg . c Componente: Numérico - Variacional Porcentaje por opciones de respuesta Competencia: Comunicación A B C D Clave: D 23 15 8 53 Este ítem explora aspectos referidos a la modelación de situaciones de variación con funciones exponenciales y al uso de técnicas de aproximación en procesos infinitos numéricos. Está relacionado además con la capacidad del estudiante para interpretar, y usar diferentes tipos de lenguaje, describir relaciones y manipular expresiones simbólicas.
  • 18. 18 ANÁLISIS DE RESULTADOS DE LAS PRUEBAS DE ESTADO Para resolver la pregunta el estudiante debía tomar la expresión y analizar tendencias: como , cuando t crece (tiende a infinito) tiende a 0 y tiende a 1, de donde crec e y se aproxima a c mg , es decir la clave es D. Un porcentaje alto, el 53%, respondió bien la pregunta, pero si la selección partiera de una comprensión de la variación esta respuesta debería ser coherente con la selección de la gráfica, pero esto no ocurrió (ver análisis ejemplo 9), puede mg ser que al observar el tipo de función concluyan que aumenta, pero no reconozcan específicamente la asíntota c . Es importante sin embargo destacar el análisis logrado para una función poco familiar. El 23% no logra interpretar el modelo y asume que v es constante posiblemente porque en el enunciado se hizo alusión a algunas constantes. El 15% que selec-ciona B y el 8% que seleccionan C pueden haber realizado un análisis parcial de la tendencia por ejemplo la exponencial decrece o se aproxima a cero y seleccionar por ello las opciones mencionadas. Ejemplo 7, profundización La gráfica que relaciona la velocidad v del objeto con el tiempo t es Componente: Numérico - Variacional Porcentaje por opciones de respuesta Competencia: Comunicación A B C D Clave: A 26 15 31 26 Esta pregunta está relacionada con análisis en representaciones gráficas cartesianas de los comportamientos de cambio de funciones y con la identificación de relaciones entre propiedades de las ecuaciones algebraicas y propiedades de las gráficas. Está referida a la capacidad del estudiante para representar y modelar usando lenguaje gráfico y algebraico. De nuevo para responder la pregunta el estudiante debía estudiar el modelo, reconocer que las variables v y c no son directamente proporcionales y desde luego que v cambia cuando t cambia. Esto permite decidir que las opciones B y C son incorrectas. A continuación bastaba analizar si la función crecía o decrecía apoyados en reconocer la imagen de cero. Como la imagen de cero es cero y a medida que t crece decrece ( se aproxima a cero), , se aproxima a 1, por tanto v tiende a mg . La opción correcta es A. c El porcentaje que seleccionó la respuesta correcta A coincide con los que seleccionaron D, es probable que los últimos identifiquen el comportamiento de un modelo exponencial pero no la condición inicial, ni el cambio. La selección de D puede provenir, además, de confundir el movimiento de la caída del objeto con la trayectoria. De todas maneras contrasta con la pregunta anterior donde el acierto fue mayor y se esperaría que los que respondieron bien a la pregunta ilustrada en el ejemplo 8 respecto a la tendencia, estuvieran en capacidad de identificar la correspondiente gráfica, sin embargo las dos preguntas son independientes.
  • 19. 19 ANÁLISIS DE RESULTADOS DE LAS PRUEBAS DE ESTADO El 31% –un porcentaje mayor que el asociado a la clave– considera que el modelo es lineal, posiblemente es la única gráfica con la que está familiarizados y no logran interpretar las condiciones. El porcentaje restante considera que la función es constante, como se comentó antes, por la alusión a las constantes en el enunciado o porque no reconocen las variables en la expresión. Contexto para los ejemplos 8 y 9 Construir espejos Para construir espejos en vidrio, una empresa diseña espejos tipo A de forma de hexágono regular, obtenidas del mayor tamaño posible a partir de láminas circula-res de vidrio de 1 metro de radio. Cortando por la mitad las piezas tipo A, se obtie-nen piezas tipo B. Ejemplo 8, núcleo común Las piezas tipo A y B se venden a $17.000 y $10.000 respectivamente. La empresa vende 5 piezas y recibe un pago por un valor total de $63.900. Si se sabe que sobre esta compra se hizo un descuento del 10% sobre el precio total de las piezas, ¿cuántas piezas se vendieron de cada tipo? A. 2 del tipo A y 3 del tipo B. B. 3 del tipo A y 2 del tipo B C. 4 del tipo A y 1 del tipo B. D. 1 del tipo A y 4 del tipo B. Componente: Numérico – Variacional Porcentaje por opciones de respuesta Competencia:Solución de problemas. A B C D Clave: B 24 43 21 11 El ítem indaga por el uso de los números reales en sus diferentes representaciones y en diversos contextos, exige dar significado a la variable e identificar diferentes métodos para solucionar sistemas de ecuaciones. Además requiere reali-zar una traducción del lenguaje natural al lenguaje simbólico formal, y desarrollar y aplicar diferentes estrategias para la solución de un problema.
  • 20. 20 ANÁLISIS DE RESULTADOS DE LAS PRUEBAS DE ESTADO El estudiante podía resolver la pregunta realizando el siguiente razonamiento, si p es el precio sin el descuento se tiene que p  0.1p  63.900, de donde p  71.000 . Si x es el número de piezas tipo A y y es el número de piezas tipo B, es posible determinar x e y resolviendo el sistema de ecuaciones lineales Sustituyendo y en la primera ecuación y reduciendo se obtiene 7.000x  21.000, de donde x = 3, y = 2 . Otra forma de encontrar la solución es ensayar con los valores dados en las opciones. Al ensayar con lo propuesto en la clave se tiene que 3 del tipo A tienen un costo de $51.000 y 2 del tipo B de $20.000, así 51.000+20.000=71.000, luego se saca el 10% al costo total que es $7.100 y restando este valor a los 71.000 se obtiene 63.900. Estrategia que resulta interesante. El 43% de los estudiantes respondieron la pregunta correctamente y el 24% posiblemente resolvieron correctamente pero se confundieron en la presentación de la respuesta (identificación de las variables), los estudiantes que seleccionaron las otras opciones posiblemente se limitan a identificar dos números que sumados den 5 (número de piezas), pero no inter-pretan las otras condiciones. Ejemplos componente Geométrico–Métrico A continuación se presenta el análisis de 5 preguntas de núcleo común y 11 de profundización; en el 2006 el énfasis en la profundización se colocó en este componente, sin embargo todas las situaciones propuestas pueden y deben ser abordadas por todos los estudiantes. Ejemplo 9, núcleo común El área que cubren 4 piezas tipo B dispuestas como lo indican la figura, es A. 3 metros cuadrados 4 B. 3 3 metros cuadrados 3 3 metros cuadrados C. 2 D. 6 3 metros cuadrados El contexto para este ejemplo se encuentra antes del ejemplo 8, en la sección de ejemplos del componente numérico-variacional.
  • 21. = , donde P es el perímetro y a el apotema, que corresponde a la altura del triángu-lo a = 1− 1 = , de donde . Como la figura 21 ANÁLISIS DE RESULTADOS DE LAS PRUEBAS DE ESTADO Componente: Geométrico – Métrico Porcentaje por opciones de respuesta Competencia:Solución de problemas. A B C D Clave: B 39 15 29 17 La pregunta indaga por el cálculo de áreas a través de la composición y descomposición de figuras exige formular un problema a partir de una representación gráfica. Para resolverlo el estudiante puede determinar el área del hexágono inscrito en un círculo de radio 1m (Pieza tipo A). Una forma es utilizar la fórmula A P a × 2 equilátero de lado uno. Se tiene que P = 6 y 3 2 4   2 2 3 3 =   A = . muestra la descomposición de dos hexágonos, entonces la respuesta sería 3 3 2     Pero, la pregunta no requería recordar la fórmula para calcular el área de un hexágono, simplemente saber determinar el área de un triángulo equilátero de lado 1, bastaría determinar la altura 3 , de donde el área sería . Con 2 12 3  . doce triángulos se recubren las cuatro piezas tipo B, por tanto el área es 3 3 4 También se puede resolver el problema hallando el área del paralelogramo, para lo cual se debe asociar la apotema del hexágono con la altura del paralelogramo, y de esta manera se obtiene que la base es 6 y la altura 3 , luego 2 3 3 A  b h  6 3  2 El porcentaje más alto de estudiantes seleccionó la opción A, es muy posible que no hayan realizado ningún tipo de cálculo y simplemente porque observaron un cuatro en el denominador, lo asociaron a 4 piezas, pero desconocen los procedi-mientos antes mencionados, no hay siquiera un reconocimiento visual de las relaciones. La selección de C, superior al 25% pudo originarse en un cálculo incorrecto o en la utilización de una fórmula no pertinente. La opción D es escogida por un porcentaje superior al asociado a la clave, aparece allí un seis y éste podría asociar a los seis lados del hexágono. En estos casos, las opciones de respuesta provenían de cálculos incorrectos pero posiblemente el estudiante no realiza éstos sino que se orienta por alguna información parcial (número de lados por ejemplo). Se esperaba que la pregunta resultara de nivel de dificultad medio, sin embargo es posible que por la aparición de números irracionales en la operatoria o por el desconocimiento de procedimientos para determinar el área de figuras planas, tan sólo la haya contestado el 15%.
  • 22. 22 ANÁLISIS DE RESULTADOS DE LAS PRUEBAS DE ESTADO Contexto para los ejemplos 10 y 11 Diseño de un parque En un lote de forma rectangular cuyos lados miden 80 y 60 metros, se va a cons-truir un parque. La figura muestra el plano del parque. Los puntos B, D, F y G son los puntos medios de los lados del rectángulo ACEH, K es un punto de AE tal que CK es perpendicular a AE . Ejemplo 10, núcleo común La longitud de AE es A. 100 metros B. 140 metros. C. 2 7 metros D. 2 35 metros. Componente: Geométrico – Métrico Porcentaje por opciones de respuesta Competencia: Solución de problemas A B C D Clave: A 31 54 8 7 El ítem indaga por la relación existente entre las medidas de los lados de un triángulo rectángulo, por la aplicación del teorema de Pitágoras.
  • 23. 23 ANÁLISIS DE RESULTADOS DE LAS PRUEBAS DE ESTADO Para dar solución a este problema se requiere que el estudiante identifique la longitud del camino AE como la hipotenusa de un triángulo rectángulo cuyos catetos miden 80 y 60 metros y posteriormente aplique el Teorema de Pitágoras: Por lo tanto la opción correcta es A. La selección de la opción B, como respuesta correcta, hecha por el 54% de los estu-diantes puede indicar que no identifican el teorema de Pitágoras como la estrategia a utilizar para dar solución al problema o que hacen una mala aplicación de este puesto que la opción resulta de sumar las medidas de los catetos. La selección de las opciones C, o, D puede indicar una equivocada aplicación del teorema, confundiendo los cuadrados de las medidas de los catetos con el doble: en el caso de la C ,o, hallando la raíz de la suma de las medidas y no de los cuadrados de estas en el caso de la D. Nótese que a pesar ser una aplicación directa del teorema, sin dificultad en la parte operatoria, solamente el 31% de los estudiantes llegó la respuesta correcta. Esto puede indicar que en las aulas el trabajo con el teorema de Pitágoras se limita a su aplicación mecánica y rutinaria, y que no se está abordando su aplicación en otro tipo de problemas como el que se presenta, en los cuales no se induce a la utilización del teorema ya que no se explicita la existencia de un triángulo rectángulo y no se utilizan los términos asociados a este (hipotenusa y catetos). Es importante trabajar otro tipo de aplicaciones. Ejemplo 11, núcleo común El área de la zona cubierta de pasto es A. 1.800 metros cuadrados. B. 2.400 metros cuadrados. C. 3.600 metros cuadrados. D. 4.800 metros cuadrados . Componente: Geométrico – Métrico Porcentaje por opciones de respuesta Competencia: Razonamiento A B C D Clave: A 43 26 16 15 El ítem indaga por la comprensión y aplicación de conceptos básicos de proporcionalidad y por el uso de argumentos geométricos para formular problemas en contextos matemáticos, se explora además la competencia del estudiante para explicar usando hechos y propiedades. Para determinar el área de la zona cubierta de pasto, basta hallar la diferencia entre las áreas de los triángulos AEH y GFH, esto es También es posible determinar la solución, hallando el área de trapecio y usando semejanza. El 43% de los estudiantes respondió correctamente la pregunta y un 26% se limitó a determinar el área de uno de los triángulos y seleccionó B. Porcentajes próximos seleccionaron C o D, sumando las áreas en lugar de restar o determinando el área de un rectángulo.
  • 24. ANÁLISIS DE RESULTADOS DE LAS PRUEBAS DE ESTADO La pregunta sería pertinente para los primeros grados de la básica secundaria e incluso para la primaria y sin embargo presenta dificultad para más de la mitad de la población que presentó la prueba. Ejemplo 12, núcleo común Triángulos Los polígonos se clasifican de acuerdo a sus propiedades y relaciones: medidas de los lados, medidas de los ángulos, relaciones entre sus lados, etc. Los triángulos se clasifican de acuerdo a las medidas de sus lados en isósceles, equiláteros y escálenos. Un triángulo con dos lados congruentes se llama isósce-les, con tres lados congruentes se llama equilátero. Un triángulo escaleno es aquel en el cual todos sus lados tienen diferente medida. De acuerdo a la clasificación de los triángulos, NO es correcto afirmar que A. si un triángulo es equilátero es isósceles. B. si un triángulo no es escaleno es equilátero. C. existen triángulos rectángulos que son isósceles. D. existen triángulos isósceles que no son equiláteros. Componente: Geométrico - Métrico Porcentaje por opciones de respuesta Competencia: Razonamiento A B C D Clave: B 41 21 22 15 La pregunta indaga por la clasificación de triángulos de acuerdo a sus propiedades, exige interpretar criterios presentados explícitamente en el contexto de la prueba, está relacionada con la generalización de propiedades y relaciones e indaga por la interpretación de definiciones y el análisis de proposiciones cuantificadas. Para contestar correctamente, el estudiante debía retomar las definiciones presentadas inicialmente e interpretar la negación. Es correcto afirmar que si es equilátero, (tiene los tres lados congruentes) tiene dos lados congruentes, es decir, es isósceles por lo tanto la opción A no es falsa. También es correcto afirmar que existen triángulos rectángulos que son isósceles, por ejemplo todo triángulo con ángulos de 45°, 45° y 90° así la opción C también es correcta. Es correcto además que existen triángulos isósceles que no son equiláteros, por ejemplo un triángulo rectángulos con lados 1, 1 y 2 , así la opción D es correcta. Pero es incorrecto afirmar que si un triángulo no es escaleno es equilátero, puede ser isósceles o equilátero, aunque la pregunta es fácil, excepto posiblemente por la negación, tan sólo la respondió correctamente el 21% Es posible que un porcentaje alto de los estudiantes haya ignorado la negación y se limite a seleccionar opciones diferentes a la clave por considerarlas correctas, lo que atrajo especialmente hacia la A, que muy seguramente es más familiar en el aula. Sería pertinente que particularmente en geometría se analicen enunciados diversos donde se expresen condiciones, negaciones y se trabaje con proposiciones compuestas; es importante destacar que las herramientas de la lógica formal se construyen en cada uno de los pensamientos no esquemáticamente en apartes dispersos del currículo. 24
  • 25. 25 ANÁLISIS DE RESULTADOS DE LAS PRUEBAS DE ESTADO Contexto para ejemplo 13. La parábola Una parábola es el lugar geométrico de todos los puntos del plano que equidistan de un punto fijo llamado foco y una recta fija llamada directriz. En el siguiente cuadro se muestran ecuaciones y gráficas que corresponden a parábolas con el vértice, el foco y la directriz ubicados en diferentes puntos del plano Ejemplo 13, núcleo común La gráfica de la parábola con foco en el punto (6,4) y directriz que pasa por el punto (0,-2) se presenta en,
  • 26. 26 ANÁLISIS DE RESULTADOS DE LAS PRUEBAS DE ESTADO Componente: Geométrico – Métrico Porcentaje por opciones de respuesta Competencia: Comunicación A B C D Clave: A 36 28 15 21 Esta pregunta hace referencia a la identificación de las características de localización de objetos geométricos en sistemas de representación cartesiana y al uso de las propiedades de las cónicas, particularmente de la parábola. Exige interpretar, representar y usar diferentes tipos de lenguaje para describir relaciones. Para resolver la pregunta el estudiante debía simplemente interpretar la información que se presenta en los recuadros iniciales, pasando por la definición de la parábola y la identificación de sus elementos, si interpreta esta caracterización puede descartar fácilmente las opciones diferentes a la clave pues, o bien los puntos de la curva no equidistan del foco y la directriz, o la recta señalada no es la directriz cuando la parábola es abierta hacía la izquierda o la derecha, en este sentido puede afirmarse que el ítem acude esencialmente a la visualización. El 36% de los estudiantes respondieron correctamente la pregunta, se esperaba que este porcentaje fuera más alto pues no requiere mayor elaboración. Porcentajes cercanos optaron por B o D posiblemente sin análisis de condiciones, sim-plemente por ubicación de la recta supuesta como directriz, si hubieran elegido un punto de estas parábolas las hubieran descartado si comprenden la definición. Obsérvese que en este contexto se plantean preguntas que indagan por la noción puramente matemática de la parábola. Es importante recuperar en el aula de clase las definiciones matemáticas y trabajar estos conceptos no sólo en aplicaciones cotidianas o de otras áreas sino también en aplicaciones al interior de la matemática misma. Contexto para los ejemplos 14 y 15 En un triángulo ABC como el que muestra la figura, a, b y c corresponden a las longitudes de sus lados. Los siguientes teoremas relacionan lados y ángulos de un triángulo ABC cualquiera. Teorema del Seno Teorema del coseno SenC c SenA  SenB  b a 2 2 2 a = b + c − 2 bcCosA 2 2 2 b = a + c − 2 acCosB c a b 2 abCosC 2 2 2 = + −
  • 27. 27 ANÁLISIS DE RESULTADOS DE LAS PRUEBAS DE ESTADO Ejemplo 14, profundización En el triángulo que muestra la figura los valores de b y Sen α son A. b = 7 y Sen B. b = 7 y Sen C. b = 5 y Sen D. b = 5 y Sen Componente: Geométrico – Métrico Porcentaje por opciones de respuesta Competencia: Razonamiento A B C D Clave: A 35 19 34 12 Se indaga en esta pregunta por la aplicación de los teoremas del seno y del coseno, está relacionado con el uso de argu-mentos geométricos, propiedades y relaciones para resolver y formular problemas. Se indaga además por la justificación de estrategias y procedimientos en el tratamiento de situaciones problema. El estudiante debe identificar la información presentada en la figura y aplicar en primer lugar el teorema del coseno Luego debe aplicar el teorema del seno de donde .
  • 28. senA = senB = senA = senA = 2 28 ANÁLISIS DE RESULTADOS DE LAS PRUEBAS DE ESTADO El 35% resolvió correctamente la pregunta pero un porcentaje similar seleccionó la opción C, se limitan a copiar un dato de la información inicial o aplican incorrectamente las relaciones al triángulo dado. El 19% selecciona B, es posible que haya aplicado correctamente el teorema del coseno, pero intercambia información al aplicar el segundo teorema. La selección de D puede provenir de una selección al azar o de composición arbitraria de los números dados sin observar condiciones. Es posible que este ítem resulte complejo debido a que el estudiante debe aplicar sucesivamente ambos teoremas, sin embargo este tipo de problemas se supone son ampliamente trabajados durante el grado décimo, además el estudiante no requiere saber los teoremas de memoria pues son presentados dentro del contexto, por lo cual se esperaría que un porcentaje más alto de la población lo resolviera correctamente, más aún al estar en la sesión de profundización y no de núcleo común, lo cual resultaría pertinente. Invitamos a los maestros y maestras a resolver este tipo de problemas con todos sus estudiantes e identificar las razones por las cuales no logran resolverlo apropiadamente y así poder reforzar estos aspectos. Ejemplo 15, profundización Si en un triángulo ABC se cumple que SenA = SenB = 2SenC, entonces el perí-metro del triángulo es A. 3b B. 5c C. 2a + 2c D. a + b + 2c Componente: Geométrico – Métrico Porcentaje por opciones de respuesta Competencia: Razonamiento A B C D Clave: B 7 10 14 69 En esta pregunta se indaga por la aplicación del teorema del seno para el cálculo del perímetro de un triángulo, está rela-cionada con el reconocimiento y contraste de propiedades y relaciones geométricas. Para resolver el problema se retoman relaciones del teorema del seno, senC c b a usando las condiciones dadas en la pregunta se tiene que senA c b a de donde, b = a y 2c = a , y por lo tanto a = b = 2c El perímetro del triángulo será entonces P = a + b + c = 2c + 2c + c = 5c La mayoría de los estudiantes que presentaron la prueba no lograron interpretar la pregunta y seleccionaron la opción D
  • 29. 29 ANÁLISIS DE RESULTADOS DE LAS PRUEBAS DE ESTADO (69%) posiblemente lo asumieron por la razón entre los senos, 1:1:2 se limitaron a copiar los números o es posible que ni siquiera recuerden la expresión para el perímetro y opten por una que se parece aunque no tenga sentido. Porcentajes bajos seleccionaron las otras opciones, por azar o aplicación de transformaciones incorrectas. Nuevamente llama la atención que siendo una pregunta de profundización y que sólo requiere una aplicación del teorema del seno para encontrar el perímetro, haya sido contestada por un porcentaje tan bajo de la población, esto posiblemente también muestra que cuando se aborda este tipo de aplicación se hace de manera rutinaria, generalmente se pide al estudiante determinar todos los valores del triángulo: medida de los lados y de los ángulos, de manera numérica y no a través del uso de relaciones entre los mismos. Es necesario proponer más y variados ejercicios de aplicación de este tipo de conceptos. Contexto para los ejemplos 16 y 17 Mosaicos En la ilustración se observan algunos mosaicos formados por polígonos regulares. En cada mosaico los lados de los polígonos que se utilizan deben tener la misma medida. Ejemplo 16, profundización En el mosaico que se muestra, la medida del ángulo α es A. 60° B. 90° C. 120° D. 150° Componente: Geométrico – Métrico Porcentaje por opciones de respuesta Competencia: Comunicación A B C D Clave: C 24 15 55 6
  • 30. ANÁLISIS DE RESULTADOS DE LAS PRUEBAS DE ESTADO El ítem explora por la capacidad de estudiante para interpretar información presentada en diferentes tipos de represen-tación y para construir argumentaciones que den solución a situaciones diversas, está relacionado con la aplicación de propiedades de figuras planas. El estudiante debe averiguar la medida de uno de los ángulos que forman el vértice de un mosaico al cual concurren dos triángulos equiláteros y dos hexágonos regulares; para dar solución a la pregunta, debe observar que la suma de los ángulos que concurren a un vértice es 360º, y plantear las relaciones que le permitan hallar la medida del ángulo solicitado: • los ángulos interiores de un triángulo equilátero son congruentes, como la suma de los ángulos interiores de todo 30 triángulo es 180º, entonces cada ángulo mide 60º • de donde La pregunta también puede resolverse si se averigua directamente la medida del ángulo interior de un hexágono regular, dividiéndolo en triángulos: Como se forman 4 triángulos entonces la suma de los ángulos interiores de un hexágono debe ser 180°× 4 = 720° , por ser hexágono regular los seis ángulos tienen la misma medida de donde y por lo tanto El 55% de los estudiantes seleccionó C, la opción correcta, aquellos que seleccionaron A, el 24%, o B, el 15%, no enten-dieron las condiciones del problema ni observaron la ilustración, pues evidentemente el ángulo α es obtuso por lo tanto no puede medir ni 60º ni 90º. Teniendo en cuenta que este no es un contexto común y seguramente resultó novedoso para muchos de los estudiantes, el porcentaje que contestó correctamente es bastante alto, lo cual muestra un dominio importante en propiedades de las figuras geométricas en el grupo que escogió profundizar en matemática. Ejemplo 17, profundización NO es posible construir un mosaico si a un mismo vértice concurren A. 2 octágonos y 1 cuadrado. B. 2 octágonos y 2 cuadrados. C. 1 hexágono regular y 4 triángulos equiláteros. D. 2 hexágonos regulares y 2 triángulos equiláteros. Componente: Geométrico – Métrico Porcentaje por opciones de respuesta Competencia: Razonamiento A B C D Clave: B 21 34 27 16 La pregunta explora la capacidad del estudiante para dar cuenta del cómo y del porqué de los caminos que se siguen para
  • 31. 180º (8 − 2) como son dos se tiene que los ángulos de los 180º (6 − 2) más 240° por los cuatro ángulos de los triángulos 31 ANÁLISIS DE RESULTADOS DE LAS PRUEBAS DE ESTADO llegar a conclusiones; está asociada la aplicación de estrategias que involucran aspectos de medición. Para dar solución a la pregunta el estudiante puede aplicar la fórmula, que se presenta en el recuadro, en cada una de las opciones hasta encontrar aquella en la cual la suma de la medida de los ángulos que concurren al vértice NO sea 360º, concluyendo de esta manera que la opción correcta es B pues: 135 • Opción A. Cada ángulo del octágono mide = ° 8 octágonos suman 270° , más 90° del cuadrado. Por lo tanto si se puede construir. • Opción B. Basta con sumar 90° a la cuenta anterior así se deduce inmediatamente que no puede construirse. 120 • Opción C. Cada ángulo del hexágono mide = ° 6 equiláteros. Por lo tanto si se puede construir. • Opción D. Los ángulos de los dos hexágonos mines 240° más 120° de los dos ángulos de los triángulos equiláteros. El 34% de los estudiantes seleccionó la opción correcta, la selección de otras opciones indica que los estudiantes no en-tendieron las condiciones que deben cumplir los ángulos de polígonos regulares que concurren al vértice de un mosaico, o se equivocaron en los reemplazos requeridos para la correcta aplicación de la fórmula. Contexto para los ejemplos 18 y 19 La siguiente figura muestra una maqueta para una construcción. La maqueta está formada por un paralelepípedo y una pirámide de base cuadrada de 20cm de lado. Las caras laterales de la pirámide son triángulos equiláteros.
  • 32. 32 ANÁLISIS DE RESULTADOS DE LAS PRUEBAS DE ESTADO Ejemplo 18, profundización La altura total de la maqueta A. está entre 10cm y 20cm. B. está entre 20cm y 25cm. C. está entre 25cm y 35cm. D. está entre 35cm y 40 cm. Componente: Geométrico – Métrico Porcentaje por opciones de respuesta Competencia: Solución de problemas A B C D Clave: C 14 19 35 31 La pregunta indaga por la altura de la pirámide, para lo cual se requiere la aplicación de propiedades de la pirámide, de triángulos equiláteros y aplicación del teorema de Pitágoras y pasar de una representación tridimensional a una bidimen-sional; aborda aspectos de referidos a la solución de problemas en los cuales se usan propiedades geométricas y explora por la capacidad de los estudiantes en el planteo y aplicación de estrategias para dar solución a diferentes situaciones. Para resolver la pregunta el estudiante debe averiguar la altura total de la maqueta sumando a la altura del paralelepípedo (15cm) la altura de la pirámide cuadrangular que está construida sobre él. Para hallar la altura de esta pirámide es necesario hallar, inicialmente, la altura de los triángulos equiláteros que forman las caras laterales Aplicando el teorema de Pitágoras se tiene que Conociendo esta altura, es posible aplicar nuevamente el teorema de Pitágoras para hallar la altura de la pirámide, consi-derando que por ser una pirámide regular el centro de la base coincide con el pie de la altura:
  • 33. 33 ANÁLISIS DE RESULTADOS DE LAS PRUEBAS DE ESTADO Como el estudiante no dispone de calculadora para realizar esta operación, debe encontrar un rango del valor de esta altura considerando que la raíz de 200 es mayor que 14 (142 =196) y menor que 15 (152=225) Por lo tanto la altura total de la pirámide está entre 29cm y 30cm. El 35% de los estudiantes seleccionó C, la opción correcta, Los estudiantes que seleccionaron la opción D, el 31% , posible-mente no entendieron las condiciones del problema o tuvieron problemas operatorios. El 14% y el 19% que seleccionaron las opciones A y B respectivamente, posiblemente resolvieron el problema de manera parcial, seleccionando como respuesta las alturas del triángulo o de la pirámide sin tener en cuenta la del paralelepípedo. Ejemplo 19, profundización La base del paralelepípedo se va a recubrir con láminas de forma rectangular de lados 4cm y 1 cm. El mínimo número de láminas que se necesitan es A. 16 B. 25 C. 75 D. 100 Componente: Geométrico – Métrico Porcentaje por opciones de respuesta Competencia: Solución de problemas A B C D Clave: D 24 26 25 24 Este ítem explora la capacidad del estudiante para diseñar e implementar estrategias que den solución a un problema, está relacionado con el cálculo de área por recubrimiento. Para averiguar el número mínimo de láminas de forma rectangular de lados 4cm y 1cm que se necesitan para recubrir un cuadrado de lado 20, el estudiante, puede determinar que es posible acomodar sobre un lado del cuadrado 5 láminas (por el lado de 4cm) y sobre el otro lado 20 láminas (por el lado de 1cm), por lo tanto se necesitan 100 láminas. También es posible dar solución a la pregunta considerando que puesto que el número de láminas que se pueden acomodar sobre los lados del cuadrado es exacto, ya que 20 es múltiplo de 4 y de 1 puede dividirse el área del cuadrado (400 cm2) entre el área de la lámina rectangular (4 cm2) obteniendo de esta manera que son necesarias 100 láminas. A pesar de ser una pregunta, sin dificultad específica aparente, solamente el 24% de los estudiantes seleccionó D, los estudiantes que seleccionaron A o B, el 24% y 26% consideraron que se podían acomodar, sobre los lados del cuadrado, 4 y 5 láminas respectivamente.
  • 34. 34 ANÁLISIS DE RESULTADOS DE LAS PRUEBAS DE ESTADO Contexto para los ejemplos 20, 21 y 22 Triángulos semejantes Dos triángulos ABC y A’B’C’ son semejantes si se cumple uno cualquiera de los siguientes criterios: 1. Los ángulos correspondientes son congruente, es decir . ∠A ≅∠A’, ∠B ≅∠B’, ∠C ≅∠C’ 2. Dos pares de lados correspondientes son proporcionales y los ángulos comprendidos son congruentes, es decir y ∠A ≅∠A’ y ∠B ≅B’ y ∠C ≅∠C’ 3. Lados correspondientes son proporcionales, es decir
  • 35. 35 ANÁLISIS DE RESULTADOS DE LAS PRUEBAS DE ESTADO Ejemplo 20, profundización En cada figura se muestra un par de triángulos De los pares de triángulos, son semejantes, los mostrados en las figuras A. 1 y 2 B. 2 y 4 C. 1 y 3 D. 3 y 4 Componente: Geométrico – Métrico Porcentaje por opciones de respuesta Competencia: Comunicación A B C D Clave: B 21 28 16 34 Está relacionado este ítem con la resolución y formulación de problemas que involucran relaciones de semejanza usando representaciones visuales. Para resolver el problema el estudiante debía determinar simplemente las razones entre los lados correspondientes de los triángulos presentados, si están en la misma razón serán semejantes. • En la figura 1, , por tanto los triángulos no son semejantes. • En la figura 2, los triángulos tienen dos pares de lados proporcionales (razón 3 ) y un par de ángulos opuestos por 5 el vértice congruentes por tanto los triángulos son semejantes. • En la figura 3, la razón entre los lados no es constante ( ) los triángulos no son semejantes. • En la figura 4, , pares de lados correspondientes son proporcionales, los triángulos son semejan-tes. Así la respuesta correcta es B, son semejantes los pares de triángulos mostrados en las figuras 2 y 4. El 28% respondió correctamente la pregunta un porcentaje muy bajo para el nivel de dificultad y teniendo en cuenta que es una pregunta de profundización, posiblemente la noción de semejanza está en el nivel de “tener la misma forma” y se asumen criterios puramente visuales, desconocen posiblemente los criterios que permiten caracterizar la relación. Nótese que no se requería para la solución la memorización de las condiciones pues justamente era lo que presentaba el
  • 36. ANÁLISIS DE RESULTADOS DE LAS PRUEBAS DE ESTADO contexto, por eso se insiste en que el porcentaje de estudiantes que contestan este tipo de preguntas debería ser mayor. La opción D, atrajo más que la clave, posiblemente por una consideración de razones incompletas de aparentes lados correspondientes. ( ). Es recomendable que los maestros y maestras retomen este tipo de ejercicios y los realicen con todos los estudiantes, la noción de semejanza aunque es un tema que generalmente aparece en el currículo de grado noveno, es ampliamente utilizada para la solución de diferentes tipos de problemas en los grados posteriores, es uno de los conceptos fundamentales que cualquier estudiantes al terminar sus estudios debe poder utilizar. Ejemplo 21, profundización Los triángulos ABC y A’B’C’ son semejantes Las medidas de los lados x y y son respectivamente Componente: Geométrico – Métrico Porcentaje por opciones de respuesta Competencia: Razonamiento A B C D Clave: A 25 31 30 13 El ítem está relacionado con la aplicación de los criterios de semejanza, requiere establecer relaciones y utilizar transfor-maciones. Como los triángulos ABC y A´B´C´ son semejantes para resolver el problema el estudiante debía plantear la razón entre los lados correspondientes es decir , de donde y . Por lo tanto la respuesta correcta es A. Solamente un 25% de los estudiantes respondió correctamente la pregunta, sin embargo un 31% respondieron B, posible-mente confundieron las variables, se podría afirmar en consecuencia que el 56% realizó procedimientos correctos. 36
  • 37. 37 ANÁLISIS DE RESULTADOS DE LAS PRUEBAS DE ESTADO Los estudiantes que seleccionaron C y D muy seguramente no identificaron correctamente los lados proporcionales, sin observar nominación de los vértices, plantearon la razón comparando lados y B´C´, por ejemplo, y determinaron el valor , seleccionando las opciones que lo contenían. Faltaría en ese caso un análisis cuidadoso de las condiciones planteadas inicialmente. Ejemplo 22, profundización Sea ABC un triángulo, D un punto de y E un punto de , como se muestra en la figura A. ∠AED ≅ ∠ABC . B. AB ≅ BC y AD = DE. C. el triángulo ADE es semejante al triángulo ABC. D. el ángulo ACB es congruente con el ángulo BAC. Componente: Geométrico – Métrico Porcentaje por opciones de respuesta Competencia: Razonamiento. A B C D Clave: C 15 23 47 14 Este ítem indaga por el reconocimiento y contrastación de propiedades y relaciones geométricas en la demostración de teoremas básicos. Pretende analizar si el estudiante reconoce que es una prueba en matemáticas y como se diferencia de otros tipos de razonamiento, distinguiendo y evaluando cadenas de argumentos. Los ángulos ∠AED y ∠ACB son congruentes por ser correspondientes entre las paralelas y . El ∠CAB es común a los triángulos ADE y ABC , por tanto estos dos triángulos tienen sus tres ángulos congruentes, por criterio Angulo-Angulo-Angulo se puede concluir que los dos triángulos son semejantes. Por ser semejantes tienen lados corres-pondientes proporcionales de donde . La opción correcta es entonces C. Las otras opciones no se pueden deducir con las condiciones dadas en el problema y así se dieran, no implican la proporcionalidad de los lados. El 47% de los estudiantes seleccionó la opción correcta, es un porcentaje importante, en algunos casos es posible que hayan realizado el razonamiento correcto, pero también existe la posibilidad de que por un criterio visual, hayan reconocido
  • 38. ANÁLISIS DE RESULTADOS DE LAS PRUEBAS DE ESTADO los dos triángulos como semejantes. Solamente en una prueba abierta que es pertinente para el aula, el profesor podría observar si el estudiante realiza un razonamiento adecuado. El resto del grupo se distribuyó con porcentajes relativamente similares, más alto para la opción B que, de nuevo, observando la figura podría asumir incorrectamente que los lados tienen la misma medida. Ejemplo 23, profundización Trayectoria de un barco La hipérbola es el lugar geométrico de los puntos del plano cuya diferencia de distancias a dos puntos fijos llamados focos es constante. En el plano cartesiano se muestra la trayectoria hiperbólica que describe un barco, dos radares A y B, ubicados en los focos de la trayectoria y un puesto de control 0. Desde cualquier punto P de la trayectoria el barco envía señales a los radares ubicados en los puntos A y B a distancias d1 y d2 respectivamente. Las señales se desplazan a una velocidad constante. Teniendo en cuenta que la trayectoria que describe el barco es hiperbólica se debe cumplir que A. d1 + d2 es constante B. d1 – d2 es constante C. d2 es constante D. d1 es constante Componente: Geométrico – Métrico Porcentaje por opciones de respuesta Competencia: Razonamiento. A B C D Clave: B 44 27 15 13 Está relacionado el ítem con la identificación de características de localización de las cónicas en un sistema de representación cartesiana. Indaga por la generalización de propiedades y relaciones y la capacidad de expresarlas matemáticamente. Para responder la pregunta el estudiante simplemente debe traducir la definición “La hipérbola es el lugar geométrico de los puntos del plano cuya diferencia de distancias a dos puntos fijos llamados focos es constante”. Si la distancia 38
  • 39. 39 ANÁLISIS DE RESULTADOS DE LAS PRUEBAS DE ESTADO del punto P a un foco es d1 y la distancia de P al otro foco es d2 , 1 2 d − d es una constante. Por tanto la clave es B. El porcentaje más alto de estudiantes seleccionó la opción A (43%), es posible que recuerden la definición analítica de la elipse, confunden las dos curvas y seleccionan por ello esta opción, o también es posible que no interpreten la palabra “diferencia” y opten por la expresión más familiar, la suma. El 27% logran realizar una traducción correcta y porcentajes similares posiblemente sin interpretar condiciones seleccionan las otras opciones. Al igual que el contexto de la parábola este debe resultar poco familiar para los estudiantes, por eso este sólo se incluyó en profundización, sin embargo se supone que esta cónica es estudiada junto con las demás en grado décimo y por lo tanto un conocimiento inicial como la definición debería resultar familiar y fácil para los estudiantes. Nótese que no se indagaba por la memorización de hipérbola, bastaba con leer el contexto y traducir matemáticamente la descripción. Contexto para los ejemplos 24 y 25 Otra mirada al teorema de Pitágoras Los triángulos sombreados que aparecen en cada figura son rectángulos. So-bre los lados de cada triángulo se han construido figuras planas semejantes.
  • 40. 40 ANÁLISIS DE RESULTADOS DE LAS PRUEBAS DE ESTADO Ejemplo 24, profundización Si el área del cuadrado 1 es la mitad del área del cuadrado 2, entonces el área del cuadrado 3 es A. la mitad del área del cuadrado 2. B. el doble del área del cuadrado 2. C. el triple del área del cuadrado 1. D. la tercera parte del área del cuadrado 1 Componente: Geométrico – Métrico Porcentaje por opciones de respuesta Competencia: Razonamiento. A B C D Clave: C 9 26 51 12 Como se indica en el nombre del contexto, en los ejemplos 25 y 26 se indaga por la aplicación del teorema de Pitágoras, mediante el establecimiento de relaciones geométricas. Para dar solución a la pregunta el estudiante debe, inicialmente, identificar los catetos y la hipotenusa del triángulo rectán-gulo, que no está en posición canónica, sobre cuyos lados se han construido cuadrados, para aplicar la relación existente entre las áreas de estos cuadrados teniendo en cuenta las condiciones del enunciado. Teniendo en cuenta la ilustración y aplicando el teorema de Pitágoras se tiene que: Área del cuadrado construido sobre la hipotenusa = suma de las áreas de los cuadrados construidos sobre los catetos, por lo tanto Área del cuadrado 3 = Área del cuadrado 1 + Área del cuadrado2 Como el área del cuadrado 1 es la mitad del área del cuadrado 2, o lo que es lo mismo, el área del cuadrado 2 es el doble del área del cuadrado 1, entonces Área del cuadrado 3 = Área del cuadrado 1 + 2 x Área del cuadrado1 = 3 x área del cuadrado 1 Determinando, de esta manera, que la respuesta correcta es C. El 51% de los estudiantes seleccionó la opción correcta, lo cual permite afirmar que identifican los catetos y la hipotenusa de triángulos rectángulos dibujados en diferentes posiciones y que aplican correctamente el teorema de Pitágoras. Los estudiantes que seleccionaron la opción B, el 26%, posiblemente utilizaron, de manera equivocada, la información del enunciado acerca de la relación entre las áreas de los cuadrados 1 y 2 y finalmente, los estudiantes que seleccionaron las opciones A , el 9%, y D, el 12% no entendieron las condiciones de problema ni utilizaron estrategias que les permitieran establecer relaciones entre las áreas de los cuadrados. La ilustración de esta pregunta es utilizada para demostrar el Teorema de Pitágoras, es decir, debe ser familiar para los estudiantes y aunque el porcentaje que contestó correctamente es alto, nuevamente se llama la atención pues es una pregunta que corresponde a la profundización, por lo tanto debería existir una mayor apropiación de las relaciones que existen entre las áreas de los cuadrados que se construyen sobre los lados y la hipotenusa de un triángulo rectángulo, es decir, está es la aplicación más conocida del Teorema de Pitágoras.
  • 41. 41 ANÁLISIS DE RESULTADOS DE LAS PRUEBAS DE ESTADO Ejemplo 25, profundización Los radios de las circunferencias en las cuales se pueden inscribir los hexágo-nos 1 y 2 son 6cm y 8cm respectivamente. El perímetro y el área del triángulo rectángulo son A. 12cm y 6cm2. B. 12cm y 24cm2. C. 24cm y 48cm2. D. 24cm y 24cm2. Componente: Geométrico – Métrico Porcentaje por opciones de respuesta Competencia: Razonamiento. A B C D Clave: D 19 36 30 13 La pregunta indaga por el perímetro y el área de un triángulo rectángulo, cuando se conocen las medidas de los radios de las circunferencias en las cuales se pueden inscribir los hexágonos regulares que aparecen construidos sobre los lados del triángulo rectángulo. Está relacionada con estándares referidos a la interpretación y aplicación de propiedades y relaciones geométricas de figuras planas; explora por la capacidad del estudiante para diseñar procedimientos y estrategias aplicando propiedades y relaciones geométricas que permitan solucionar situaciones planteadas en diferentes contextos. Para dar respuesta al problema, el estudiante debe deducir, inicialmente, que como el radio de una circunferencia en la cual se ha inscrito un hexágono es congruente con el lado de éste, los catetos del triángulo rectángulo miden 6cm y 8cm. Una vez conocidas las medidas de los catetos puede aplicar el teorema de Pitágoras para encontrar que la medida la hipotenusa, es de 10 cm completando de esta manera la información necesaria para hallar el perímetro y el área: Perímetro: ; Área: El 13% de los estudiantes utilizó correctamente la relación de congruencia entre el radio de la circunferencia y el lado del hexágono inscrito, utilizó el teorema de Pitágoras y aplicó el concepto de altura de un triángulo para hallar la opción correcta D. Los estudiantes que escogieron C identificaron las condiciones del problema pero no calcularon de manera correcta el área del triángulo. Los estudiantes que seleccionaron las otras opciones posiblemente se limitaron a realizar operaciones con la información numérica que aparece en el enunciado.
  • 42. ANÁLISIS DE RESULTADOS DE LAS PRUEBAS DE ESTADO 3.3 Ejemplos componente Aleatorio Se presenta a continuación el análisis de 6 ejemplos correspondientes a núcleo común. Contexto para los ejemplos 26 y 27 Salarios Los salarios mensuales de los 25 empleados de una empresa están distribuidos de la siguiente manera • 21 empleados ganan 1 salario mínimo mensual • 2 empleados ganan 10 salarios mínimos mensuales • 1 empleado gana 14 salarios mínimos mensuales • 1 empleado gana 25 salarios mínimos mensuales 42 Ejemplo 26, núcleo común El valor que mejor representa el conjunto de datos sobre el salario mensual del grupo de empleados es A. 1 salario mínimo mensual. B. 10 salarios mínimos mensuales. C. 14 salarios mínimos mensuales. D. 25 salarios mínimos mensuales. Componente: Aleatorio Porcentaje por opciones de respuesta Competencia: Razonamiento A B C D Clave: A 50 9 6 34 La pregunta indaga por la comprensión del significado de las medidas de tendencia central, en este caso específico por la interpretación de la moda como el dato más representativo de una información numérica. Para dar solución a la pregunta el estudiante debe deducir, de la información que se presenta en el recuadro, que de un total de 25 empleados 21 ganan 1 salario mínimo, es decir que 1 salario mínimo mensual es el valor que con más frecuencia se presenta en el conjunto de datos. Luego el dato que mejor describe el conjunto es el de 1 salario mínimo mensual, por lo tanto la opción correcta es A, que fue seleccionada por la mitad de la población. Quienes escogen las otras opciones pueden hacerlo por una selección al azar o por análisis erróneos. Esta pregunta es bastante sencilla y aunque la contesto la mitad de la población, llama la atención el porcentaje tan alto que escogió como opción correcta la D, es probable que simplemente sea porque la empresa tiene 25 empleados. Es importante que los maestras y maestras al enseñar los conceptos de promedio, media, media y moda, además de ejercitar el cálculo de estos, se trabaje en su significado y cuando un concepto u otro representa mejor un conjunto de datos, siempre se cree que es el promedio, en este caso esa opción no se colocó para no inducir a los estudiantes a ese error.
  • 43. 43 ANÁLISIS DE RESULTADOS DE LAS PRUEBAS DE ESTADO Ejemplo 27, núcleo común En el departamento de producción de la empresa trabajan 4 mujeres y 6 hombres. La edad promedio de las mujeres es 30 años y la de los hombres es 40. La edad promedio de los trabajadores del departamento de producción es A. 30 años. B. 35 años. C. 36 años. D. 40 años. Componente: Aleatorio Porcentaje por opciones de respuesta Competencia: Solución de problemas A B C D Clave: C 8 43 27 21 La pregunta está orientada a explorar la capacidad del estudiante para aplicar diferentes estrategias en la solución de problemas relacionados con conceptos básicos asociados con medidas de tendencia central, en este caso se indaga por la correcta utilización del concepto de promedio ponderado. Para dar solución al problema, es necesario hallar la suma de las edades, en años, de todos los empleados teniendo en cuenta el número de mujeres y de hombres que trabajan en el departamento de producción: en el caso de las mujeres puesto que son 4 y su edad promedio es 30, el número total de años se obtiene efectuando 4 x 30 = 120 años y de manera análoga, para los hombres multiplicando 6 x 40 = 240 años. La suma de este número de años es entonces 360; como en total son 10 empleados, para encontrar la edad promedio de todos los trabajadores es necesario efectuar encontrando así que la opción correcta es C. El 27% de los estudiantes resolvió correctamente la pregunta, mientras que un 43% seleccionó la opción B que resulta de hallar el promedio de las edades promedio, de mujeres y hombres, sin tener en cuenta el número de trabajadores del depar-tamento. Los estudiantes que seleccionan las opciones A, o, D, posiblemente, no entienden las condiciones del problema y escogen opciones en las cuales aparecen las edades promedio de hombres o mujeres del enunciado de la pregunta. Es necesario que en las aulas se sigan trabajando actividades de profundización que permitan al estudiante diferenciar y aplicar correctamente los conceptos de promedio y promedio ponderado.
  • 44. 4 ANÁLISIS DE RESULTADOS DE LAS PRUEBAS DE ESTADO Contexto para el ejemplo 28 Diseño de placas El Ministerio de Transporte es la Institución en Colombia encargada de diseñar y establecer las características de la placa única nacional para los vehículos auto-motores. A partir de 1990 las placas tienen tres letras y tres dígitos, debajo llevan el nombre del municipio donde se encuentra matriculado el vehículo. Para la fabri-cación de las placas se utilizan 27 letras y 10 dígitos. La empresa que fabrica las placas ha comprobado que de una producción de 100 placas fabricadas aproxima-damente 5 placas tienen algún defecto. Ejemplo 28, núcleo común La primera letra de la placa de los carros particulares matriculados en Bogotá es A o B. El número total de placas que pueden fabricarse para identificar carros particulares matriculados en Bogotá es A. 272 x 103 B. 273 x 102 C. 2 x 272 x 102 D. 2 x 272 x 103 Componente: Aleatorio Porcentaje por opciones de respuesta Competencia: Solución de problemas A B C D Clave: D 25 25 28 22 El ítem indaga por la formulación y resolución de problemas usando conceptos básicos de conteo (combinaciones, permu-taciones, arreglos condicionados) y explora el desarrollo y aplicación de diversas estrategias para resolver problemas. Para encontrar la respuesta correcta el estudiante debía realizar un argumento como el siguiente: como la primera letra es A o B entonces solo se tienen dos posibilidades de selección, para las otras letras se tienen 272 opciones y para seleccionar los números 103 maneras. En total el número de placas posibles en Bogotá es 2 x 272 x 103. Los porcentajes de elección de las opciones de respuesta se distribuyeron de manera similar –alrededor del 25%–, con una leve preferencia por la opción C. Esto parece indicar que la selección o bien fue realizada al azar o los estudiantes intentaron realizar un conteo pero no se precisaron las posibilidades para cada posición o no se entendieron bien. Se esperaría que en una pregunta que requiere un conteo sencillo el porcentaje de estudiantes que responda correctamen-te fuera mucho mayor, aunque en los últimos años se han ido incorporando estos temas en los currículos, es necesario profundizar más en estos y de ser posible incluirlo desde el grado octavo o noveno de manera que se llegue a ejercicios que requieran técnicas de conteo más sofisticadas en los cursos superiores. Este tipo de temas, además, tienen muchas aplicaciones en situaciones de la vida cotidiana que pueden ser aprovechadas para la correcta apropiación.
  • 45. 45 ANÁLISIS DE RESULTADOS DE LAS PRUEBAS DE ESTADO Contexto para los ejemplos 29, 30 y 31 Mundiales de fútbol Cada cuatro años la FIFA (Federation International Football Association) rea-liza el Campeonato Mundial de Fútbol en el que participan 32 selecciones. Las 32 selecciones se distribuyen mediante un sorteo, en 8 grupos de 4 equi-pos cada uno. Para evitar el enfrentamiento entre favoritos, en la primera ronda eliminatoria los 8 equipos considerados como los mejores se asignan como cabeza de grupo. En la primera ronda cada equipo juega una vez contra cada uno de los demás equipos de su grupo y se eliminan dos equipos de cada grupo. Entre los 16 clasificados se eliminan 8 y en la siguiente ronda se eliminan 4. Entre los 4 que quedan se determina el campeón, subcampeón, tercero y cuarto. Ejemplo 29, núcleo común Si en la primera ronda de un campeonato, en uno de los grupos el promedio de goles anotado por partido fue de 2,5 goles, el total de goles anotados en ese grupo fue A. 10 B. 15 C. 20 D. 24 Componente: Aleatorio Porcentaje por opciones de respuesta Competencia: Razonamiento A B C D Clave: B 42 27 21 10 La pregunta está relacionada con estándares asociados a la utilización comprensiva de conceptos básicos de conteo y de medidas de tendencia central, en este caso con el concepto de promedio. Teniendo como contexto una lectura referente a los mundiales de fútbol, el ítem indaga por el número de goles anotados, durante la primera ronda de un campeonato mundial en uno de los grupos, cuando se sabe el número de equipos de cada grupo y el promedio de goles por partido. Para dar solución al problema, el estudiante debe, inicialmente, determinar que en el grupo se jugaron 6 partidos puesto que en cada grupo hay 4 equipos y cada uno de ellos juega 1 vez contra los otros, este conteo puede realizarse mediante la construcción del listado de partidos
  • 46. 46 ANÁLISIS DE RESULTADOS DE LAS PRUEBAS DE ESTADO Por lo tanto en total se juegan 6 partidos. O de manera más formal, encontrando el número de combinaciones sin repetición que se pueden hacer con cuatro equipos para formar grupos de 2 Posteriormente, teniendo en cuenta que el promedio de goles por partido fue de 2,5 y que se jugaron 6 partidos debe efectuar 2,5 x 6 para concluir que el número de goles anotados en este grupo fue de 15. Tal vez, por ser una pregunta cuya respuesta no es inmediata pues la estrategia de solución exige tanto la aplicación de diferentes conceptos asociados con el pensamiento aleatorio, como la aplicación de un proceso de reversibilidad (el rela-cionado con el concepto de promedio) solamente un 27% de los estudiantes seleccionó B la opción correcta. Los estudiantes que seleccionaron otras opciones, posiblemente no entendieron las condiciones del problema, pues como puede observarse éste no presenta dificultad de tipo operatorio. Un alto porcentaje de estudiantes, el 42%, seleccionó la opción A, que resulta de multiplicar el promedio de goles por el número de equipos del grupo. El 21% seleccionó la opción C que resulta de adicionar algunos números que aparecen en la lectura inicial, Los estudiantes que seleccionaron la opción D, el 10%, multiplicaron el número de partidos –6- por el número de equipos de cada grupo –4-.
  • 47. 47 ANÁLISIS DE RESULTADOS DE LAS PRUEBAS DE ESTADO Ejemplo 30, núcleo común La probabilidad de que en un mundial el equipo campeón, no sea uno de los equipos cabeza de grupo es A. 7 8 1 B. 8 C. 3 4 D. 1 4 Componente: Aleatorio Porcentaje por opciones de respuesta Competencia: Solución de problemas A B C D Clave: C 14 32 22 31 Este ítem está relacionado con el cálculo de la probabilidad de eventos y el uso de conceptos básicos de probabilidad. Exige el planteamiento y solución de un problema a partir de situaciones o contextos externos a la matemática. Para resolverlo el estudiante debe tomar el total de los equipos 32 y restar los 8 cabeza de grupo, quedan 24 (casos favo-rables al evento: no ser cabeza de grupo). La probabilidad es entonces por lo tanto la opción correcta es la C que fue elegida por el 22% de los estudiantes. El 32% de los estudiantes seleccionó la opción B posiblemente pensando que como hay 8 equipos cabeza de grupo cada uno tiene de probabilidad de ser campeón, no interpretan la condición del problema o simplemente los atrae el 8, algo similar pudo pasar con el 14% que selecciono A. Respecto a la opción D hubo un porcentaje alto que pensó en la probabilidad del complemento , pero, en este caso no interpretan la negación, y calculan la probabilidad de que un equipo cabeza de grupo sea campeón. Nótese que se indaga por una probabilidad muy fácil de calcular, que debería resultar familiar para los estudiantes, sin embargo es probable que la dificultad aumente por el tipo de contexto empleado, que si bien es conocido, seguramente no se usa para indagar por conceptos de probabilidad. Es importante insistir que la noción de probabilidad debe introducirse en los primeros grados, de tal manera que un estudiante al terminar su secundaria pueda además resolver problemas de probabilidad condicional y a través del uso de herramientas más sofisticadas.
  • 48. 48 ANÁLISIS DE RESULTADOS DE LAS PRUEBAS DE ESTADO Ejemplo 31, núcleo común En la siguiente gráfica se muestra el número total de partidos jugados y el número total de goles anotados en algunos de los campeonatos mundiales de fútbol. . El promedio de goles por partido fue mayor en el campeonato mundial de A. España 82. B. México 86. C. Italia 90. D. Francia 98. Componente: Aleatorio Porcentaje por opciones de respuesta Competencia: Comunicación A B C D Clave: A 19 2 6 72 Utilizando como contexto información numérica que aparece en un diagrama de barras, referente al número de goles y al número de partidos jugados en diferentes campeonatos mundiales de fútbol, la pregunta indaga por el campeonato con mayor promedio de goles. Esta pregunta explora la capacidad del estudiante para interpretar diferentes tipos de represen-tación de información numérica, específicamente un diagrama de barras; está asociada con estándares relacionados con conceptos de medidas de tendencia central. Para dar solución a la pregunta, el estudiante debe aplicar el concepto de promedio hallando el mayor de los cocientes obtenidos al dividir el número de goles entre el número de partidos, en cada uno de los campeonatos mundiales, aunque basta con hacerlo con los datos de los mundiales que aparecen en las opciones de respuesta. Una correcta interpretación de la información le permite descartar los campeonatos de Italia 90 y México 86 en los cuales se jugó el mismo número de partidos que en España 82 pero con menos goles en total. De manera que para seleccionar la respuesta correcta debe determinar que el cociente 146/52 correspondiente al promedio de goles en España 82 es mayor que el cociente 171/64 correspondiente a Francia 90. Únicamente el 19% de los estudiantes interpretó la información y aplicó correctamente el concepto de promedio seleccio-nando España 82, la opción correcta. La mayoría de los estudiantes, el 72%, seleccionó la opción D Francia 98, es posible
  • 49. 49 ANÁLISIS DE RESULTADOS DE LAS PRUEBAS DE ESTADO que hayan asociado el mayor número de goles al mayor promedio, es decir, consideraron que por ser la barra del número de goles la más alta, este campeonato era el de mayor promedio de goles. Esto muestra que si bien se está trabajando en las aulas la lectura de gráficas de barras, se hace de manera muy inicial y tradicional, es decir traducir el valor que muestra la barra, pero es necesario introducir más problemas que requieran usar la información para llegar a otro tipo de conclusiones y que involucren otros conceptos. 4. Conclusiones y recomendaciones  El promedio de la prueba en ambos calendarios ha seguido aumentando en los últimos dos años, destacándose especialmente calendario B con aproximadamente 5 puntos, aunque también es el calendario en el que más aumenta la dispersión. Esto muestra que el incremento del promedio se debe a que un grupo de personas obtiene mejores resultados pero se abre más la brecha entre altos y bajos puntajes. Como se dijo en el documento de análisis de resultados del año 2005, las practicas de aula deben encaminarse a promover desarrollos significativos en toda la población.  Aproximadamente el 95% de los estudiantes siguen ubicándose en el nivel medio y más del 50% de la población sólo alcanza puntajes hasta de 50 puntos y más del 90% llega sólo hasta 65 puntos. Esto ratifica que aunque el promedio nacional se ha incrementado, una gran mayoría de la población se ubica en puntajes de la categoría media. Es necesario seguir trabajando en las aulas en el fortalecimiento de conceptos básicos de la matemática escolar, conceptos que deben ser claros para cualquier estudiante que termina su secundaria y que debe darle la posibilidad de usarlos de manera significativa en diferentes situaciones, como las propuestas en el examen. Sola-mente mediante el fortalecimiento de los aspectos básicos será posible que los estudiantes se enfrenten con éxito a situaciones no rutinarias que les exigen poner en práctica diferentes conceptos, relacionarlos y utilizar diversas estrategias de solución para llegar a la respuesta correcta.  En profundización, por ser una prueba opcional se esperaría que los resultados fueran más altos. Sin embargo, en el 2006 el 13.5% de los estudiantes de calendario B y el 34% de los de calendario A quedaron ubicados en grado básico, es decir no alcanzaron los requisitos mínimos del primer grado. En contraste tan sólo el 15% de los estu-diantes de calendario B y el 2% de los de calendario A se ubicaron en el grado superior, grado III.  Respecto al componente numérico-variacional, se aprecia un manejo adecuado de información numérica presen-tada en gráficas o tablas, pero un número importante de los estudiantes no dan significado alguno al concepto, ni diferencian propiedades de los sistemas numéricos. Es el caso por ejemplo de los números reales, las preguntas referidas a su representación y estructura resultaron de especial complejidad. Se requiere un trabajo más cuidadoso con este concepto, que va mas allá de la introducción prematura de una árida clasificación. No hay que olvidar que este concepto es justamente uno de los pilares para la estructuración del pensamiento variacional al finalizar la educación media. El conocimiento que se pone de manifiesto respecto a los modelos funcionales se reduce generalmente al modelo lineal, los otros modelos generan problemas en el análisis y la representación. Sin embargo, las preguntas que indagan sobre variación lineal revisten dificultad para un grupo importante, este es un punto que merece atención, desde los grados de la básica, no es pertinente introducir listados de funciones diversas para manipular algebrai-camente sin que se haya logrado la apropiación del modelo fundamental.  Respecto al componente geométrico-métrico los estudiantes reconocen figuras planas, usan algunas de sus propie-dades y determinan áreas de figuras simples. Sin embargo tienen dificultades cuando se trata de aplicar resultados básicos como el teorema de Pitágoras en contextos no rutinarios, es necesario proponer más y variados problemas de aplicación en diversidad de situaciones, con miras a que el estudiante profundice en los conceptos y relaciones. Es de anotar que los teoremas básicos de semejanza y los del seno y el coseno se presentaron explícitamente (en profundización) con el objeto de apreciar la interpretación y aplicación, pero este análisis resulto muy complejo. Sería
  • 50. 50 ANÁLISIS DE RESULTADOS DE LAS PRUEBAS DE ESTADO pertinente que en las aulas se planteen situaciones en las que se propongan definiciones, se analicen enunciados diversos donde se expresen condiciones, negaciones y se trabaje con proposiciones diversas relativas al pensamiento espacial. Es importante destacar que las herramientas de la lógica formal se construyen al interior de cada uno de los pensamientos no esquemáticamente en apartes dispersos del currículo.  En cuanto al componente aleatorio se refiere, se aprecia que en las aulas se está trabajando la lectura de gráficas, el uso de información tabular y gráfica y algunos aspectos relativos a las medidas de tendencia central, especialmente el promedio. Pero posiblemente falta ir más halla del cálculo, pues los estudiantes evaluados presentan dificultad para usar información y llegar a conclusiones, y en las preguntas que lo requerían tienen problemas para dar significado a las medidas de tendencia central y discutir su pertinencia. Un aspecto que posiblemente no se trabaja en las aulas y es muy importante en el desarrollo del pensamiento aleatorio, por la diversidad y riqueza de aplicaciones, es el relacionado con las nociones iniciales de conteo. Los problemas planteados sobre este tópico que se consideraban de un primer nivel de dificultad resultaron muy complejas, es posible que esto este relacionado con la poca o inexistente experiencia de trabajo con este tipo de tarea. Algo similar sucedió con las nociones iniciales de probabilidad, es importante que los docentes revisen los currículos, su coherencia con los Estándares Básicos de Competencias y que todos los aspectos mencionados sean ampliamente trabajados en los diferentes grados. 5. Bibliografía ICFES, Grupo de Evaluación de la Educación Básica y Media, Serie Examen de Estado para Ingreso a la Educación Superior, Análisis de Resultados 2005, Matemáticas por ACEVEDO, Myriam y otros. Bogotá, Octubre de 2006. ICFES, Grupo de Evaluación de la Educación Básica y Media. ¿Qué evalúan las pruebas 2006?. Documento interno, página www.icfes.gov.co MEN, Serie Lineamientos Curriculares, Matemáticas. Bogotá, 1998. MEN, Documento N°3, Estándares Básicos de Competencias en Lenguaje, Matemáticas, Ciencias y Ciudadanas. Bogotá, 2006.